Neurology & Pupils (No 467)

Download as docx, pdf, or txt
Download as docx, pdf, or txt
You are on page 1of 231

Neurology & Pupils (No=467)

Kjer optic atrophy is characterised by all of the following EXCEPT:

visual loss is usually slowly progressive throughout life

autosomal dominant inheritance

onset between 5 and 10 years of age

tritanopia

Your answer was INCORRECT


Explanation
Kjer optic neuropathy (dominant optic atrophy) is characterised by:

autosomal dominant inheritance


manifests between 5 and 10 years of age
insiduous onset of visual loss
visual loss may progress until the mid-teens or 20's at which point it often stabilizes
usually mild degree of visual dysfunction (6/18 or better), absence of night blindness and
absence of acute or sub-acute progression
 ongoing progression occurs in only 1 third of families (reason unknown)
 tritanopia
Reference: Newman et al. Hereditary optic neuropathies. Eye (2004) 18, 1144–1160.

If cold water is irrigated in the left ear of an awake patient, in what direction is the slow phase of
the nystagmus?

down

right

left

up

Your answer was CORRECT


Explanation
The mnemonic COWS (cold opposite; warm same; in awake patient) is useful and gives the
direction of the fast phase of the nystagmus. In this case, the left eye is irrigated with cold water,
so the fast phase will be toward the right and slow phase to the left.

Characteristics of congenital motor nystagmus include each of the following EXCEPT:


affected patients typically have visual acuity ranging from 6/6 to 6/18
a null point may exist
the nystagmus does not occur while the patient is asleep
oscillopsia is frequently present

Your answer was CORRECT


Explanation
Characteristics of congenital motor nystagmus:
 conjugate, jerk or pendular nystagmus
 onset in perinatal period
 visual acuity relatively good
 no change in nystagmus with unilateral occlusion or blurring (compare to manifest-latent)
 fast phase switches: to right in right-gaze, to left in left-gaze
 null point can occur in any position of gaze
 compensatory head postures vary by patient: face-turns either way or chin up, chin down
depending on the position of the null point
 titubation
 tends to dampen with convergence, darkness, sleep, when eye is covered
 increases with fixation
 paradoxical OKN response
Characteristics of manifest-latent nystagmus:
 bilateral
 becomes manifest when one eye occluded, blurred or intermittently suppressed
 jerk-type nystagmus
 null point in adduction
 fast phase towards fixing eye
 face turn towards the fixing eye dampens nystagmus
 associated with interruptions to binocular development: congenital esotropia, but also
monocular congenital cataracts

A patient presents with a “down-and-out” left eye, left ptosis and left-sided dysdiadokinesia
and dysmetria.

What is the diagnosis?


Nothnagel's syndrome
Weber's syndrome
Foville's syndrome
Benedikt's syndrome
Claude's syndrome

Your answer was CORRECT


Explanation
The features described are in keeping with Nothnagel syndrome.

Third nerve fascicle syndromes:

Benedikt's syndrome, which is characterised by: 1

 ipsilateral CN III palsy


 contralateral cerebellar ataxia with intention tremor
 contralateral hemiparesis (usually partial) with hyperactive tendon reflexes
Claude syndrome, which is characterised by: 2

 ipsilateral 3rd nerve palsy


 contralateral cerebellar ataxia
Weber syndrome, which is characterised by:
 ipsilateral 3rd nerve
 contralateral hemi-paresis
Nothnagel syndrome, which is characterised by:
 ipsilateral 3rd nerve
 ipsilateral cerebellar ataxia

1. Reference for Benedikt syndrome


2. Reference for Claude syndrome
3. General reference for 3rd nerve syndromes

n which condition can psammoma bodies be found?

optic nerve glioma

Kaposi's sarcoma

pleomorphic adenoma
Merkel cell carcinoma

optic nerve sheath meningioma

Your answer was CORRECT


Explanation
Psammoma bodies are a histological finding in meningioma.

Which of the following signs will help distinguish congenital from acquired Horner's
syndrome?

ptosis which cannot be overcome by frontalis overaction

failure of pupil to dilate to 1:1000 adrenaline

iris heterochromia
facial asymmetry

Your answer was CORRECT


Explanation
Sympathetic innervation plays a role in the development of pigmentation of the iris.
Congenital interruption of the sympathetics to one eye will result in the ipsilateral iris having
less pigment than the fellow eye.
Note: facial asymmetry is a feature of congenital fourth nerve palsy.

What is the antidote for the crisis caused by an overdose of edrophonium (Tensilon)?

verapamil

atropine

dantrolene

propranolol

epinephrine

Your answer was CORRECT


Explanation
Tensilon prolongs the effect of acetylcholine in the synaptic cleft, causing a cholinergic crisis
(sweating, nausea and vomiting, salivation, fever). Atropine blocks acetylcholine receptor
sites on the postsynaptic terminal.

Which statement regarding the extraocular muscles is FALSE?

the inferior oblique is innervated by the ipsilateral III nucleus

the superior rectus is innervated by the contralateral III nucleus

the levator palpebrae is innervated by a fused central nucleus

the superior oblique is innervated by the ipsilateral IV nucleus

Your answer was CORRECT


Explanation
The levator palpebrae are innervated by a fused central nucleus. Therefore, unilateral ptosis as
a result of a nuclear lesion is not possible.

Which segment of the optic nerve is the longest:


intracranial
intracanalicular
intraocular
intraorbital

Your answer was CORRECT


Explanation
The optic nerve is 45mm long and is divided into the following segments:
 intraocular: 1mm
 intraorbital: 30mm
 intracanalicular: 6mm
 intracranial: 10mm

The yoke muscle of the left superior oblique is the:

right inferior rectus

left inferior oblique

right superior oblique

left inferior rectus

Your answer was CORRECT


Explanation
Yoke muscles are pairs of muscles in different eyes which produce conjugate, directional
gaze. Thus the yoke muscle of the left superior oblique (acting to cause the left eye to depress
while the globe is in adduction) is the right inferior rectus (causing the right eye to depress
while the globe is in abduction) for gaze down and right.

Synergists are pairs of muscles in the same eye which move the eye in the same direction (e.g.
right superior rectus and right inferior oblique)

Agonist-antagonists are pairs of muscles in the same eye that move the eye in opposite
directions (e.g. right lateral rectus and right medial rectus)

Which is FALSE regarding Argyll-Robertson pupil?


the pupil is miosed
the speed of constriction and dilatation is normal
occurs typically in elderly patients
cocaine 4-10% has no effect on the pupil size
caused by a lesion in the ventral midbrain

Your answer was CORRECT


Explanation
Argyll-Robertson pupil is due to a lesion in the dorsal midbrain. Specifically, the pretectal
interneurons to the Edinger Westphal nucleus are involved while sparing ventrally located
accommodative reflex neurons; hence the light-near dissociation seen.

Features of Argyll-Robertson pupils:


 small irregular pupils
 typically bilateral
 asymptomatic
 caused by lesion in dorsal midbrain
 light-near dissociation (responds better to near stimulus than light)
 respond poorly to pharmacological dilation

Causes of Argyll-Robertson-like include:


 syphilis (traditional cause)
 MS
 alcoholism
 trauma
 surgery
 aberrant 3rd nerve regeneration
 SLE
 HZO

Which muscle is most effective as a depressor of the eye when it is abducted 23 degrees from
the midline?

superior oblique

inferior oblique

inferior rectus

medial rectus

Your answer was CORRECT


Explanation
Figure: Schematic showing the cardinal positions of eye movement in which the 6 extra-
ocular muscles are best isolated. Thus supero-lateral gaze is the best position to isolate and
assess the superior rectus muscle

The primary action of the inferior rectus is depression of the globe. This effect is isolated
when the eye is abducted 23 degrees from the midline.

The secondary actions of the inferior rectus are adduction and extorsion. At 67 degrees of
adduction the inferior rectus acts only as an extorter.

Which investigation is LEAST likely to reveal helpful information in a malingerer?


Frisby test
Goldmann perimetry
red-green duochrome
Humphrey field test

Your answer was CORRECT


Explanation
Malingerers can fake a Humphrey field test, even in the presence of reliability indicators.

Goldmann perimetry may be useful in malingerers by revealing spiraling or crossing of the


isopters.

Frisby stereoscopic testing requires both eyes to resolve a disparity, and can be useful in
unmasking malingerers who complain of dramatic unilateral vision loss.

Red-green duochrome is also useful for malingerers complaining of severe unilateral vision
loss. It induces a patient to read with an eye that supposedly cannot see by making the patient
think that he is using both eyes. The eye behind the red lens will see letters on both sides of
the chart while the eye behind the green lens will only see those letters on the green side.

Other useful tests for malingerers include:


 the mirror test: moving a mirror in front of the patient and looking for eye movements
 the threatening stimulus test: looking for a blink reaction to a threatening stimulus such as
a pin or flicking a finger at close range
 the 10 PD base out or base up test: looking for appropriate compensatory eye movements

A 76-year-old diabetic man has diplopia on upgaze. On ocular motility testing, abduction and
adduction are full bilaterally, but on upgaze the left eye only elevates halfway up. There is a
partial left ptosis. His pupils and the remainder of the ocular examination appear normal. You
arrange an MRI brain and orbits.

Where would you expect the lesion in this case?

cavernous sinus

juncture of the posterior communicating and internal carotid arteries


orbital apex

brainstem

Your answer was CORRECT


Explanation
This patient has a left superior division third nerve palsy. The only place that an oculomotor
superior division nerve palsy could occur without affecting additional cranial nerves or the
inferior division of cranial nerve III is in the orbital apex. Posterior to the orbital apex, the
superior and inferior divisions are joined, and a compressive lesion selective to one division
would be very improbable.

Which auto-antibody is most commonly found in patients with generalised myasthenia gravis?

antibodies binding pre-synaptic calcium channels

antibodies binding acetylcholine

antibodies binding acetylcholinesterase

antibodies binding acetylcholine receptors

Your answer was CORRECT


Explanation
Anti-ACh receptor antibodies are found in 90% of patients with generalised myasthenia
gravis.

Optic disc drusen is associated with:


retinitis pigmentosa
giant cell arteritis
migraines
normal tension glaucoma
pseudotumor cerebri

Your answer was CORRECT


Explanation
Figure: Optic disc drusen as seen on autofluorescence imaging.

Optic disc drusen may be caused by:


 idiopathic (often familial in an AD pattern)
 pseudoxanthoma elasticum
 angioid streaks
 retinitis pigmentosa

The parasympathetic fibers to the lacrimal gland synapse in which ganglion?

ciliary

superior cervical

sphenopalatine

geniculate

Your answer was CORRECT


Explanation
The parasympathetic fibres to the lacrimal gland synapse in the sphenopalatine (or
pterygopalatine) ganglion.

The geniculate ganglion contains the cell bodies that provide the sense of taste from the
anterior two-thirds of the tongue.

The ciliary ganglion is the intraorbital location where the primary parasympathetic fibers from
the Edinger-Westphal nucleus synapse with the secondary parasympathetic nerves that
innervate the ciliary body and iris sphincter muscle to provide accommodation and
constriction of the pupil.

The superior cervical ganglion contains the cell bodies of the sympathetic fibers, which
provide innervation to the superior tarsal muscle, the pupillary dilator muscle, facial blood
vessels, skin and sweat glands.

Which of the following extraocular muscles can perform intortion of the globe:
the medial rectus
the superior oblique
the inferior rectus
the inferior oblique

Your answer was CORRECT


Explanation
The primary, secondary and tertiary actions of the recti are respectively:
 Medial rectus: Adduction (no secondary or tertiary actions)
 Superior rectus: Elevation, Intortion, Adduction
 Inferior rectus: Depression, Extorsion, Adduction
 Inferior oblique: Extorsion, Elevation, Abduction
 Superior oblique: Intorsion, Depression, Abduction
 Lateral rectus: Abduction (no secondary or tertiary actions)

Damage to the cerebellum results in:

hemiballismus

difficulty in initiating and terminating movements

ipsilateral ataxia

auditory hallucinations

falling towards the contralateral side

Your answer was CORRECT


Explanation
A cerebellar lesion will cause nystagmus and ataxia towards the ipsilateral side.
Hemiballismus will occur in a lesion of the subthalamic nucleus. Difficulty in initiating and
terminating movements occurs in Parkinsonism. Auditory hallucinations do not occur with an
isolated cerebellar lesion.

Which structure(s) lie immediately posterior to the optic chiasm:


pituitary stalk
internal carotid artery
anterior communicating arteries
lamina terminalis

Your answer was CORRECT


Explanation
The relations of the optic chiasm are as follows:
 inferior: diaphragma sellae
 superior: lamina terminalis
 anterior: anterior cerebra and communicating arteries
 posterior: pituitary stalk
 lateral: internal carotid artery

Which of the following is FALSE regarding Leber's hereditary optic atrophy:

it is the result of point mutations

it is mitochondrially inherited

the papillomacular bundle is the last to be affected

the enzyme affected is ATPase 6

it results in demyelination of the optic nerve

Your answer was CORRECT


Explanation
LHOA affects the papillomacular bundle most severely. Other options are true.

What vitamin supplementation is required for a patient with pernicious anaemia-related optic
neuropathy?

Vitamin B1

Vitamin A
Vitamin B6

Vitamin E

Vitamin B12

Your answer was CORRECT


Explanation
Pernicious anaemia is an auto-immune disease where loss of parietal cells in the gastric lining
causes malabsorption of vitamin B12.

Which of the following would NOT normally be found in congenital nystagmus?

amplitude dampened by convergence

paradoxic optokinetic nystagmus

oscillopsia

amplitude increased by fixation

normal visual acuity

Your answer was CORRECT


Explanation
Oscillopsia (the sensation of the world moving) is not a feature of congenital nystamus.

Characteristics of congenital nystagmus:


 positive family history often present
 presents in first 4 months of life
 jerk or pendular nystagmus
 normal or near-normal visual acuity
 no change in nystagmus with unilateral occlusion or blurring (compare to: manifest-latent
nystagmus)
 fast phase switches: to right in right-gaze, to left in left-gaze
 null point can occur in any position of gaze
 compensatory head postures vary by patient: face-turns either way or chin up, chin down
depending on the position of the null point
 titubation
 tends to dampen with convergence, darkness, sleep, when eye is covered
 increases with fixation
 paradoxical OKN response
 nystagmus blockage syndrome may occur: associated with esotropia
A patient presents with bilateral sixth nerve palsy. Where is the most likely location of nerve
compression?
the pons at the level of the abducens nuclei

the clivus

the cavernous sinus

the cerebello-pontine angle

Your answer was CORRECT


Explanation
The most common cause of a bilateral sixth nerve palsy is raised intracranial pressure. The
sixth nerve travels through the subarachnoid space where it ascends the clivus and enters the
cavernous sinus. Within the subarachnoid space, the sixth nerve can be stretched against the
clivus as the brain stem herniates through the foramen magnum due to increased intracranial
pressure, which can cause a bilateral sixth nerve palsy.

This question came in the FRCS (Glasgow) Part 2 exam in October 2014.

Which is FALSE regarding light reflection tests?


the Hirschberg test is based on Purkinje image 2
an abnormal light reflection test may be due to eccentric fixation
in Bruckner's test the duller reflex comes from the fixing eye
a Hirschberg reflex on the pupil border suggests a squint of 30 PD

Your answer was CORRECT


Explanation
The Hirschberg test is based on Purkinje image 1. Other options above are true.

Regarding the Hirschberg corneal reflection test:

1 mm displacement = approximately 7 degrees = 15 PD


 Displacement to pupil edge = 2 mm = 15 degrees = 30 PD
 Displacement midway from pupil to limbus = 4mm = 30 degrees = 60PD
 Displacement to the limbus = 6mm = 45 degrees = 90PD

The clinical finding that all three types of Duane's syndrome share is:
exotropia

globe retraction with adduction

esotropia

a deficit of adduction
a deficit of abduction

Your answer was CORRECT


Explanation
Duane's syndrome has various presentations but retraction of the adducted globe appears most
consistently.
The three types of Duane's syndrome are distinguished by the relative ability to adduct or
abduct:
 Type 1, limited abduction but full adduction
 Type 2, normal abduction but limited adduction
 Type 3, both abduction and adduction are limited.

Duane's retraction syndrome (DRS) is characterised by:


 unilateral or bilateral abnormality of horizontal gaze
 caused by co-contraction of medial and lateral recti
 retraction of the globe on attempted adduction
 upshooting or downshooting of the globe on adduction
 the left eye is affected more frequently than the right
 females are affected more frequently than males
 Type 1: abduction limitation greater than adduction, eso (most common)
 Type 2: adduction limitation greater than abduction, exo (least common)
 Type 3: abduction and adduction limited equally
 amblyopia in 10%

Associations with Duane's retraction syndrome:


 cataracts
 iris anomalies
 Marcus Gunn jaw winking
 microphthalmos
 crocodile tears
 Goldenhar's syndrome
 maternal thalidomide
 Klippel-Feil syndrome

A patient suffers a severe road traffic accident requiring intensive care admission but makes a
good recovery. He complains of excessive sweating and flushing every time he eats.

What is the diagnosis?


abberant regeneration of the fifth cranial nerve
crocodile tears
Frey's syndrome
autonomic dysregulation

Your answer was CORRECT


Explanation
Aberrant regeneration may occur after trauma or tumour.

The clinical picture above describes Frey's syndrome, where there is aberrant regeneration of
the 9th cranial nerve, with synkinesis between salivation fibres and sympathetic fibres. This
causes flushing and sweating when eating.

Other aberrant regeneration syndromes:


 Crocodile tears: tearing when eating. Abberant regeneration of the seventh cranial nerve
with synkinesis between salivation and lacrimation fibres.
 Aberrant third nerve regeneration:
o inverse Duane's sign: lid elevation on adduction
o pseudo-von Graefe sign: lid elevation on depression
o pseudo-Argyl-Robertson: pupil constriction on adduction
o pupil constriction on depression

Which of the following would be the best initial choice for prophylaxis of acute, severe
migraine headache?

paracetamol

methysergide

aspirin

propranolol

sumatriptan

Your answer was CORRECT


Explanation
Methysergide is an ergot alkaloid which can be used for migraine prophylaxis. However, there
are numerous potential side effects, including retro-peritoneal fibrosis which limits its routine
use. Beta-blockers such as propranolol are effective and safer in migraine prophylaxis.

Sumatriptan is a serotonin agonist used to halt an acute migraine attack.

Which of the following is MOST likely to cause an optic neuropathy?


tamoxifen
gold
cidofovir
amiodarone

Your answer was CORRECT


Explanation
Drugs which cause optic neuropathy include:
 ethambutol
 isoniazid
 amiodarone
 vigabatrin
 chloramphenicol
 hydroxchloroquine
 penicillamine
 cisplatin
 vincristine
Tamoxifen causes a crystalline maculopathy, cidofovir causes uveitis, while gold can cause
corneal deposition.

This question came in the 2014 FRCOphth.

Which of the following congenital optic disc anomalies is most closely associated with glial
proliferation and folding of the retina?

hyaloid remnant

optic disc coloboma

morning glory

optic nerve hypoplasia

optic nerve pit

Your answer was CORRECT


Explanation
Morning glory disc anomaly is characterised by an excavated optic nerve head with overlying
glial proliferation. The surrounding retina is often thrown into folds. There is an association
with retinal detachment. Most cases are unilateral.

Which of the following medications is most clearly associated with drug-induced myasthenia?
penicillamine
warfarin
methotrexate
diltiazem
ranitidine

Your answer was CORRECT


Explanation
Drugs that can cause a drug-induced myasthenia (and which should therefore be avoided in
patients with myasthenia) include:
 aminoglycosides (e.g. gentamicin)
 neuromuscular junction blockers (e.g. suxamethonium)
 chlorpromazine
 respiratory suppressants (e.g. morphine)
 procainamide
 penicillamine

A 35-year-old woman presents with a 3-day history of reduced vision left eye. She has noticed
a dull left periorbital ache for 1 week, which is worse on eye movement. On examination,
visual acuity is RE 6/6, LE 6/36. There is a left RAPD.

What is the most appropriate next step?


oral prednisone

MRI

IV methylprednisolone

electrodiagnostics

Your answer was CORRECT


Explanation
Clinically, this patient most likely has optic neuritis (ON). The presence of periventricular
plaques on MRI increases her risk of developing multiple sclerosis. The presence or absence
of plaques is important to establish when counseling the patient regarding her future risk of
MS, but perhaps more importantly has implications for treatment of her optic neuritis. Higher-
risk ON patients (those with multiple plaques), who receive IV methylprednisolone have a
reduced rate of developing multiple sclerosis over the next 2 years. For patients at lower risk
(no plaques), observation of ON could suffice. Treatment of ON with oral prednisone alone
increases the rate of recurrence of optic neuritis and is thus contraindicated, as per the ONTT
(optic neuritis treatment trial).

All of the following medications can cause optic neuropathy EXCEPT:

ethambutol

ganciclovir

vigabatrin

isoniazid

Your answer was CORRECT


Explanation
A large number of medications have been associated with optic neuropathy. However,
ganciclovir has not been implicated.
Causes of drug-induced optic neuropathy include:
 ethambutol
 chloramphenicol
 linezolid
 erythromycin
 streptomycin
 anti-retrovirals
 amiodarone
 infliximab
 clioquinol
 dapsone
 quinine
 pheniprazine
 suramin
 isoniazid

A man wakes up from a coma with bilateral complete ophthalmoplegia involving the pupils
and visual disturbance. His motor function in arms and legs is otherwise preserved.

In which vessel is the embolus?


internal carotid artery
posterior inferior cerebellar artery
posterior communicating artery
basilar artery

Your answer was CORRECT


Explanation
The ‘top of the basilar artery’ syndrome is caused by an embolus at the top of the basilar
artery, which causes ischaemia of the rostral brainstem, thalamus and portions of the cerebral
hemispheres fed by the posterior cerebral arteries.
Features include:
 altered consciousness
 behavioural abnormalities
 visual abnormalities
 oculomotor palsy involving the pupils
 No significant motor weakness in the extremities

This question appeared in the 2014 FRCOphth Part 2.

Which feature is universal in internuclear ophthalmoplegia:


skew deviation
small-angle exotropia
vertical nystagmus
ipsilateral adduction deficit

Your answer was CORRECT


Explanation
INO is characterised by the triad of:
 failure of ipsilateral adduction (often best seen with saccades)
 ataxic nystagmus of abducting contralateral eye
 normal convergence (if posterior INO)
However, other features can include:
 slowing of saccades in adducting eye
 asymmetric OKN response when drum rotated towards MLF lesion
 horizontal nystagmus of adducting eye
 impaired horizontal vestibulo-ocular reflex
 impaired convergence (for Cogan's anterior INO)
 manifest exotropia (for bilateral INO)
 vertical nystagmus
 impaired vertical pursuit
 impaired vertical vestibulo-ocular reflex
 difficulty maintaining upgaze

Which is TRUE regarding congenital fibrosis of the extraocular muscles?

there is rarely any limitation of adduction

the eyes are usually anchored in downgaze

the condition is usually progressive

the age of onset is typically 3-5 years of age

Your answer was CORRECT


Explanation
Patients are typically born with non-progressive ophthalmoplegia. The aetiology is abnormal
development of oculomotor subnuclei. The eyes are usually frozen in downgaze and abduction
with little or no ability to adduct, depress or elevate.

All of the following drugs can cause optic neuropathy EXCEPT:


isonioazid
rifampicin
ethambutol
chloramphenicol

Your answer was CORRECT


Explanation
Drugs that can cause optic neuropathy include:
 vigabatrin
 amiodarone
 hydroxychloroquine
 penicillamine
 cisplatin
 vincristine
 ethambutol
 isoniazid
 chloramphenicol

Which statement regarding the extraocular muscles is FALSE?

the inferior oblique is innervated by the ipsilateral III nucleus

the superior rectus is innervated by the contralateral III nucleus

the superior oblique is innervated by the ipsilateral IV nucleus

the levator palpebrae is innervated by a fused central nucleus

Your answer was CORRECT


Explanation
The levator palpebrae are innervated by a fused central nucleus. Therefore, unilateral ptosis as
a result of a nuclear lesion is not possible.

Ipsilateral nuclei innervate the inferior rectus, medial rectus, inferior oblique, and lateral
rectus muscles.

The nuclei controlling the superior rectus and superior oblique muscles have crossed
projections.

What percentage of patients with Bell's palsy will experience complete spontaneous recovery?

25%

50%

85%

5%

75%

Your answer was CORRECT


Explanation
Approximately 85% of patients with Bell's palsy spontaneously recover, although some will
have a degree of aberrant regeneration. Recovery usually begins within 3 weeks of diagnosis
and usually becomes complete by 2 to 3 months.

Afferent pupillary fibers exit the optic tract at:

the pretectal olivary nuclei

the medial longitudinal fasciculus

the frontal lobe

the occipital cortex

the lateral geniculate body

Your answer was CORRECT


Explanation
Pupillary fibers from the optic tract exit before reaching the lateral geniculate body and exit
into the pretectal olivary nuclei.

A 27-year-old man with Leber's hereditary optic neuropathy would like to start a family. He
asks the chances that his children would be affected?

100% of daughters are carriers while 0% chance for male offspring

0%

50%

100%

Your answer was CORRECT


Explanation
Leber's hereditary optic neuropathy (LHON) is caused by an abnormality in mitochondrial
DNA. Since mitochondrial DNA is only transmitted along the maternal line, men cannot pass
this disease to their offspring.

A 42-year-old woman with a long history of relapsing-remitting multiple sclerosis develops


double vision. On examination of her eye movements, abduction of either eye elicits
nystagmus in that eye; adduction is impaired in both eyes.

On MRI scanning, where will a new white matter lesion probably be evident:
cingulate gyrus

medial longitudinal fasciculus

optic chiasm

cerebellum

parietal lobes

Your answer was CORRECT


Explanation
Failure of adduction in both eyes signifies a bilateral internuclear ophthalmoplegia. This is
often accompanied by nystagmus of the abducting eye. The area of the brain affected is the
medial longitudinal fascicles in the brain stem which connects the third and sixth nerve nuclei.

Increased incidence of pheochromocytomas is a feature of:


Bourneville's syndrome
Louis-Bar syndrome
Sturge-Weber syndrome
von Hippel-Lindau syndrome

Your answer was CORRECT


Explanation
von Hippel-Lindau is characterised by:
 AD inheritance
 retinal capillary haemangiomas (can lead to exudative RD)
 cerebellar haemangioblastomas
 pancreatic, hepatic, and renal cysts
 renal cell carcinoma
 pheochromocytomas

An 83-year-old lady undergoes a total hip replacement. While convalescing on the ward, she
develops weakness and double vision. On examination, she has right facial weakness and
anaesthesia, an esotropia with failure of abduction of the right eye on doll's head movements.
There is failure of right gaze. She has a partial right ptosis and miosis.

What is the diagnosis?


Foville syndrome
Millard Gubler syndrome
Gerstmann syndrome
Raymond syndrome
Weber's syndrome
Your answer was CORRECT
Explanation
This patient has Foville syndrome, which is a syndrome of the sixth nerve fasciculus
characterised by:
 5, 6 and 7 nerve palsies
 PPRF causing an ipsilateral horizontal gaze palsy
 central Horner's syndrome

Which muscles are affected most commonly in thyroid eye disease?


inferior oblique, inferior rectus
lateral rectus, superior oblique
superior rectus, inferior oblique
medial rectus, inferior rectus

Your answer was CORRECT


Explanation
The order of involvement of ocular muscles in thyroid eye disease is (most common first):
 inferior rectus
 medial rectus
 superior rectus
 levator
 lateral rectus
Oblique muscles are rarely involved in TED.

A 69-year-old man develops acute weakness of the left side of his face. His forehead is not
affected.

Which of the following is the most likely cause:


lacunar infarction
parotid gland pathology
Bell's palsy
Lyme disease
sarcoidosis

Your answer was CORRECT


Explanation
Sparing of the forehead in a 7th nerve lesion implies that the lesion is an upper motor neuron
one. This is because the forehead is innervated bilaterally (i.e from both 7th nerve nuclei).
Therefore a lower motor neuron lesion will not spare the forehead.

Causes of a lower motor neuron facial nerve lesion:


 Parotid pathology
 Bell's (idiopathic)
 Sarcoidosis
 Lyme disease
 Trauma
 Mononeuritis multiplex

All of the following substances can cause congenital optic nerve hypoplasia with foetal
exposure EXCEPT:
LSD
alcohol
chloramphenicol
phenytoin
quinine

Your answer was CORRECT


Explanation
Optic disc hypoplasia can be associated with:
 septo-optic dysplasia (de Morsier's syndrome)
 aniridia
 fetal alcohol syndrome
 maternal exposure to: LSD, quinine, and phenytoin

Retinal detachments are MOST likely to be a feature of which phakomatosis:


von Recklinghausen disease
von Hippel-Lindau syndrome
Wyburn-Mason syndrome
Louis-Bar syndrome

Your answer was CORRECT


Explanation
von Hippel-Lindau presents with:
 retinal capillary haemangiomas (which can cause exudative RD)
 cerebellar haemangioblastomas
 pancreatic, hepatic, and renal cysts
 renal cell carcinoma
 pheochromocytomas

A 69-year-old woman presents with headache and diplopia. On examination, she has
limitation of elevation and adduction of the left eye with a left exotropia and partial ptosis.
The left pupil is dilated.
How would you manage?
ask the GP to monitor cardiovascular risk factors

prisms and orthoptic review

urgent ESR and CRP

urgent referral to neuro-surgery

Your answer was CORRECT


Explanation
The most appropriate answer of the choices provided is an urgent referral to neurosurgery.
Perhaps the ideal answer would be an urgent referral to neurology for imaging and appropriate
intervention as third nerve palsies due to aneurysms are often treated by coiling under
neurologists rather than by open surgery; and neurosurgeons usually do not accept patients
without a confirmed surgical diagnosis!

This question came in the FRCS (Glasgow) October 2014 exam.

When there is unilateral damage to the cervical sympathetic ganglia, ocular findings
include all of the following EXCEPT:

inferonasal

inferotemporal

superonasal

superotemporal

Your answer was CORRECT


Explanation
Figure: junctional scotoma caused by lesion affecting optic nerve and von Willebrand knee.

The knee of von Willebrand refers to the infero-nasal retinal fibers that cross in the optic
chiasm and course anteriorly into the contralateral optic nerve before running posteriorly. The
anatomy of von Willebrand's knee helps to explain the phenomenon of a junctional scotoma,
which is a contralateral supero-temporal visual field defect together with an ipsilateral optic
nerve defect, caused by a unilateral lesion of the anterior portion of the optic chiasm/optic
nerve junction.

A 32-year-old woman complains of a 4 day history of pain on eye movements and blurred
vision in her right eye. Systemic enquiry reveals a 6-week history of paraesthesias in the right
arm approximately 6 months before the onset of her visual disturbance. On examination,
acuities are right eye 6/24, left eye 6/6. There is reduced right colour vision and fields disclose
a central scotoma in the right eye and are normal for the left eye. There is no RAPD.

Which one of the following is TRUE?

the patient probably had a similar episode affect her left eye in the past

the patient probably has a hereditary optic neuropathy

the patient probably has factitious visual loss

oral steroid therapy is indicated

the patient probably has an acute maculopathy

Your answer was CORRECT


Explanation
This case suggests optic neuritis and with the history of paresthesias, the diagnosis of MS
should be entertained. An RAPD is almost invariably present in the acute phase. This patient
most likely had a subclinical contralateral episode of optic neuritis in the past and therefore no
detectable RAPD.

Note: In the Optic Neuritis Treatment Trial (ONTT), oral steroid therapy offered no
improvement in long-term prognosis and had a higher rate of subsequent optic neuritis
recurrence.

A 55-year-old right-handed man presents with a left homonymous hemianopia without sparing
of the macula.

Which of the following is most likely to accompany his presentation:


prosopognosia
alexia
speech impairment
object agnosia

Your answer was CORRECT


Explanation
This man has a lesion of his non-dominant parieto-occipital lobe.

Non-dominant parietal lobe lesions cause:


 hemispatial neglect
 difficulty with spatial orientation
 dressing apraxia
 object agnosia (or astereoagnosia: unable to name object in hand with eyes closed)

Dominant parietal lobe lesions cause:


 dyscalcula
 dysgraphia
 left-right disorientation
 finger agnosia
 alexia
 speech disturbance

Note the first four features above constitute Gerstmann syndrome

Other, retrochiasmal visual disorders:


 alexia with agraphia: dominant parietal lobe (angular gyrus)
 alexia without agraphia: dominant occipital lobe and corpus callosum
 hemi-alexia: corpus callosum
 prosopagnosia: bilateral inferior occipito-temporal junction
 formed hallucinations: temporal lobe
 unformed hallucinations: occipital lobe
 Balint syndrome (bilateral superior parieto-occipital lesions):
o inability to perceive the visual field as a whole (simultanagnosia)
o difficulty in fixating the eyes (oculomotor apraxia)
o inability to move the hand to a specific object by using vision (optic ataxia)

This question came in the FRCS (Glasgow) October 2014 exam.

All of the following are true of the optic canal EXCEPT:

it is formed by the two roots of the lesser wing of the sphenoid

it is narrowest posteriorly

it transmits the ophthalmic artery

it passes anteriorly, inferiorly and laterally from the middle cranial fossa to the orbit

Your answer was CORRECT


Explanation
The optic canal is narrowest anteriorly. Other options supplied are true.

A child with a squint is tested on a synoptophore. She is asked to "place the lion inside the
cage."

This is a test for:


stereopsis

subjective angle of deviation

central suppression

objective angle of deviation

angle kappa

Your answer was CORRECT


Explanation
When using the synoptophore, a patient is shown 2 images: from one eyepiece a lion, and
from the other eyepiece a cage. The patient is asked if they can see both the lion and the cage,
which is a test for simultaneous perception (the crudest form of BSV). The patient is then
asked to “put the lion inside the cage” by adjusting the angle of one eyepiece to overlap the
image of the lion with the image of the cage. This creates a subjective measure of the angle of
deviation, which can be compared with an objective measure taken by the examiner later
using either corneal reflections or a version of the alternate cover test.
When there is unilateral damage to the cervical sympathetic ganglia, ocular findings include
all of the following EXCEPT:

failure of the pupil to react to light

low intraocular pressure

updrawing of the lower eyelid

paralysis of Muller 's muscle

Your answer was CORRECT


Explanation
Horner 's syndrome causes miosis but the pupil can still react to light which is mediated by the
parasympathetic autonomic nervous system.

All of the following are consistent with a diagnosis of idiopathic intracranial hypertension
EXCEPT:
pleocytosis on CSF analysis
empty sella on MRI
normal ventricles on CT brain scan
opening pressure on lumbar puncture above 250 mm water

Your answer was CORRECT


Explanation
Idiopathic intracranial hypertension is characterised by:
 raised opening pressure over 250mm water
 normal brain imaging or imaging that cannot explain a cause for raised ICP
 normal CSF analysis
Ventricles may appear normal or small on brain imaging and an empty sella may also occur in
IIH.

A 14-year-old boy is very short for his age and obese. He frequently bumps into pedestrians
on the street. He is being investigated for delayed sexual maturation.

What is the most likely visual field defect?


binasal hemianopia
bitemporal hemianopia with denser inferotemporal defect
central bitemporal hemianopia
bitemporal hemianopia with denser superotemporal defect

Your answer was CORRECT


Explanation

Figure: T1 MRI with contrast of a papillary craniopharyngioma compressing chiasm from


above.

The history in the question above is suggestive of craniopharyngioma, which initially


compresses the chiasm superiorly and posteriorly damaging the upper nasal fibres. This results
in a bilateral infero-temporal field defect that progresses to a bitemporal hemianopia as the
lesion enlarges.

Craniopharygioma is characterised by:


 slow growing tumour
 arises from remnants of Rathke pouch
 children present with: dwarfism, delayed puberty, obesity
 adults present with visual impairment and field defects
 VF: inferotemporal fields as chiasm compressed from above
 MRI solid tumour iso-intense on T1; may have cystic components
 Treatment: surgery with or without radiation

The trigeminal nerve is responsible for all of the following EXCEPT:


tactile sensation to the external ear pinna
motor innervation of the muscles of mastication
proprioception to the muscles of facial expression
nociceptive sensation to the anterior scalp

Your answer was CORRECT


Explanation
The skin of the pinna is supplied by branches of the cervical nerves.

The trigeminal nerve is responsible for:


 tactile, nociceptive and thermal sensation to the face, and up to the vortex of the scalp
 motor innervation of the muscles of mastication
 proprioceptive sensation from the muscles of mastication, muscles of facial expression
and the extraocular muscles

A 42-year-old man has anisocoria with a larger left pupil. More anisocoria is present in the
dark than in the light. There is partial right ptosis but extra-ocular movements are full. All of
the following statements are true EXCEPT:

reduced tendon reflexes is a possible association

neither pupil should constrict to pilocarpine 0.1%

the presence of a right abduction deficit localises the lesion to the right cavernous sinus

the right pupil would dilate poorly after cocaine 10% instillation

Your answer was CORRECT


Explanation
The patient has right ptosis and miosis, or a right Horner's syndrome. The post-ganglionic
sympathetic chain and the sixth nerve travel together only in the cavernous sinus. Thus, a
cavernous sinus lesion would be most likely if an abduction deficit were present.
Parasympathetic denervation supersensitivity is not present in Horner's syndrome, so neither
pupil should constrict to dilute pilocarpine. A Horner's pupil does not dilate with cocaine 10%.

Reduced tendon reflexes are a feature of Holmes-Adie pupil, not Horner's syndrome.

While undergoing retinal examination, a patient claims to see his retinal vessels. This is an
example of:
formed hallucinations

Purkinje tree

the Pulfrich phenomenon

Charles Bonnet's syndrome

blindsight
Your answer was CORRECT
Explanation
The Purkinje tree is a physiological entoptic phenomenon. It can be seen by shining a beam of
a small bright penlight through the pupil from the periphery. This results in an image of the
light being focused on the periphery of the retina. Light from this spot then casts shadows of
the blood vessels onto unadapted portions of the retina. Normally the image of the retinal
blood vessels is invisible because of adaptation. Unless the light moves, the image disappears
within a second or so. If the light is moved at about 1 Hz, adaptation is defeated, and a clear
image of the retinal vascular tree can be seen indefinitely. The vascular figure that is perceived
is referred to as a Purkinje tree and is often seen by patients during an ophthalmic
examination.

A patient is diagnosed with normal tension glaucoma and prescribed latanoprost drops at night
to both eyes. On routine review 3-month later, it is noted that he has reduced visual acuity in
his right eye and visual field progression compared to the last visit bilaterally.

Which investigation is most important?


MRI

FFA

retinal nerve fibre layer OCT

electrodiagnostics

Your answer was CORRECT


Explanation
The differential of NTG must always include lesions of the visual pathway. A sudden
progression of visual fields and loss of visual acuity are not consistent with glaucoma, and an
alternative diagnosis should be sought urgently by arranging an MRI of the brain and orbits.

This question came in the FRCS (Glasgow) Part 2 in October 2014.

A man wakes up from a coma with bilateral complete ophthalmoplegia involving the pupils
and visual disturbance. His motor function in arms and legs is otherwise preserved.

In which vessel is the embolus?


posterior inferior cerebellar artery
basilar artery
internal carotid artery
posterior communicating artery

Your answer was CORRECT


Explanation
The ‘top of the basilar artery’ syndrome is caused by an embolus at the top of the basilar
artery, which causes ischaemia of the rostral brainstem, thalamus and portions of the cerebral
hemispheres fed by the posterior cerebral arteries.
Features include:
 altered consciousness
 behavioural abnormalities
 visual abnormalities
 oculomotor palsy involving the pupils
 No significant motor weakness in the extremities

This question appeared in the 2014 FRCOphth Part 2.

All of the following features are characteristic of an ipsilateral, posterior occipital lobe lesion
EXCEPT?

macular sparing

unformed hallucinations

asymmetric OKN drum responses

high congruity of field loss

Your answer was CORRECT


Explanation
Although OKN asymmetry may occur with occipital lesions, this finding tends to occur with
lateral occipital lobe involvment and especially parietal lobe involvment, which controls
ipsilateral pursuit (slow-phase of OKN).

Unformed visual hallucinations occur with occipital lobe lesions, while formed hallucinations
occur with temporal lobe pathology.

Occipital lobe lesions tend to produce highly congruous field defects. The only location in the
occipital lobe that bucks this rule is the anterior-most portion of the visual cortex, which sub-
serves the temporal crescent (from 60 degrees to 90 degrees temporally) in the contralateral
eye, producing a monocular temporal crescent field defect.

The dorsal column of the spine transmits:

autonomic motor pathways

volitional motor pathways

pain and temperature

vibration, touch and proprioception


Your answer was CORRECT
Explanation
The dorsal column of the spine transmits vibration, light touch and proprioception. Fibres
ascend the column, synapse in the gracile and cuneate nucleus of the medulla where the
decussate before ascending to the thalamus.

The most common cause of acquired fourth nerve palsy in adults is:

tumour

trauma

vasculopathy

aneurysm

Your answer was CORRECT


Explanation
The trochlear nerve has the longest intracranial course of all the cranial nerves, leaving it
especially susceptible to damage from closed head trauma. This occurs because of contrecoup
injury from the free tentorial edge. In fact, the fourth nerve is the most commonly damaged
cranial nerve from head trauma.

Which one of the following is NOT a cause of congenital sensory nystagmus?

Arnold-Chiari malformation

congenital cataracts

aniridia with foveal hypoplasia

rod monochromatism

Your answer was CORRECT


Explanation
Congenital sensory nystagmus can result from any condition where visual information does
not reach the occipital cortex properly during early visual development. Media opacities, such
as congenital cataracts, foveal or optic nerve hypoplasia, and retinal degenerations, may all
lead to sensory nystagmus.

An Arnold-Chiari malformation causes a downbeat motor nystagmus, not a sensory


deprivation nystagmus.
Which of the following statements regarding the actions of the extraocular muscle is FALSE?
in 39 degrees of abduction the inferior oblique acts almost exclusively as an extorter
the primary action of the superior rectus is elevation
the primary action of the superior oblique is depression
the secondary actions of the inferior rectus are adduction and extorsion

Your answer was CORRECT


Explanation
The primary action of the superior oblique is intorsion.

Some helpful rules to try and remember the muscle actions:


 the primary action of the obliques is intorsion/extorsion
 the primary action of the recti is in the direction of their insertion (e.g. medial adduct etc)
 vertical recti (sup and inf) always ADDuct as a secondary action
 obliques always ABDuct as a secondary action
 superior muscles INtort
 inferior muscles EXtort
 39 degrees abduction isolates primary action of the obliques (intort/extort)
 51 degrees adduction isolates secondary action of the obliques (elevate/depress)
 23 degrees abduction isolates primary action of sup/inf recti (elevate/depress)
 67 degrees adduction isolates secondary action of sup/inf recti (intort/extort)

A variant of Guillain-Barre syndrome that involves mainly the brainstem and cranial nerves is
known as:

Lambert-Eaton syndrome

WEBINO

Millard-Gubler syndrome

Miller-Fisher syndrome

Foville's syndrome

Your answer was CORRECT


Explanation
Miller Fisher syndrome, which is a variant of Guillan-Barre syndrome, is characterised by:

 ophthalmoplegia
 ataxia
 areflexia
 descending paralysis (as opposed to GBS where it is ascending)
 positive anti-GQ1b antibodies in 90%
 treatment by plasmapheresis and iv immunoglobulin
A 35-year-old fit and well Caucasian lady is diagnosed with a first episode of optic neuritis.
She undergoes an MRI brain scan, which is entirely normal. She would like to know her risk
of developing multiple sclerosis in the next 10 years.

What should you advise her?


80%
35%
50%
20%

Your answer was CORRECT


Explanation
In a patient with optic neuritis, the overall 10-year risk of developing MS is 38%. If there are
no MRI lesions, the 10-year risk is 22%, while if there are lesions on MRI the 10-year risk of
MS is 56%.

A 'pie-in-the-sky' visual field defect is caused by a lesion in the:

temporal lobe

optic tract

parietal lobe

optic chiasm

Your answer was CORRECT


Explanation
A 'pie-in-the-sky' defect is an incongruous, homonymous, superior quadrantanopic defect. It is
the result of a lesion affecting the anterior temporal lobe (Meyer's loop) contralateral to the
side of the visual field defect.

Glaucoma is MOST likely to be encountered in which phakomatosis:

von Hippel-Lindau syndrome

Sturge-Weber syndrome

Bourneville's syndrome

Wyburn-Mason syndrome

Your answer was CORRECT


Explanation
Sturge-Weber is characterised by:

 sporadic condition (not inherited)


 port wine stain
 ipsilateral intracranial haemangioma
 seizures
 mental retardation
 diffuse choroidal haemangioma
 conjunctival or epislceral haemangioma
 glaucoma from elevated episcleral venous pressure

A right homonymous hemianopia with asymmetric OKN drum responses suggests a lesion in
the:
left parietal lobe
right occiptal lobe
right parietal lobe
left occipital lobe

Your answer was CORRECT


Explanation
Parietal lobe lesions classically produce an inferior quadrantanopia, but they can also produce
an incongruous homonymous hemianopia. The parietal lobe is responsible for ipsilateral
pursuit, which means lesions of the parietal lobe cause aymmetric OKN drum responses with
slowing of the response when the drum is rotated towards the side of the lesion (away from
the visual field defect).

A right homonymous hemianopia with asymmetric OKN drum responses suggests a lesion in
the:
left parietal lobe
right occiptal lobe
right parietal lobe
left occipital lobe

Your answer was CORRECT


Explanation
Parietal lobe lesions classically produce an inferior quadrantanopia, but they can also produce
an incongruous homonymous hemianopia. The parietal lobe is responsible for ipsilateral
pursuit, which means lesions of the parietal lobe cause aymmetric OKN drum responses with
slowing of the response when the drum is rotated towards the side of the lesion (away from
the visual field defect).
A 62-year-old Korean man presents with distant, binocular, horizontal diplopia. There is a
right face turn. On examination, the right eye is proptosed. Systemic enquiry reveals he has
been experiencing frequent epistaxis. There is no hearing loss and corneal sensation is normal.

What is the most likely diagnosis?


high-flow carotid-cavernous fistula
invasive pituitary adenoma
nasopharyngeal tumour
Tolosa-Hunt syndrome
acoustic neuroma

Your answer was CORRECT


Explanation
Figure: Right nasopharyngeal carcinoma T1 with contrast

In the question above, the patient has a right isolated sixth nerve palsy, with proptosis
suggesting a mass and nasal symptoms (epistaxis). The findings are most in keeping with a
nasopharyngeal carcinoma, which is more common in patients from the Far East.

Other options above would be expected to produce multiple cranial nerve palsies (e.g. 5th, 7th
and 8th nerve with acoustic neuroma or 2nd, 3rd, 4th and V1 with carotid cavernous fistula,
Tolosa-Hunt or invasive pituitary adenoma).

A patient has a right third nerve palsy. Full neurological examination reveals left-sided hemi-
ataxia with left intention tremor, left partial hemiparesis and brisk left deep tendon reflexes.
These symptoms suggest a lesion in the:

midbrain

pons

primary motor cortex

medulla

cerebellum

Your answer was CORRECT


Explanation
The features described above suggest Benedikt's syndrome, which is caused by a midbrain
lesion affecting the third nerve fasciculus and the red nucleus.

Features of Benedikt's syndrome: 1

 ipsilateral CN III palsy


 contralateral cerebellar ataxia with intention tremor
 contralateral hemiparesis with hyperactive tendon reflexes
Claude syndrome is characterised by: 2

 ipsilateral 3rd nerve palsy


 contralateral cerebellar ataxia
Weber syndrome is characterised by:
 ipsilateral 3rd nerve
 contralateral hemi-paresis
Nothnagel syndrome is characterised by:
 ipsilateral 3rd nerve
 ipsilateral cerebellar ataxia

1. Reference for Benedikt syndrome


2. Reference for Claude syndrome
3. General reference for 3rd nerve syndromes
A paralytic extraocular muscle disorder can be BEST distinguished from a restrictive disorder
by:
double Maddox rod test
forced duction test
4 prism diopter test
3-step test

Your answer was CORRECT


Explanation
The forced duction test is positive in a restrictive disorder but negative in a paretic disorder.

Ischaemia due to occlusion of a calcarine artery would be expected to produce:


congruous contralateral homonymous hemianopia
incongruous contralateral homonymous hemianopia
congruous contralateral homonymous hemianopia with macular sparing
incongruous contralateral homonymous hemianopia with macular sparing

Your answer was CORRECT


Explanation
Ischaemia to a calcarine artery produces a congruous defect (more posterior the defect, more
congruous) and there is often macular sparing because of anastomoses between the middle and
posterior cerebral arteries at the very pole, where the macular fibres terminate.

A 36-year-old lady who was in a car accident several days ago complains of vertical diplopia
since the accident. On examination, she has a right hypertropia worse on left gaze and
improved by left tilt.

What would you use to measure the amount of torsion that this patient has?
neutral density filters
double Maddox rods
red filter and a light
alternate cover test with prisms

Your answer was CORRECT


Explanation
The Maddox rod takes a point source of light and converts it into a straight line. With a
Maddox rod in front of each eye (in a trial frame), the patient can rotate one of the rods until
the lines he sees in each eye are parallel. The difference in the axes of the two Maddox rods is
the degree of torsion.
A patient with a clear cornea, pigmentary retinopathy, optic atrophy and progressive dementia
is MOST likely to have:
Hurler-Scheie syndrome
Sanfilippo syndrome
Maroteaux-Lamy syndrome
Morquio syndrome

Your answer was CORRECT


Explanation
Sanfilippo syndrome is part of the mucopolysaccharidoses group of enzyme deficiency
disorders. Patients may have a clear cornea, with pigmentary retinopathy, optic atrophy, mild
dysmorphism and progressive dementia.

A 35-year-old woman was involved in a road traffic accident and since then has noticed a
difference in pupil size. On examination, her right pupil is larger than the left. When 1%
apraclonidine is instilled, the left pupil dilates but the right pupil does not.

What is the diagnosis?


right Horner's
left Horner's
right Adies
left Adies

Your answer was CORRECT


Explanation
Apraclonidine (Iopidine) is an alpha-adrenergic receptor agonist normally used for the short-
term reduction of intra-ocular pressure (IOP). Its IOP-lowering effect results from reduced
production of aqueous humour via its action on post-junctional alpha-2-receptors in the ciliary
body. The alpha-1-activity of apraclonidine does not affect aqueous production, but results in
conjunctival vasoconstriction that is often noted with its use. In Horner's syndrome,
sympathetic denervation results in upregulation of alpha-receptors which, in turn, unmasks the
weak effect of apraclonidine on alpha-1-receptors in the pupil dilator muscle by causing
mydriasis. In a normal pupil, there is no response to apraclonidine 1%. 

Note: the traditional test for Horner's involves cocaine drops, which cause dilation of a normal
pupil but NOT a Horner's pupil.  However, the iopidine test can be just as sensitive, and the
drops are much easier to locate in clinic. The figure above shows an approach to anisocoria
testing, using drops that are readily available in clinic: apraclonidine, phenylephrine and
pilocarpine. 

This question came in the FRCS (Glasgow) Part 2 exam in October 2014.

All of the following are characteristic findings in chiasmal compression EXCEPT:

color desaturation nasally


nystagmus

blindness post-fixation

intractable diplopia

Your answer was CORRECT


Explanation
Reds and greens often appear washed out in the temporal hemifield of affected patients.
Postfixation blindness is a necessary concomitant of bitemporal hemianopia. Objects behind
the point of fixation are in the temporal hemifield of each eye. With loss of these fields,
nothing beyond the point of fixation is visible. Hemifield slip refers to the diplopia these
patients may also notice by mechanisms that are not entirely understood; binocular input at the
vertical midline seems necessary for motor fusion.

Features of Chiasmal Syndrome


 Visual field defects (depending on location)
o Bitemporal hemianopia, involving superior fields first (classic)
o Incongruous homonymous hemianopia (optic tract involved)
o Bitemporal central scotoma (macular fibres)
o Unilateral centro-coecal scotoma (optic nerve)
o Junctional scotoma (junction of nerve and chiasm)
 Optic atrophy (variable)
o Normal looking disc
o Temporal pallor (papillomacular bundle)
o Bow tie atrophy
o Dense optic atrophy
 Hemifield slip (non-paretic diplopia)
 Post-fixation blindness
 Visual hallucination
 See-saw nystagmus

Which statement about diabetic papillopathy is TRUE?


the disorder seems to be independent of the extent of blood sugar control
it is typically followed by the development of disc neovascularization
visual loss is usually severe
it is typically painful

Your answer was CORRECT


Explanation
The development of diabetic papillopathy appears to be independent of serum glucose levels.
Diabetic papillopathy is classically seen in young adults with type 1 diabetes with moderate to
severe retinopathy. It is painless, and associated visual loss is generally mild. The disorder
generally resolves spontaneously.
Which of the following is most likely to occur in early retrobulbar neuritis?
an inferior altitudinal defect
a pale disc
pain on eye movement
a swollen disc

Your answer was CORRECT


Explanation
In early retrobulbar neuritis, the optic disc appears normal and there is pain on eye movement.
The visual field defect is typically centro-caecal.

A 19-year-old presents with decreased visual acuity to 6/18 bilaterally. He states that the
vision in his left eye started to decline gradually over the past 3 months. His right eye has just
recently become affected. Examination shows evidence of bilateral optic neuropathy and you
suspect Leber's hereditary optic neuropathy.

Which is the best test to confirm your suspicion?


CT brain
MRI brain
DNA analysis
Serum electrophoresis
Lumbar puncture

Your answer was CORRECT


Explanation
Leber's hereditary optic neuropathy (LHON) is caused by an abnormality in mitochondrial
DNA. Since mitochondrial DNA is only transmitted along the maternal line, men cannot pass
this disease to their offspring.

In the acute phase, the optic nerve in LHON is hyperaemic and swollen with telangiectatic
capillaries. The nerve does not leak on fluorescein angiography. Later stages may only
manifest optic atrophy.

Findings in a patient with von Hippel-Lindau disease may include all of the following except:
pancreatic and renal cysts
renal cell carcinoma
pheochromocytoma
cafe-au-lait spots
haemangioblastomas of the brainstem

Your answer was CORRECT


Explanation
Cafe-au-lait spots are characteristic of neurofibromatosis and are not seen with von Hippel-
Lindau disease.
The fifth cranial nerve:
is involved in the blink reflex
supplies parasympathetic fibres to the salivary gland
arises from the midbrain
innervates the muscle of facial expression
provides sensory innervations to the whole face

Your answer was CORRECT


Explanation
The fifth cranial nerve provides sensation to the face except for a region around the angle of
the mandible which is supplied by C2 and C3 in the form of the great auricular nerve. The
mandibular branch of the fifth nerve supplies the muscles of mastication, but the seventh
nerve supplies muscles of facial expression. Corneal sensation is provided by the ophthalmic
branch of the fifth nerve and is involved in the blink reflex. The fifth cranial nerve does not
contain parasympathetic nerves fibres of its own.

Which of the following drops causes a post-ganglionic Horner's pupil to dilate?


adrenaline 1:1000
pilocarpine 0.1%
hydroxyamphetamine 1%
cocaine 4%

Your answer was CORRECT


Explanation
Due to denervation hypersensitivity, a post-ganglionic Horner's pupil will dilate with dilute
adrenaline.

The pharmacological tests in Horner's syndrome include the following:

Identify Horner's syndrome


Cocaine 4% blocks reuptake of catecholamines at the synapse (causing increased sympathetic
activity and therefore mydriasis in a normal pupil) but in Horner's syndrome, there is no native
catecholamine present at the synapse, therefore no dilation.

Apraclonidine 0.5% is an alternative to topical cocaine (which can be difficult to source) to


confirm Horner's syndrome. Apraclonidine is an alpha adrenergic agonist. It causes pupillary
dilation in the Horner's pupil due to denervation supersensitivity while producing a mild
pupillary constriction in the normal pupil presumably by down-regulating the norepinephrine
release at the synaptic cleft. A reversal of anisocoria after instilling two drops of 0.5%
apraclonidine is suggestive of Horner's syndrome.
Distinguish Second- from Third- Order Horner's
Hydroxyamphetamine 1% stimulates release of catecholamines from the presynaptic neuron at
the effector junction. If hydroxyamphetamine fails to dilate a Horner's pupil the lesion must be
post-ganglionic (third-order).

Dilute adrenaline 1:1000 (or simply phenylephrine 1%) can also be useful in distinguishing
pre-ganglionic from post-ganglionic Horner's syndrome. Both adrenaline and phenylephrine
are direct agonists at the post-synaptic cleft. In a third-order Horner's there is denervation
hypersensitivity, which will cause dilation, while there is no response (or comparatively less
response) with a normal pupil, or a pupil that has a pre-ganglionic Horner's.

A patient with congenital nystagmus has a null zone in right gaze and has adopted an extreme
left head turn.

What might the treatment for this patient include:


prism base in, in the right eye and base out, in the left eye
prism base up in both eyes
prism base in, in both eyes
prism base out, in the right eye and base in, in the left eye
prism base down in both eyes

Your answer was CORRECT


Explanation
This combination of prisms will force the child to gaze to the right in order to see objects
straight ahead. This will obviate a head turn, which is typically the end point of treatment.

Supranuclear vertical gaze abnormalities can be a feature of all the following EXCEPT:
Parinaud's syndromoe
Parkinson's disease
ataxia-telangiectasia
pineal region tumors
myasthenia gravis

Your answer was CORRECT


Explanation
Myasthenia gravis produces ophthalmoplegia due to neuromuscular junction pathology.
Parkinson's disease, pineal region tumors (dorsal midbrain syndrome), and ataxia-
telangiectasia all can cause supranuclear gaze palsy.

To elicit the Pulfrich phenomenon, the examiner should:


ask the patient to glance quickly back and forth horizontally and report any photopsias
ask the patient to climb several flights of stairs briskly and report any visual loss
spin the examining chair and watch for any nystagmus while the patient fixates on her
outstretched thumb
ask the patient to watch the pendulum on the grandfather clock across the room and
report any three-dimensional movement

Your answer was CORRECT


Explanation
Pulfrich phenomenon reflects delayed conduction in the demyelinated nerve and occurs where
oscillating objects viewed binocularly and moving in one plane (e.g. pendulum) appear to
have three-dimensional movement.

Uhthoff's symptom occurs with optic neuritis and is a decrease in vision with an increase in
body temperature e.g. with exercise or hot showers.

Lhermitte's sign is the electric shock sensation with neck flexion and is found in patients with
multiple sclerosis.

A Goldmann visual field shows a bitemporal hemianopic central scotoma. What is the most
likely site of the lesion:
mid-chiasm
anterior chiasm
posterior chiasm
thalamus
optic nerve

Your answer was CORRECT


Explanation
A bitemporal hemianopic central scotoma is produced by a lesion compressing the macular
fibers of the posterior chiasm.

A 45-year-old woman comes to the emergency room with complaints of double vision and
headache for the last 2 days. On examination, the left eye is turned downward and outward.
The pupil is larger compared to the right.

What is the site and type of lesion:


laceration or contusion in the parotid region
stretching of the nerve as it courses around the brainstem
fracture of the cribriform plate
pressure from herniating uncus on the nerve or fracture in the cavernous sinus
acoustic neuroma

Your answer was INCORRECT


Explanation
Pressure from the uncus, a fracture in the cavernous sinus, or aneurysms (especially of the
posterior communicating artery) can damage the occulomotor nerve (III), resulting in a dilated
pupil, ptosis, the eye being turned downward and outward, and a loss of the pupillary light
reflex on the side of the lesion.
Optic disc drusen are LEAST likely to be visualised on:
CT
ophthalmic ultrasonography
fundus photos using standard fluorescein angiography filters
MRI

Your answer was CORRECT


Explanation
Since MRI does not image calcium, this modality is not useful in imaging optic disc drusen.
By contrast, optic nerve drusen are readily visible on CT and on ocular ultrsound as hyper-
reflective masses. Drusen also autofluoresce, which can be detected on fundus photography
through standard fluorescein filters, provided the drusen is near the disc surface.

A brainstem lesion that involves the medial longitudinal fasciculus as well as the ipsilateral
abducens nucleus will most likely cause:
INO with skew
WEBINO
Fisher's syndrome
one-and-a-half syndrome
Foville's syndrome

Your answer was CORRECT


Explanation
A lesion of the abducens nucleus will often involve the nearby PPRF and result in an
ipsilateral gaze paresis. A lesion that also disrupts the ipsilateral MLF will result in the
combination of an ipsilateral gaze palsy and internuclear ophthalmoplegia. This combination
has been termed the one-and-a-half syndrome. The only horizontal eye movement that
remains is contralateral abduction.
A 34-year-old man who was in a car accident several days ago complains of vertical diplopia
since the accident. His monocular acuity is 6/4 in each eye. His motility is diagramed above.

What is the MOST likely diagnosis?


right inferior oblique
left superior rectus
left inferior rectus
right superior oblique

Your answer was CORRECT


Explanation
The Park's three-step test can be used to determine which muscle is palsied. The patient has a
right hypertropia in primary position, which is worse on left gaze. The hypertropia also
worsens on right head tilt. This pattern indicates a right superior oblique palsy

A patient has complete inability to recognise faces. A lesion in which area is most likely to
explain his symptoms:

unilateral occipital lobe


bilateral inferior occipitotemporal junction

dominant angular gyrus

non-dominant parietal lobe

occipital lobe and corpus callosum

Your answer was CORRECT


Explanation
Retrochiasmal visual disorders:
 alexia with agraphia: dominant parietal lobe (angular gyrus)
 alexia without agraphia: dominant occipital lobe and corpus callosum
 hemi-alexia: corpus callosum
 prosopagnosia: bilateral inferior occipito-temporal junction
 formed hallucinations: temporal lobe
 unformed hallucinations: occipital lobe

Which of the following structures contains fifth cranial nerve fibers:


otic ganglion
trigeminal ganglion
ciliary ganglion
geniculate ganglion

Your answer was CORRECT


Explanation
The otic ganglion is a parasympathetic ganglion located immediately below the foramen
ovale. It communicates with the glossopharyngeal and facial nerves.
The geniculate ganglion contains the facial nerve.

The trigeminal ganglion contains the fifth cranial nerve.

The ciliary ganglion contains the third cranial nerve.

The pterygopalatine ganglion (or sphenopalatine ganglion) is a parasympathetic


ganglion found in the pterygopalatine fossa. It receives a sensory, a motor, and a
sympathetic root. Its sensory root is derived from two sphenopalatine branches of the
maxillary nerve. Its motor root is derived from the nervus intermedius through the
greater superficial petrosal nerve. Its sympathetic root is derived from the carotid
plexus through the deep petrosal nerve.

The LEAST frequently involved rectus muscle in Graves' ophthalmopathy is the:


inferior rectus

medial rectus

lateral rectus

superior rectus

Your answer was CORRECT


Explanation
In descending order from most to least frequently involved in TED:

 inferior rectus
 medial rectus
 superior rectus
 lateral rectus

NOTE: remember the pnemonic I M Stuart Little

Which one of the following fits the classic phenotype for a patient with pseudotumour cerebri?
10-year-old girl with ANA positive pauci-articular arthritis
75-year-old woman with history of TIAs
58-year-old man with a type A personality
35-year-old overweight woman

Your answer was CORRECT


Explanation
Patients with pseudotumour cerebri have a distinctive profile. They are typically obese women
between the ages of 20 and 40 years. The exact aetiology of this condition is unknown but a
hormonal imbalance has been postulated because it may be exacerbated by pregnancy.

Besides the optic nerve, in which area is it possible to have a lesion that causes a monocular
visual field defect:
primary visual cortex
optic radiation
lateral geniculate body
optic tract

Your answer was CORRECT


Explanation
In each eye, the temporal field (detected by the nasal retina) is wider than the corresponding
nasal field. As a result, there is a crescent in the binocular visual field temporally that is only
seen by nasal retina of the ipsilateral eye and not by the contralateral eye. This area is located
most anteriorly in the primary visual cortex, and it may be the only portion of the visual field
spared after occipital lobe damage. Conversely, a lesion solely affecting this area of the visual
cortex can cause a monocular temporal field defect.

All of the following are advantages of CT scans over MRI scans except:
quicker
visualisation of abnormal flow in blood vessels
less patient cooperation is needed
excellent visualisation of bony abnormalities

Your answer was CORRECT


Explanation
MRI is better than CT at detecting abnormal flow in blood vessels (e.g. MR angiography).
MRI is also better at distinguishing orbital soft tissue, making it invaluable in assessing
thyroid eye disease and tumours of the orbital cone such as meningioma. CT scans are best for
demonstrating bony structures, including erosion; thus CT is the investigation of choice for
preseptal cellulitis and dermoid cysts.

A patient complains of decreased right vision. His acuities are 6/36 right, 6/6 left. The
examination is entirely unremarkable, including normal pupillary reactions and the diagnosis
of factitious visual loss is considered.

Which test would be particularly useful in establishing this diagnosis:


optokinetic nystagmus (OKN) drum
performing a fogging refraction
gently rocking a large mirror in front of the patient, with the good eye occluded
introducing a prism base-up in front of the left eye when the patient is reading binocularly

Your answer was CORRECT


Explanation
The OKN drum, the rocking mirror test, and the base-up prism test are useful to discover
factitious monocular blindness that is severe at the HM, PL and NPL level. These tests are not
sensitive enough to diagnose factitious visual deficit at the 6/36 level. For such relatively mild
visual deficits, a fogging refraction, stereoacuity, and red-green glasses may be useful in
diagnosing factitious visual loss.

An anterior communicating artery aneurysm is most likely to produce:


a homonymous lower quadrantanopia
a bitemporal upper quadrantanopia
a bitemporal lower quadrantanopia
a homonymous upper quadrantanopia

Your answer was CORRECT


Explanation
An anterior communicating artery aneurysm is most likely to compress the upper portion of
the optic chiasm, producing a bitemporal lower quandrantanopia.

A patient presents with an abnormality of eye movements. On laevoversion, you note that her
left eye abducts while exhibiting nystagmus but the right eye does not adduct. On attempted
dextroversion, neither eye moves.

What is the diagnosis?


right MLF lesion
left MLF lesion
left one-and-a-half syndrome
right one-and-a-half syndrome

Your answer was CORRECT


Explanation
This patient has one-and-a-half syndrome, which is a unilateral lesion involving the MLF and
PPRF. The only possible horizontal eye movement is that of contralateral abduction, albeit
with abducting nystagmus.

A 71-year-old man develops sudden onset diplopia. Examination reveals right-sided intention
tremor and dysdiadochokinesis. The right pupil is dilated and the right eye has limited
adduction, elevation and depression.

What is the most likely diagnosis?

Foville syndrome

Weber syndrome

Claude syndrome

Nothnagel syndrome

Benedikt syndrome

Your answer was CORRECT


Explanation
The symptoms are consistent with Nothnagel syndrome, which involves the third nerve
fasciculus and the superior cerebellar peduncle, which causes third nerve palsy and ipsilateral
cerebellar ataxia.

Features of Benedikt's syndrome: 1

 ipsilateral CN III palsy


 contralateral cerebellar ataxia with intention tremor (major feature)
 contralateral hemiparesis with hyperactive tendon reflexes (lesser feature)
Claude syndrome is characterised by: 2

 ipsilateral 3rd nerve palsy


 contralateral cerebellar ataxia
Weber syndrome is characterised by:
 ipsilateral 3rd nerve
 contralateral hemi-paresis
Nothnagel syndrome is characterised by:
 ipsilateral 3rd nerve
 ipsilateral cerebellar ataxia

1. Reference for Benedikt syndrome


2. Reference for Claude syndrome
3. General reference for 3rd nerve syndromes

Which of the following statements about optic neuritis in childhood is TRUE?

the most common presentation is retrobulbar neuritis

optic neuritis is more commonly bilateral than unilateral

the aetiology is typically autoimmune demyelination

the visual prognosis is poor

enlargement of optic nerves on neuroimaging suggests an alternative diagnosis

Your answer was CORRECT


Explanation
Optic neuritis in childhood is:

 usually bilateral (versus unilateral in adults)


 aetiology post-infectious (versus autoimmune in adults)
 main site papillitis (versus retrobulbar in adults)
 visual loss can be severe, but recovery is generally good
 diffuse enlargement of the optic nerve on imaging may be seen in this condition,
mimicking a neoplasm of the optic nerve sheath

A patient has a right Duane's syndrome type 1 with a 25 prism dioptre esotropia at distance.

What is the most appropriate procedure?


right medial rectus recession
right medial rectus recession and lateral rectus resection
bilateral medial recti recessions
bilateral lateral recti resections
Your answer was CORRECT
Explanation
This is a small degree of esotropia and the recommended procedure is unilateral medial rectus
recession. For larger deviations, bilateral medical recti recessions are warranted and even
vertical recti transpositions can be considered. Lateral recti muscles should not be resected in
Duane's because this will usually increase the globe retraction.

A left superior homonymous quadrantanopia is most likely to be caused by a lesion in the:


right occipital lobe
right temporal lobe
left optic nerve
right parietal lobe
right optic tract

Your answer was CORRECT


Explanation
A homonymous visual field defect is caused by a contralateral lesion posterior to the chiasm.
A superior quadrantanopia is caused by injury to the inferior fibers of the optic radiation that
must detour through the temporal lobe to avoid the ventricles (Meyer's loop). An inferior
quadrantanopia is caused by lesions affecting more superior fibres in the parietal lobe.

A posterior communicating artery aneurysm is most likely to cause:


a sixth nerve pasy
a fourth nerve palsy
a third nerve palsy
a seventh nerve palsy
a fifth nerve pasy

Your answer was CORRECT


Explanation
A posterior communicating artery aneurysm is most likely to cause a third nerve palsy as the
third nerve on its exit from the brainstem runs alongside and lateral to the posterior
communicating artery.

All of the following clinical features are in keeping with a complete nuclear third nerve palsy
EXCEPT:

ipsilateral ptosis

bilateral mydriasis

contralateral superior rectus palsy


ipsilateral inferior oblique palsy

Your answer was CORRECT


Explanation
Ptosis is bilateral in a nuclear third nerve palsy, because the levator palpebrae superioris is
supplied by a single, unpaired nucleus.

Third nerve nuclei:

 Unpaired: levator nucleus and Edinger Westphal (bilateral ptosis and mydriasis)
 Paired supplying contralateral: superior rectus (contralateral palsy)
 Paired supplying ipsilateral: MR, IR, IO (ipsilateral palsy)
Note: the superior oblique (fourth) nucleus is paired and supplies the contralateral muscle.

A man of no fixed abode is admitted to hospital with confusion and peripheral neuropathy. 2
days later he complains of painless bilateral reduced vision, though fundoscopy is normal.

What is the most likely diagnosis?

demyelinating disease

nutritional optic neuropathy

non-organic visual loss

Wolfram syndrome

Friedrich ataxia

Your answer was CORRECT


Explanation
This patient has vitamin B deficiency (Wernicke's encephalopathy).

Vitamin B12 deficiency is characterised by:

 peripheral neuropathy in a glove-and-stocking distribution


 confusion
 cognitive decline
 GI symptoms
 glossitis
 ocular features:
o early: the optic discs appear normal or may be hyperaemic
o later: if untreated, permanent visual loss ensues and temporal pallor develops

Which of the following muscles is supplied by the contralateral oculomotor nucleus:


inferior rectus

medial rectus

superior rectus

inferior oblique

Your answer was CORRECT


Explanation
The axons for most of the extraocular muscles are uncrossed from the nucleus to the eye (in
other words, ipsilateral innervation).

However there are a few exceptions:

 the levator palpebrae on either side derive their innervation from a single caudal, dorsal
midline nucleus
 the superior rectus derives innervation from the contralateral superior rectus sub-nucleus
 the superior oblique derives innervation from the contra-lateral trochlear nucleus, since
the trochlear nerve axons cross the midline before emerging from the brainstem posteriorly

Which statement regarding Adie's tonic pupil is FALSE?


usually unilateral involvement
female preponderance
pupillary constriction in response to 0.25% pilocarpine is conclusive evidence of
denervation hypersensitivity
most patients will manifest Adie's syndrome
the differential diagnosis of a tonic pupil includes herpes zoster, syphilis, GCA and diabetes
mellitus

Your answer was CORRECT


Explanation
Although 0.25% pilocarpine is very dilute, patients with normal pupils may respond to this
dose. Therefore, a weaker preparation (0.10%) is recommended. Most patients affected will
have unilateral pupillary involvement and depressed tendon reflexes. Females are more
commonly affected. After many months or years, an Adie's pupil will become miotic.

All of the following features are characteristic of latent nystagmus EXCEPT:

fast phase towards fixing eye

null point always in adduction

is associated with congenital monocular cataracts

bilateral
face turn towards the non-fixing eye

Your answer was CORRECT


Explanation
Face-turn is towards the fixing eye, to keep the fixing eye in its null point in adduction. The
null point in manifest-latent nystagmus is always in adduction, in contrast to congenital
nystagmus, where it can be in any position of gaze.

Characteristics of manifest-latent nystagmus:

 bilateral
 becomes manifest when one eye occluded, blurred or intermittently suppressed
 jerk-type nystagmus
 null point in adduction
 fast phase towards fixing eye
 face turn towards the fixing eye as this dampens nystagmus
 associated with interruptions to binocular development: congenital esotropia, but also
monocular congenital cataracts
Characteristics of congenital nystagmus:
 jerk or pendular nystagmus
 normal or near-normal visual acuity
 no change in nystagmus with unilateral occlusion or blurring
 fast phase switches: to right in right-gaze, to left in left-gaze
 null point can occur in any position of gaze
 compensatory head postures vary by patient: face-turns either way or chin up, chin down
depending on the position of the null point
 titubation
 tends to dampen with convergence, darkness, sleep, when eye is covered
 increases with fixation
 paradoxical OKN response

A child with spasmus nutans should be:

investigated with brain imaging

treated with bifocals

treated with cycloplegic penalisation

reassured that it usually resolves by 6 years of age

Your answer was CORRECT


Explanation
Spasmus nutans usually resolves by 6 years of age, but it is difficult to differentiate clinically
from monocular nystagmus of childhood (which can be caused by a chiasmal glioma). Hence,
any patient with suspected spasmus nutans should undergo neuroimaging.

The classic triad of spasmus nutans includes:

 monocular or dissociated small-amplitude nystagmus


 head bobbing
 torticollis
The differential diagnosis for this disorder includes:
 chiasmatic gliomas
 subacute necrotising encephalomyopathy
The above two disorders may produce optic atrophy, irritability, vomiting, and increased
intracranial pressure. Spasmus nutans should be considered only after these two disorders are
excluded.

Retinopathy of prematurity-like proliferative retinopathy is a feature of which neurocutaneous


syndrome:

Sturge-Weber syndrome

Wyburn Mason syndrome

incontinentia pigmentii

Louis-Bar syndrome

von Hippel Lindau syndrome

Your answer was CORRECT


Explanation
Incontinential pigmentii is characterised by:

 X-linked dominant inheritance


 lethal in utero for boys
 erythema and bullae on the skin of the extremities
 wart like skin changes
 hyperpigmented macules in Christmas-tree pattern on trunk
 epilepsy
 mental retardation
 malformation of teeth, hair, nails, bone
 hydrocephalus
 vitreoretinal dysplasia with proliferative retinopathy, retrolental masses, leukocoria

A 53-year-old black woman noticed a change in left vision yesterday. On examination, visual
acuity is 6/6 right, 6/60 left. There is a left RAPD and a swollen left optic disc. The remainder
of the examination is normal.

Which of the following facts from her past medical history is LEAST likely to be pertinent to
this presentation?
hyperthyroidism treated 1 year ago with radio-iodine
recent 1-month episode of left arm numbness
hilar adenopathy on recent chest radiograph
diabetes treated for 5 years with metformin

Your answer was CORRECT


Explanation
This lady has left optic neuropathy with a swollen nerve. Differential is wide but could include
ischaemic optic neuropathy (diabetes significant), compressive optic neuropathy (from
sarcoid), inflammatory optic neuropathy (demyelination or sarcoid, with other neurological
signs such as numbness).

Importantly, there are no orbital signs such as proptosis, chemosis, strabismus, or specific
signs of thyroid orbitopathy such as lid retraction, so TED is very unlikely here and the history
of treated hyperthyroidism unlikely to be significant.

A 19-year-old patient presents to her GP with bilateral progressive visual loss. She has pale-
brown macules on her torso and rubbery papules on her back and arms. One of her brothers,
her mother and her uncle have similar lesions.

What is the most likely ocular diagnosis?

optic nerve meningioma

diffuse choroidal haemangioma

optic nerve glioma

Tolosa-Hunt syndrome

Your answer was CORRECT


Explanation
Figure: Cafe-au-lait patches (black arrows) and neurofibroma (white arrow) in patient with
NF-1.

The clinical features in the question above suggest neurofibromatosis type 1 (von
Recklinghausen's disease or NF-1) with neurofibromas and café-au-lait spots. NF-1 is
associated with optic nerve gliomas.

NF-1 is characterised by:

 AD inheritence
 café-au-lait patches
 axial freckles (pathognomonic)
 short stature
 macrocephaly
 facial hemi-atrophy
 Lisch nodules
 ON gliomas
 meningiomas
 plexiform neurofibromas
 choroidal hamartomas (present in up to 100% of patients, many detectable only on SLO)
 retinal tumours
o astrocytic hamartomas
o combined hamartomas of RPE and retina
 absence of greater wing of sphenoid
 prominent corneal nerves
A 70-year-old hypertensive male presents with horizontal diplopia on right gaze. He also
complains of altered sensation and weakness of the right side of the face and altered sensation
and weakness of the left side of the body.

What is the diagnosis?


intra-cavernous lesion of the right 6th nerve
Foville's syndrome
Raymond's syndrome
Millard-Gubler syndrome

Your answer was CORRECT


Explanation
Foville’s syndrome is caused by blockage of the perforating branches of the basilar artery in
the pons. Structures affected by the infarct are the PPRF, nuclei of cranial nerves V, VI and
VII, corticospinal tract, medial lemniscus, and the MLF. This produces ipsilateral horizontal
gaze palsy and facial nerve palsy and contralateral hemiparesis, hemisensory loss, and
internuclear ophthalmoplegia.

Facial myokymia in a child is frequently associated with:


cerebellar hemangioblastoma
spasmus nutans
nasopharyngeal carcinoma
pontine glioma
chiasmal glioma

Your answer was CORRECT


Explanation
Facial myokymia (involuntary, spontaneous, localised quivering of facial muscles) is caused
by disease in the pons involving the facial nucleus or fascicle. The most common causes
include multiple sclerosis in adults and pontine glioma in children.

Brainstem nuclei crucial for the generation of normal vertical eye movements include all
EXCEPT:
the interstitial nucleus of Cajal (INC)
the rostral interstitial nucleus of the medial longitudinal fasciculus
paramedian pontine reticular formation
the abducens nucleus

Your answer was CORRECT


Explanation
The most important nuclei for the initiation of vertical eye movements are the rostral
interstitial nucleus of the medial longitudinal fasciculus (riMLF) and the interstitial nucleus of
Cajal (INC). The PPRF also has a lesser role.

A 74-year-old hypertensive man suffers a localised stroke causing hallucinations and vivid
deja-vu experiences.

What visual field defect would be MOST likely?


homonymous incongruous inferior defect
binasal superior defect
homonymous incongruous superior defect
bitemporal hemianopia

Your answer was CORRECT


Explanation
Hallucinations and deja vu experiences suggest a temporal lobe lesion, which causes an
incongruous superior quadrantanopia.

In a patient with myasthenia gravis which investigation is LEAST likely to be positive?


anti-calcium-channel antibodies
anti-striated muscle antibodies
anti-muscle-specific receptor tyrosine kinase antibodies
anti-acetycholine receptor antibodies

Your answer was CORRECT


Explanation
Anti-voltage-gated calcium channel antibodies are found in Eaton-Lambert syndrome which is
a paraneoplastic syndrome, often associated with small cell lung cancer. The other antibodies
listed may be found in myasthenia gravis.

An anterior communicating artery aneurysm is most likely to produce:


a bitemporal upper quadrantanopia
a homonymous upper quadrantanopia
a bitemporal lower quadrantanopia
a homonymous lower quadrantanopia

Your answer was CORRECT


Explanation
An anterior communicating artery aneurysm is most likely to compress the upper portion of
the optic chiasm, producing a bitemporal lower quandrantanopia.
Which of the following nerves is positioned most medially in the cavernous sinus:
trochlear nerve
oculomotor nerve
abducent nerve
maxillary nerve

Your answer was CORRECT


Explanation

Figure: Cavernous sinus

The abducent nerve is located most medially in the cavernous sinus. It is more likely to be
involved in cavernous sinus thrombosis because it is less protected than the other nerves III,
IV, V1 and V2 which are against and protected by the lateral wall of the sinus.

All are true of carotid cavernous fistulae EXCEPT:


direct fistulae are caused by meningeal branches of the internal carotid artery
connecting to the cavernous sinus

both indirect and, less commonly, direct fistulae may close spontaneously without treatment

they are typically vision threatening but not life threatening

pulsatile proptosis with a thrill and bruit are typical of direct lesions

Your answer was CORRECT


Explanation
Direct CCF are caused by a connection between the internal carotid artery (ICA) and the
cavernous sinus.
Indirect CCF are caused by:
 meningeal branches of ICA to cavernous sinus
 meningeal branches of external carotid artery to cavernous sinus
 meningeal branches of both the ICA and ECA to the cavernous sinus

A 27-year-old woman with Grave's disease had mild proptosis and conjunctival injection on
initial presentation, which was treated symptomatically with lubricants. She was treated by
endocrinologists with radioactive iodine, subsequently became hypothyroid and was
supplemented with levothyroxine. Now, 6 months later, she re-presents to ophthalmology with
bilateral conjunctival injection and a right relative afferent pupillary defect.

What is the most appropriate next step in management?


follow-up in 6 weeks
urgent repeat of thyroid function tests
oral prednisone
urgent MRI of orbits

Your answer was CORRECT


Explanation
The patient now has an optic neuropathy. Patients with thyroid dysfunction can develop
worsening of thyroid eye disease at any time after appropriate systemic treatment. Thus, the
first consideration should be to rule out optic nerve compression by enlarged extraocular
muscles. MRI scan with STIR sequence would be the best test. Visual fields would also be
helpful. Once the diagnosis has been established, steroids should be initiated to decrease optic
nerve compression until definitive treatment (orbital radiation, orbital decompression) can be
instituted. With a new RAPD present, initial steroid treatment is preferably with pulsed iv
doses to ensure maximal acute effect. Thyroid function tests to ascertain appropriate systemic
treatment should be obtained but are not the first, acute consideration.

Which statement about the optic chiasm is TRUE?


an equivalent number of fibers cross as do not cross
the anterior portion of the chiasm has a high density of macular fibers
the inferior fibers are the first to cross
the chiasm typically lies 1 mm above the anterior pituitary gland

Your answer was CORRECT


Explanation
53% of the retinal ganglion cells cross in the chiasm (this difference is occasionally important
clinically, for example, in producing a contralateral RAPD in an optic tract lesion). The
macular fibers constitute a large portion of the optic chiasm and most decussate in the
posterior chiasm. The chiasm lies approximately 1 cm (not 1 mm) above the anterior pituitary
gland. The inferior nasal retinal fibers cross in the anterior chiasm and were thought to loop
anteriorly in the contralateral optic nerve before traveling posteriorly, leading to the term
Wilbrand's knee.
A 64-year-old man presents with difficulty reading. On examination, there is defective near
convergence, limited upgaze, skew deviation, lid retraction on attempted upgaze and light-
near dissociation of pupils.

What is the most likely diagnosis?

Balint syndrome

Parinaud syndrome

Steele-Richardson-Olszewski syndrome

Wallenberg syndrome

Nothnagel syndrome

Your answer was CORRECT


Explanation
The features described are typical of dorsal midbrain syndrome, also known as Parinaud's
syndrome.

Features of Parinaud's dorsal midbrain syndrome include:

 light-near dissociation
 mydriasis
 Collier's sign (lid retraction in primary position)
 paralysis of convergence and accommodation
 paralysis of upgaze (supranuclear palsy)
 convergence-retraction nystagmus (worsened by downward rotation of OKN drum)
 skew deviation

A patient has light-near dissociation of pupil responses. On oculomotility examination, he has


upgaze palsy, skew deviation and nystagmus best elicited with the OKN drum rotating
downwards.

Which of the following is MOST likely to cause this clinical picture?

pinealoma

blockage of posterior inferior cerebellar artery

nasopharyngeal carcinoma

acoustic neuroma

Your answer was CORRECT


Explanation
This case is in keeping with Parinaud's dorsal midbrain syndrome.

Parinaud's is characterised by:

 light-near dissociation
 mydriasis
 Collier's sign (lid retraction in primary position)
 paralysis of convergence and accommodation
 paralysis of upgaze (supranuclear palsy)
 convergence-retraction nystagmus (best elicited with OKN drum rotating downwards)
 other possible features: skew deviation, 3rd or 4th nerve palsy, INO

Causes of Parinaud's:
 hydrocephalus
 pinealoma
 head injury
 AV malformation
 MS
 vascular
 degenerative (Wernicke's)

Which one of the following is NOT a typical feature of Adie's pupil?

hypersensitivity to parasympathomimetic drugs

anisocoria usually greater in the dark

vermiform movement of iris border

light-near dissociation

Your answer was CORRECT


Explanation
An Adie's tonic pupil is typically dilated (though some chronic Adie's pupils can be small). In
the light, the abnormally large Adie's pupil does not constrict properly and the anisocoria is
more pronounced.

The most common ocular manifestation of congenital cytomegalovirus infection is:

Peters' anomaly

cataract

retinochoroiditis

microphthalmia

Your answer was CORRECT


Explanation
Ocular features of congenital cytomegalovirus infection:

 chorioretinitis (most common feature)


 optic atrophy
 pigmentary retinopathy
 strabismus
Systemic features of congenital CMV:
 hepatosplenomegaly
 pneumonia
 sensorineural hearing impairment
 microcephaly
 seizures

The ptosis associated with Marcus Gunn's syndrome is caused by aberrant connections
between the levator muscle and which cranial nerve?
seven
five
ten
nine
three

Your answer was CORRECT


Explanation

Figure: Marcus Gunn jaw wink

Levator innervation in Marcus Gunn's syndrome is derived from the trigeminal supply (CN V)
to the pterygoids and masseters.

Which of the following statements about light-near dissociation syndromes is TRUE?

the most common aetiology for the dorsal midbrain syndrome in a patient older than 60 years
is multiple sclerosis

Argyll Robertson pupils react to light, but do not have a near response

Argyll Robertson pupils are characteristically small and regular


the most common aetiology of the dorsal midbrain syndrome in a child younger than 10
years is a pineal gland tumor

Your answer was CORRECT


Explanation
Characteristics of Argyll Robertson pupils include:

 small and irregular pupils


 bilateral
 light-near dissociation (pupils respond to near, not light)
 caused by syphilis, but also DM, trauma, degeneration, alcohol, MS
 tends to occur in the elderly
 normal time for pupil constriction, dilation and accommodation (unlike a tonic pupil)
 pupils do not respond to cocaine 4-10%
Regarding Parinaud's dorsal midbrain syndrome, the most common causes are:
 infants: hydrocephalus
 young children: pinealoma
 young adults: head trauma and multiple sclerosis
 older adults (60 years): stroke

Which one of the following conditions would have positive forced ductions?

convergence excess esotropia

chronic progressive external ophthalmoplegia

myasthenia gravis

thyroid eye disease

Your answer was CORRECT


Explanation
Positive forced ductions indicate a restrictive process that may be caused by:

 thyroid eye disease


 blow-out fractures
 fat adherence syndrome
 Brown's syndrome
 muscle fibrosis (post-trauma or inflammation)
Disorders of the neuromuscular junction (myasthenia) and of the muscle itself (CPEO) do not
cause restriction.

A 23-year-old student discovers her right pupil to be several millimeters larger than her left
pupil. She denies diplopia, but she has had several headaches in the past week.
Each of the following findings below would be helpful in suggesting an underlying cause for
anisocoria EXCEPT:
a right relative afferent pupillary defect
2 mm of right upper eyelid ptosis
a small right hypotropia on upgaze
segmental contraction of the right iris

Your answer was CORRECT


Explanation
A relative afferent pupillary defect never causes anisocoria because the pupillary fiber
decussations in the chiasm and posterior commissure ensure equal efferent input to both iris
sphincter muscles.

Right upper eyelid ptosis and a right hypotropia on upgaze could both be signs of third nerve
dysfunction. Segmental iris contraction is a sign of Adie's tonic pupil.

Which of the following features is NOT consistent with a parietal lobe lesion?

right-left confusion

homonymous hemianopia denser inferiorly

agnosia

OKN abnormality when drum is rotated away from the lesion

Your answer was CORRECT


Explanation
Parietal lobe lesions cause a defective OKN response when the drum is rotated towards the
side of the lesion (as the parietal lobe controls ipsilateral pursuit). Unlike temporal lobe
lesions, which produce hemianopias that are denser superiorly, parietal lobe lesions produce
hemianopias that are denser inferiorly.

Parietal lobe lesions also cause a variety of other neurologic manifestations.

Dominant parietal lobe lesions cause:

 dyscalcula
 dysgraphia
 left-right disorientation
 finger agnosia
 alexia
 speech disturbance

Note the first four features above constitute Gerstmann syndrome

Non-dominant parietal lobe lesions cause:


 hemispatial neglect
 dressing apraxia

All are true of craniopharyngioma EXCEPT:


CT scans will typically show suprasellar calcification
bitemporal field defects which begin superiorly are typical
it arises from remnants of Rathke's pouch
it consists of both solid and cystic components

Your answer was CORRECT


Explanation
Craniopharyngioma compresses the optic chiasm from above, causing a bitemporal field
defect that initially affects the infero-temporal visual fields. Other options are true.

Which of the following statements about Leber's hereditary optic neuropathy is FALSE?

a small percentage of patients will have partial or complete recovery

all offspring of a female carrier are either affected or carriers

less than 1% of female carriers will be affected

there is generally sequential asymmetric bilateral involvement

Your answer was CORRECT


Explanation
LHON is characterised by:

 mitochondrial inheritance
 males more commonly affected
 age of onset usually 10-30 years
 10% of female carriers are affected
 50% of male carriers are affected
 generally sequential bilateral optic nerve involvement
 smoking and alcohol appear to be risk factors for vision loss
 acute: optic disc swelling, hyperaemia, dilated telangiecactic vessels but no leak on FFA
 chronic: optic atrophy
 a small percentage have partial or complete recovery of vision

What percentage of patients with myasthenia gravis have thymomas visible on CT scanning?
50%
10%
1%
25%
75%

Your answer was CORRECT


Explanation
Radiologic screening is mandatory for all myasthenics in order to discover thymic
enlargement.

Which is FALSE regarding optokinetic nystagmus (OKN)?


in congenital motor nystagmus there can be a reversal of the OKN
the slow phase occurs in the direction of movement of the visual stimulus
the frontal lobes control the slow pursuit movement
an OKN response in an infant indicates there is some visual input present

Your answer was CORRECT


Explanation
In the OKN response, the parieto-occipital lobe controls the slow pursuit component, whereas
the frontal lobes control the saccadic component. Other options supplied are true.

Which of the following statements about optic nerve glioma is FALSE?

optociliary shunts may be seen on presentation

it is more likely to be malignant in children than in adults

the most common systemic association is neurofibromatosis

a fusiform enlargement of the optic nerve on MRI is consistent with the diagnosis

Your answer was CORRECT


Explanation
Figure: Axial T2 MRI of left ON glioma.

Optic nerve glioma is characterised by:

 more frequent in children (90% in the first 2 decades)


 common association with NF-1 (14%-60% of all gliomas)
 can present with: vision loss, strabismus, proptosis
 optociliary shunts may be present
 malignant gliomas are rare (more likely in adults than in children)
 CT/MRI: optic nerve is enlarged, and the mass may either be fusiform or exophytic in
appearance; additionally the nerve may be elongated with kinking or buckling; the lesion is iso
to hypointense on T1 MRI but hyperintense on T2

A 45-year-old patient presents via a routine referral from his opticians. On Humphrey visual
field 24-2 testing he has bilateral supero-temporal defects. His IOP is 17 mmHg bilaterally.
Gonioscopy reveals open irido-corneal angles. On examination, his optic discs are healthy
except for subtle nasal tilting and an area of peripapillary atrophy infero-nasally.

How would you manage this patient?


MRI brain scan
reassure and discharge
observe with repeat visual fields
commence prostaglandin analogue with review in 3 months

Your answer was CORRECT


Explanation
Nasal tilting and an area of peripapillary atrophy infero-nasally MAY explain supero-temporal
defects extending from the blind spot. However, supero-temporal defects are not characteristic
of glaucoma and a chiasmal lesion must always be considered. The most suitable management
is to perform an MRI brain scan to exclude serious, life-threatening pathology.
This question came in the FRCS (Glasgow) Part 2 in October 2014

A lesion in which of the following locations will cause difficulty with eye lid closure?
cavernous sinus
ventral midbrain
cerebellopontine angle
lateral medulla

Your answer was CORRECT


Explanation
A cerebello-pontine angle tumour affects cranial nerves 5, 6, 7 and 8. A deficit in cranial
nerve 7, which supplies the orbicularis oculi, will affect eyelid closure.

This question appeared in the 2014 FRCOphth Part 2.

Uhthoff's phenomenon describes:


the inability to distinguish faces
the ability to see moving objects but not stationary ones
an electric shock sensation with neck flexion
the decrease in vision with an increase in body temperature
the mistaken perception of 3-dimensional motion in an object swinging in one plane

Your answer was CORRECT


Explanation
Uhthoff's symptom occurs with optic neuritis and is a decrease in vision with an increase in
body temperature. Exercise or hot showers may trigger this symptom.

Lhermitte's sign is the electric shock sensation with neck flexion and is found in patients with
multiple sclerosis.

Pulfrich phenomenon occurs where lateral motion (e.g. pendulum) is perceived by the visual
cortex as having depth (circular motion) due to relative difference between the visual
pathways from an optic nerve lesion.

Prosopagnosia is the inability to distinguish faces and occurs with bilateral medial
occipitotemporal lesion.

The Riddoch phenomenon occurs in patients with cortical blindness who are able to perceive
objects in motion, but cannot see stationary objects. Such patients may suffer from Anton
syndrome, where they deny they are blind even in the face of clear evidence of their blindness.

The seventh cranial nerve exits the brainstem:


from the dorsal surface of the pons
at the cerebellopontine angle
at the border of the midbrain and pons laterally
from the ventral midbrain

Your answer was CORRECT


Explanation
The seventh cranial nerve exits the brainstem at the cerebellopontine angle.

The most common cause of third nerve palsy in the paediatric population is:

traumatic

migrainous

congenital

tumor

inflammatory

Your answer was CORRECT


Explanation
The aetiologies for paediatric third nerve palsy, in descending frequency are:

 congenital
 traumatic
 inflammatory
 migrainous
 neoplastic

In adults, the most common cause of third nerve palsy is:

traumatic

migrainous

microvascular

inflammatory

tumour

Your answer was CORRECT


Explanation
The aetiologies for adult third nerve palsy, in descending frequency, are:
 microvascular
 aneurysmal
 traumatic
 neoplastic

A 32-year-old woman has developed diplopia on upgaze over the past 4 weeks. Her GP
requested an MRI of the brain which was reported as normal. On examination, you note she
exibits Dalrymple's sign.

Which investigation is most likely to yield the diagnosis?


anti-acetylcholine receptor antibodies
echocardiogram
thyroid function test
ocular B-scan
tensilon test

Your answer was CORRECT


Explanation
Dalrymple's sign refers to lid retraction in primary position and is a sign of thyroid eye disease
(TED) caused by overaction of Muller's muscle.
von Graefe's sign refers to lid lag on downgaze and is another feature of TED. Thyroid
function tests in this case would be most likely to yield the diagnosis.

A 52-year-old man with diabetes presents with a painful partial third nerve palsy. On his first
follow-up visit, you notice that, when looking down, his upper eyelid appears to retract or lag.

This finding essentially rules out the possibility of:

trauma

meningioma

syphilitic gumma

aneurysm

diabetic third nerve palsy

Your answer was CORRECT


Explanation
Third nerve aberrant regeneration never occurs with diabetic or microvascular palsy. Aberrant
regeneration of the third nerve implies another aetiology, most commonly aneurysm, tumor,
inflammation, or trauma.

Classic findings of aberrant 3rd regeneration include:


 inverse Duane's sign: lid elevation on adduction
 pseudo-von Graefe sign: lid elevation on depression
 pseudo-Argyl-Robertson: pupil constriction on adduction
 pupil constriction on depression or elevation
 persistent vertical gaze limitation secondary to simultaneous contraction of superior and
inferior recti

Which of the following statements about optic nerve meningioma is TRUE?

most patients will have associated neurofibromatosis

these tumours are more likely to be aggressively malignant in children than in adults

optic nerve meningiomas primarily affect children

on CT scanning with contrast the central portion of the optic nerve is typically hyper-intense

Your answer was CORRECT


Explanation
Figure: CT with contrast showing left ON meningioma with classic rail-road or tram-track
sign, caused by hyperintense tumour surrounding a non-enhancing optic nerve.

Optic nerve meningioma is characterised by:

 adult-onset mainly
 female preponderance
 association with neurofibromatosis (but small proportion)
 optociliary shunts may be present
 more likely to be aggressively malignant in children
 CT contrast: the peripheral tumour surrounding the nerve may show enhancement, while
the optic nerve is non-enhancing, resulting in the railroad track sign

Optic nerve glioma is characterised by:


 more frequent in children (90% in the first 2 decades)
 common association with NF-1 (14%-60% of all gliomas)
 can present with: vision loss, strabismus, proptosis
 optociliary shunts may be present
 malignant gliomas are rare (more likely in adults than in children)
 CT/MRI: optic nerve is enlarged, and the mass may either be fusiform or exophytic in
appearance; additionally the nerve may be elongated with kinking or buckling; the lesion is iso
to hypointense on T1 MRI but hyperintense on T2

A bilateral centrocoecal scotoma on visual field testing is MOST likely to be produced by:
hereditary optic neuropathy
glaucoma
optic nerve pit with serous retinal detachment
toxic/nutritional optic neuropathy
optic neuritis

Your answer was CORRECT


Explanation
A centrocoecal scotoma involves fixation and the blind spot and can be produced by toxic-
nutritional neuropathies (always bilateral) or an optic pit with serous retinal detachment
(usually unilateral). Optic neuritis typically produces a unilateral central scotoma, though
many visual field patterns may be seen.

A patient with a right INO will typically show nystagmus on:

downgaze

left gaze

right gaze

upgaze

Your answer was CORRECT


Explanation
Patients with internuclear ophthalmoplegia classically have abduction nystagmus of the
contralateral eye.

INO is characterised by the triad of:

 failure of ipslateral adduction (often best seen with saccades)


 ataxic nystagmus of abducting contralateral eye
 normal convergence (if posterior INO)
However, other features can include:
 slowing of saccades in adducting eye
 asymmetric OKN response when drum rotated towards MLF lesion
 horizontal nystagmus of adducting eye
 impaired horizontal vestibulo-ocular reflex
 impaired convergence (for Cogan's anterior INO)
 manifest exotropia (for bilateral INO)
 vertical nystagmus
 impaired vertical pursuit
 impaired vertical vestibulo-ocular reflex
 difficulty maintaining upgaze

Myasthenia gravis patients are at higher risk for all of the following EXCEPT:
thymoma
Graves' disease
systemic lupus erythematosus
leukaemia

Your answer was CORRECT


Explanation
Patients with myasthenia gravis are at risk of other autoimmune diseases, including SLE,
rheumatoid arthritis, and hyperthyroidism. Thymic hyperplasia and thymomas are also more
common, and patients should have a chest CT to investigate this possibility. Leukaemia is not
a recognised association.

Which is FALSE regarding the Hess chart?


the visual targets are red lights
to test the left eye, the red lens is placed on the right eye and green in front the left
the patient should be seated 1 meter from the screen
each square represents 5 degrees

Your answer was CORRECT


Explanation
In a Hess chart, each square is 5 degrees. The central field is 15 degrees from primary position
while the peripheral field is 30 degrees from primary position. The patient is seated at 50cm
from the screen. To test the left eye, the red lens is placed in front of the right eye, and green
lens in front the left eye. The red targets are seen only by the right eye (fixing eye). The
patient is asked to superimpose the green light onto the red light and these positions are
marked.

The characteristic posterior segment tumour of von Hippel-Lindau disease is:

diffuse choroidal haemangioma

retinal capillary haemangioma

racemose angioma

retinal cavernous haemangioma

retinal astrocytoma
Your answer was CORRECT
Explanation

Figure: Retinal capillary haemangioma associated with Von Hippel Lindau

Retinal lesions and phakomatoses:


 retinal astrocytoma: NF-1 and tuberous sclerosis
 retinal capillary haemangioma: von Hippel Lindau
 retinal cavernous haemangioma: rare, no specific association but can occur with similar
skin and CNS lesions
 choroidal cavernous haemangioma: Sturge-Weber syndrome
 racemose angioma: Wyburn-Mason
 combined hamartoma of RPE and retina: NF-2 (weaker associations with NF-1, Gorlin
Goltz)

An ischaemic event in the left temporal lobe would most likely produce:
a bitemporal heminanopia
a contralateral upper quadrantanopia
a contralateral lower quadrantanopia
a contralateral congruous homonymous hemianopia
a contralateral incongruous homonymous hemianopia
Your answer was CORRECT
Explanation
The temporal lobe receives those fibres of the geniculocalcarine tract (optic radiation)
corresponding to the inferior retina, which is the upper half of the visual field. It will therefore
result in a contralateral upper quadrantanopia.

A teenager with deafness, diabetes and short stature complains of nyctalopia, diplopia and
bilateral ptosis. He has been lethargic and unsteady on his feet recently.

What is the most likely diagnosis?

Kearns-Sayre syndrome

oculopharyngeal dystrophy

myasthenia gravis

myotonic dystrophy

Your answer was CORRECT


Explanation
Figure: Pigmentary retinopathy in Kearns-Sayre syndrome

The features described in the question above are most suggestive of Kearns-Sayre syndrome.

Kearns-Sayre syndrome is characterised by:

 mitochondrial inheritence
 ragged red fibres on muscle biopsy
 presentation in 1st or 2nd decades
 ptosis
 external ophthalmoplegia
 cardiac conduction defects
 deafness
 diabetes
 short stature
 pigmentary retinopathy

Myotonic dystrophy would need to be entertained in the differential of this case, but is not
known to be associated specifically with diabetes or short stature.

Features of myotonic dystrophy include :


 autosomal dominant inheritance
 presentation in 2nd decade or beyond
 hypotonia
 mental retardation
 respiratory insufficiency
 cardiac conduction defects
 males sub-fertile
 proximal muscle weakness
 mournful facial expression
 frontal balding
 ptosis
 external ophthalmoplegia
 Christmas-tree cataracts
 strabismus
 pigmentary retinopathy in butterfly pattern

Retinal racemose haemangioma is a feature of:


Wyburn Mason syndrome
von Hippel Lindau syndrome
Louis-Bar syndrome
incontinentia pigmentii
Sturge-Weber syndrome

Your answer was CORRECT


Explanation
A retinal racemose haemangioma (shown in the image above) is associated with Wyburn-
Mason syndrome, which is a sporadic condition with ipsilateral AV malformations involving
the midbrain, basofrontal region and posterior fossa.

Which is the most appropriate investigation for a patient with suspected demyelination?
CT with contrast
MRI with STIR sequence
MRI with FLAIR sequence
MR angiography

Your answer was CORRECT


Explanation
MRI with FLAIR sequence (fluid-attenuated inversion recovery) is employed for fluid
suppression and provides good delineation of paraventricular pathology (e.g. paraventricular
plaques), making it the preferred imaging for MS.

MRI with STIR sequence is the best modality for monitoring thyroid eye disease.

Each of the following is a characteristic of spasmus nutans EXCEPT:

head bobbing

usually disappears within 2 years

optic atrophy

monocular or dissociated nystagmus, which is rapid and of small amplitude

Your answer was CORRECT


Explanation
The classic triad of spasmus nutans includes:

 monocular or dissociated small-amplitude nystagmus


 head bobbing
 torticollis
The differential diagnosis for this disorder includes:
 chiasmatic gliomas
 subacute necrotising encephalomyopathy
The above two disorders may produce optic atrophy, irritability, vomiting, and increased
intracranial pressure. Spasmus nutans should be considered only after these two disorders are
excluded. Spasmus nutans usually disappears within 2 years of onset.

Astrocytic hamartomas of the retina or optic nerve head may be seen in:
ataxia-telangiectasia
incontinentia pigmentii
Wyburn-Mason
neurofibromatosis
Sturge-Weber syndrome

Your answer was CORRECT


Explanation
Figure. Retinal astrocytoma (astrocytic hamartoma)

Astrocytic hamartomas can be associated with tuberous sclerosis or neurofibromatosis, or they


can be sporadic. They are located in the nerve fiber layer of the retina or occasionally the optic
disc.

The differential diagnosis for presumed posterior ischaemic optic neuropathy (PION) should
include all of the following EXCEPT:

giant cell arteritis

post-coronary artery bypass graft surgery

radiation optic neuropathy

malignant hypertension

Your answer was CORRECT


Explanation
The differential diagnosis for posterior ischemic optic neuropathy (PION) should include:

 radiation optic neuropathy


 status post-coronary artery bypass graft
 anaemia
 acute systemic hypotension
 giant cell arteritis
Malignant hypertension produces posterior segment features such as AV nipping, exudates
and haemorrhages, which are not compatible with PION.

A 30-year-old man has recently suffered a closed head injury in an accident and complains of
vertical diplopia since then. Your orthoptist reports right hypotropia with left inferior oblique
over-action.

What is the most likely finding on head tilt testing?


no change in image separation either right or left tilt
increased image separation on head tilt to the right
increased image separation on both right and left tilt
increased image separation on head tilt to the left

Your answer was CORRECT


Explanation
This patient has a left fourth nerve palsy. The Bielschowski head tilt will cause worsening of
the hypertropia (that is, greater separation of images for the patient) on ipsilateral tilt, i.e. on
tilt to the left.

The optic canal transmits all of the following EXCEPT:


post-ganglionic parasympathetic fibres
meningeal coverings
the ophthalmic artery
the optic nerve

Your answer was CORRECT


Explanation
The optic canal transmits sympathetic nerves in association with the ophthalmic artery but
does not transmit parasympathetic fibres. All other options are true.

All of the following are characteristic of Leber's hereditary optic neuropathy EXCEPT:
age of onset 15 to 35 years
smoking is a risk factor for vision loss
late disc leakage on fluorescein angiography during the acute phase
male preponderance
disc swelling and hyperaemia in the acute phase

Your answer was CORRECT


Explanation
In LHON, the disc is swollen (NFL swelling) and hyperaemic in the acute phase with dilated
disc caillaries but there is no leakage on fluorescein angiography. It is therefore a pseudo-
oedema of the optic disc, by comparison to true disc oedema where there is fluorescein
leakage.

A 4-year-old girl has a progressive right exotropia noticed by parents over the past 12 months.
Her visual acuity is 6/60 right eye, 6/6 left eye. Dilated fundoscopy reveals a slightly elevated,
charcoal grey mass centred at the optic disc and involving the retina. The lesion appears to
involve both the RPE and retina and it is covered by thickened grey-white retinal and
preretinal glial tissue. There is no associated retinal detachment, haemorrhage, exudation, or
vitreous inflammation. There is a strong family history of hearing loss and skin lesions.

What is the most likely diagnosis?


Tuberous sclerosis
Neurofibromatosis type 2
Sturge Weber
Louis Bar syndrome
von Recklinghausen

Your answer was CORRECT


Explanation
Figure: Combined hamartoma of RPE and retina

The description of the retinal lesion in the question above is consistent with a combined
hamartoma of the RPE and retina, which is associated with NF-2. Bilateral acoustic neuromas
and skin neurofibromas are common features of NF-2.

Which of the following is BEST for treatment of acute, severe migraine headache?
sumatriptan
paracetamol
methysergide
aspirin
propranolol

Your answer was CORRECT


Explanation
Sumatriptan is a serotonin 5-HT1 agonist available orally, by injection, or as a nasal spray.
The triptan class of anti-migraine drugs are used for symptomatic relief of migraines but are
contraindicated in patients with basilar artery migraine. They can also cause myocardial
infarction and should be used with caution in patients with severe hypertension or coronary
artery disease. Methysergide (antagonist at some serotonin receptors, partial agonist at others)
is a prophylactic treatment against recurrent migraine, but is not useful for pain relief in an
acute attack. Use of methysergide is limited by its side-effect profile, specifically retro-
peritoneal fibrosis. Propranolol is a beta blocker, which can be helpful for migraine
prophylaxis.

A 23-year-old presents with painless, rapid loss of vision in the right eye to 6/36. There are
peripapillary telangiectasia and a dense centrocoecal scotoma. The temporal border of the
optic disc is swollen but there is no leak on FFA.

These findings are most characteristic of which condition?

Leber hereditary optic neuropathy

spheno-orbital meningioma

demyelinating optic neuritis

dominant optic neuropathy of Kjer

Wolfram syndrome

Your answer was CORRECT


Explanation

Figure: the acute phase of Leber's hereditary optic neuropathy with disc hyperaemia, minimal
disc swelling and peripapillary telangiectatic vessels.

In the question above, the peri-papillary telangiectasia and pseudo-oedema of the optic disc
(i.e. no leak on FFA) are characteristic of Leber's optic neuropathy.

LHON is characterised by:


 mitochondrial inheritance
 typically painless vision loss
 males more commonly affected (80% of cases)
 age of onset usually 10-30 years
 10% of female carriers are affected
 50% of male carriers are affected
 generally sequential bilateral optic nerve involvement
 smoking and alcohol appear to be risk factors for vision loss
 acute: optic disc swelling, hyperaemia, dilated telangiecactic vessels but no leak on FFA
 chronic: optic atrophy
 a small percentage have partial or complete recovery of vision
 type of mutation most important prognostic factor for recovery
 11778 most common mutation (poor prognosis)
 association with ECG cardiac conduction abnormalities, especially preexcitation
syndrome
 association with dystonia and MS-like illness

A 26-year-old ITU nurse notices her right pupil is several millimeters larger than her left. She
is otherwise systemically well. On examination, her right pupil does not react to light.
Otherwise neurologic and slit lamp examination is normal.

The next diagnostic step would be:


1 drop of 10% cocaine both eyes
1 drop of 1% pilocarpine both eyes
cerebral arteriogram
1 drop of 0.1% pilocarpine both eyes

Your answer was CORRECT


Explanation
The most likely cause is pharmacologic dilatation because of the lack of other findings, but the
differential still includes partial third nerve dysfunction and Adie's pupil. Pharmacologic
dilatation can be confirmed if her right pupil does NOT constrict to 1% pilocarpine. However,
if you proceed with 1% pilocarpine and her pupils both constrict, she still could have either an
Adie's tonic pupil or a partial oculomotor nerve palsy. At that point, it is too late to use the
0.1% pilocarpine drops. Thus, testing for denervation supersensitivity (0.1% pilocarpine)
should be done first. Cerebral arteriogram is appropriate if testing indicates third nerve
dysfunction.

A 77-year old lady presents with sudden onset diplopia. On examination, there is right ptosis,
right exotropia with dilated, non-reactive right pupil. On neurological examination, you detect
left sided weakness.

What is the diagnosis?

Benedikt's syndrome
Nothnagel's syndrome

Weber's syndrome

Claude's syndrome

Foville's syndrome

Your answer was CORRECT


Explanation
Features are most in keeping with Weber's syndrome. Note, both Benedikt's and Weber's can
have third nerve palsy with contralateral hemi-paresis, but in Benedikt's there is associated
contralateral tremor from involvement of the red nucleus; also the hemi-paresis is more
pronounced in Weber's than in Benedikt's.

Third nerve fascicle syndromes:

Benedikt's syndrome, which is characterised by: 1

 ipsilateral CN III palsy


 contralateral cerebellar ataxia with intention tremor
 contralateral hemiparesis (usually partial) with hyperactive tendon reflexes
Claude syndrome, which is characterised by: 2

 ipsilateral 3rd nerve palsy


 contralateral cerebellar ataxia
Weber syndrome, which is characterised by:
 ipsilateral 3rd nerve
 contralateral hemi-paresis
Nothnagel syndrome, which is characterised by:
 ipsilateral 3rd nerve
 ipsilateral cerebellar ataxia

1. Reference for Benedikt syndrome


2. Reference for Claude syndrome
3. General reference for 3rd nerve syndromes

Which optic disc anomaly may be found in conjunction with aniridia?

persistent myelination of the nerve fibre at the disc

optic nerve pit

morning glory

optic disc coloboma

optic nerve hypoplasia


Your answer was CORRECT
Explanation
Aniridia can present with optic disc hypoplasia and foveal hypoplasia.

Optic disc hypoplasia can be associated with:

 septo-optic dysplasia
 aniridia
 fetal alcohol syndrome
 maternal: LSD, quinine, and phenytoin

Which of the following is MOST likely to cause a bitemporal hemianopia?


low flow carotid-cavernous fistula
high flow carotid-cavernous fistula
sphenoid wing meningioma
craniopharyngioma
optic nerve glioma

Your answer was CORRECT


Explanation
A craniopharyngioma often compresses the optic chiasm causing bitemporal hemianopia and
pituitary abnormalities. Initially, the compression occurs from above, causing a bitemporal
hemianopia that initially affects the superior fibres, corresponding to the infero-temporal
visual field.

The signs below are all typical of Miller Fisher syndrome EXCEPT:
ataxia
ophthalmoplegia
distal limb weakness
areflexia

Your answer was CORRECT


Explanation
Miller Fisher syndrome is a variant of Guillain-Barre syndrome. It is classically associated
with the triad of ataxia, areflexia and ophthalmoplegia. Unlike GBS, it does not cause an acute
ascending motor axonal neuropathy. Most patients with Miller Fisher syndrome have anti-
GQ1b IgG antibodies.

muscle is a synergist to the lateral rectus


A patient develops variable ophthalmoplegia, difficulty with balance and co-ordination,
areflexia and a descending paralysis after a bout of traveller's diarrhoea.

Which test is MOST likely to confirm your clinical suspicions?

anti-ACh receptor antibodies

anti-dsDNA

anti-voltage gated calcium channel antibodies

anti-GQ1b antibodies

Your answer was CORRECT


Explanation
This patient appears to have Miller Fisher syndrome, which is a variant of Guillan-Barre
syndrome, characterised by:

 ophthalmoplegia
 ataxia
 areflexia
 descending paralysis (as opposed to GBS where it is ascending)
 positive anti-GQ1b antibodies in 90%
 treatment by plasmapheresis and iv immunoglobulin

A 10-year-old boy presents to clinic with non-specific visual symptoms, including difficulty
looking up. Systemic enquiry reveals occasional morning headaches but denies any nausea or
vomiting. Examination reveals visual acuity of 6/12 in the right eye and 6/7.5 in the left eye.
The patient has marked symmetric weakness of upgaze bilaterally. His pupils are 7 mm and
are poorly reactive to light, with better reaction to a near target. There is approximately 2 mm
of superior scleral show bilaterally. Fundus examination suggests optic atrophy in both eyes.
Systemic review uncovers an increased consumption of water, with frequent urination at night.

The most likely diagnosis is:

cerebellar astrocytoma

pontine glioma

pinealoma

chiasmal glioma

hereditary optic atrophy

Your answer was CORRECT


Explanation
Features suggest Parinaud's dorsal midbrain syndrome, which is characterised by:
 light-near dissociation
 mydriasis
 Collier's sign (lid retraction in primary position)
 paralysis of convergence and accommodation
 paralysis of upgaze (supranuclear palsy)
 convergence-retraction nystagmus (worsened by upward rotation of OKN drum)
 skew deviation
 papilloedema or optic atrophy (if ventricular flow compromised)

The most common cause of Parinaud's depends on the patient age-group:


 infants: hydrocephalus
 young children: pinealoma
 young adults: head trauma and multiple sclerosis
 older adults (60 years): stroke

Which pattern of nystagmus is most localising?

upbeat nystagmus

downbeat nystagmus

seesaw nystagmus

periodic alternating nystagmus

Your answer was CORRECT


Explanation
Upbeat nystagmus can be caused by:

 Lesions of the posterior fossa: including the anterior vermis and lower brainstem
 Drugs
 Wernicke's encephalopathy
Downbeat nystagmus may be caused by:
 craniocervical junction lesions (e.g., Arnold-Chiari malformation)
 intoxications (alcohol, lithium, phenytoin)
 paraneoplastic syndrome
Periodic alternating nystagmus:
 craniocervical junction
 posterior fossa disease
Seesaw nystagmus results from third ventricle tumors or diencephalic lesions (including
parasellar/chiasmal lesions) involving the connections to the interstitial nucleus of Cajal (INC)
and is therefore the most localising of those mentioned.

Convergence-retraction nystagmus is a feature of Parinaud's dorsal midbrain syndrome, and is


caused by lesions of the dorsal midbrain such as pinealomas.

Dissociated divergence nystagmus is a feature of posterior fossa lesion


A 31-year-old woman complains of increasing right deafness. She has diplopia looking into
the distance. She also has moderate weakness and numbness on the right side of her face, and
a right-sided partial ptosis and miosis.

What is the most likely diagnosis?


Millard-Gubler syndrome
Foville syndrome
Claude syndrome
Raymond syndrome
Parinaud dorsal midbrain syndrome

Your answer was CORRECT


Explanation
This lady has 5th, 6th, 7th and 8th nerve palsy with Horner's syndrome. These findings are in
keeping with Foville syndrome, which involves a lesion of the 6th nerve fasciculus as it passes
through the para-pontine reticular formation (PPRF) with effects on 5th, 6th, 7th and 8th
nerves and a central Horner’s syndrome.

An internal carotid artery dissection is MOST likely to cause a:

third-order Horner's

first-order Horner's

second-order Horner's

fourth-order Horner's

Your answer was CORRECT


Explanation
Internal carotid artery dissection can damage the sympathetic plexus as it ascends along the
artery which corresponds to a third-order Horner's.

Aetiology of Horner's:

 First Order: (lesions of hypothalamus and brain stem)


o stroke
o tumour (e.g. glioma)
o demyelination
o trauma
o syrinx (syringomyelia and syringobulbia)
 Second Order: (lesions of thoracic outlet, mediastinum, pulmonary apex, neck)
o subclavian artery aneurysm
o cervical rib
o Pancoast tumour
o mediastinal tumours
o thyroid malignancy
o surgery to neck
 Third order: (lesions of superior cervical ganglion, ICA, cavernous sinus, sellar regions)
o superior cervical ganglion trauma or invasion
o ICA dissection, aneurysm, thrombosis, invasion by tumour, stenting
o cavernous sinus thrombosis, aneurysm, inflammation, tumour
o invasive pituitary tumours
o cluster headache

Note: there is no such thing as a fourth-order Horner's!

Which cranial nerve is most commonly traumatised with a closed head injury?
Cranial nerve III
Cranial nerve II
Cranial nerve IV
Cranial nerve VI

Your answer was CORRECT


Explanation
The trochlear nerve has the longest intracranial course and is the most commonly injured
nerve following closed head injury.

Cranial nerve VI palsy can also be injured with head trauma. Any condition that causes
increased intracranial pressure may result in cranial nerve VI palsy, called the false-localising
sign.

A patient with a recent cerebrovascular accident has difficulty in pursuit to the left and
asymmetric responses to an OKN drum.

What is the most probable visual field defect?


left superior quadrantanopia
bitemporal hemianopia
left inferior quadrantanopia
right inferior quadrantanopia
right superior quadrantanopia

Your answer was CORRECT


Explanation
Parietal lobe lesions cause failure of ipsilateral pursuit. They also cause asymmetric responses
to an OKN drum, with poor OKN response when the drum is rotated towards the side of the
lesion. Thus, in the patient described above, the lesion is in the left parietal lobe.

Parietal lobe lesions classically cause a contralateral homonymous inferior quadrantanopia.

All statements regarding third cranial nerve palsy are true EXCEPT:
aneurysmal third nerve palsy is likely to involve the pupil
diabetic third nerve palsy is very rarely painful
spontaneous resolution is rare in aneurysmal third nerve palsy
diabetic third nerve palsy involves the pupil in 10-20% of cases
it may progress from pupil-sparing to pupil-involving over time

Your answer was CORRECT


Explanation
Third nerve palsy caused by aneurysm is almost invariably painful; while palsy from diabetes
may be painful or painless. The presence of pain is not a particularly specific or helpful sign in
determining the underlying aetiology of a third nerve palsy.

A 62-year-old man presents with acute vertigo and falls to the right. He is a heavy smoker. On
examination, he has a right-sided Horner's, incoordination and ataxia. His speech is slurred,
and he has difficulty swallowing fluids, which come up through his nose. On examination of
his limbs, there is normal power, tone and reflexes in all 4 limbs; sensory loss is apparent with
diminished pain and temperature down the left.

Which artery has been infarcted:


left posterior inferior cerebellar
right posterior cerebral
right basilar
left posterior cerebral
right posterior inferior cerebellar

Your answer was CORRECT


Explanation
This is the lateral medullary syndrome, accounted for by infarction of the ipsilateral posterior
inferior cerebellar artery. The cerebellum, IX and X nuclei, and descending sympathetic tracts
(giving a Horner's) are affected on the same side. The spinothalamic tract is also affected; this
decussates, therefore giving contralateral long tract loss of pain and temperature. The dorsal
columns are relatively spared. The trigeminal nerve may also be affected, producing ipsilateral
facial sensory abnormalities.

The vertebral artery is derived from:


the basilar artery
the ascending aorta
the subclavian artery
the spinal artery
Your answer was CORRECT
Explanation
The vertebral arteries are derived from the subclavian artery. They give rise to the anterior
spinal arteries and the posterior inferior cerebellar arteries. They then join together to form the
basilar artery.

A 14-year-old girl complains of gradual-onset right blurred vision. She has a relative afferent
pupillary defect and a CT scan shows a fusiform enlargement of the right optic nerve.

The LEAST likely finding is:


right upper lid swelling with an S-shaped lid contour
good vision after surgical resection of the lesion
propensity to develop CNS tumors
a brother with the same problem

Your answer was CORRECT


Explanation
This girl has an optic nerve glioma. Optic nerve gliomas have a fusiform appearance on CT
scan. Optic nerve meningiomas, conversely, appear as parallel thickening of the optic nerve
sheath called railroad track sign with associated calcification.

The only way to remove an optic nerve glioma is to remove the optic nerve. Optic nerve
gliomas, Lisch nodules, plexiform lid neurofibromas and CNS tumors are often seen in
association with neurofibromatosis type 1 (NF-1). NF-1 is inherited in an autosomal dominant
manner.

Which of the following does NOT occur in aberrant regeneration of the third nerve?
retraction of the eyelid on attempted adduction
unilateral suppressed vertical optokinetic response
better pupil response to light than to near
pupil constriction on downgaze

Your answer was CORRECT


Explanation
In aberrant regeneration of the third nerve, the pupil does not react or reacts poorly to light,
but constricts on adduction and/or downgaze. Because the pupil responds on adduction, it will
appear to constrict more on convergence than to light (pseudo-Argyll Robertson). These
reactions are thought to be due to innervation of the pupillary sphincter muscle by the medial
or inferior rectus fibres.

Other effects of aberrant regeneration include lid retraction on adduction or downgaze (the
latter causing pseudo-Von Graefe sign).
Argyll-Robertson pupils are due to a defect at the level of the:
short ciliary nerves
ciliary ganglion
lateral geniculate nucleus
long ciliary nerves
dorsal midbrain

Your answer was CORRECT


Explanation
Argyll Robertson pupil is due to a lesion in the dorsal midbrain. Specifically, the pretectal
interneurons to the Edinger Westphal nucleus are involved while sparing ventrally located
accommodative reflex neurons.

Holmes-Adie tonic pupils are caused by a defect at the level of the ciliary ganglion or the
short ciliary nerves.

A miotic pupil in which there is no dilation of the pupil after instillation of cocaine 10%
suggests:
a first-order Horner's
a second-order Horner's
a Horner's of any order
a third-order Horner's

Your answer was CORRECT


Explanation
Cocaine prevents reuptake of norepinephrine into the presynaptic terminal from the synaptic
cleft. With the sympathetics intact, cocaine increases the duration that norepinephrine remains
in the synaptic cleft, causing pupillary dilation. In Horner's syndrome (first, second, and third
order), there is little activity at the cleft because of sympathetic disruption and hence no
dilation.

A child presents with both eyes in the adducted position. To determine if this is the result of a
bilateral lateral rectus palsy, you could try each of the following EXCEPT:
saccadic eye movements generated by an OKN drum
patching one eye and testing ductions
forced duction testing
doll's head movements

Your answer was CORRECT


Explanation
Forced duction helps to identify restriction and has no role in determining the presence of a
paretic deficit. All other tests listed above help to encourage abduction of the eye, to
determine whether the 6th nerve is intact or palsied. Patching one eye will force the other eye
to abduct to see things temporally. Doll's head movements and OKN drum use well-
recognised spinal reflexes to help elicit abduction from an intact sixth nerve.

Which of the following syndromes involves a lesion of CN 6:


Claude's syndrome
Nathnagel's syndrome
Foville's syndrome
Weber's syndrome

Your answer was CORRECT


Explanation
The other options involve cranial nerve 3.

Seizures are MOST likely to be encountered in which of the following conditions:

Wyburn-Mason syndrome

von Hippel-Lindau syndrome

Louis-Bar syndrome

Bourneville syndrome

von Recklinghausen disease

Your answer was CORRECT


Explanation
While seizures may occur in many phakomatoses as a consequence of CNS tumours or
hamartomas (including in von Recklinghausen's and von Hippel-Lindau), they are most
common and characteristic of tuberous sclerosis or Bourneville's syndrome.

Tuberous sclerosis is characterised by:

 cerebral cortical tubers, after which the disease is named


 astrocytic hamartomas of retina and brain (often calcified)
 seizures
 mental retardation
 renal angiomyolipomas
 adenoma sebaceum
 cafe-au-lait spots
 ash leaf depigmentation
 shagreen patches
 periungual fibromas
Which muscle is innervated bilaterally by a single unpaired nucleus?
levator palpebrae superioris
superior rectus
superior oblique
inferior oblique

Your answer was CORRECT


Explanation
The levator muscle is innervated bilaterally by a single, unpaired nucleus.

With regard to the oculomotor muscles, they are all innervated by ipsilateral nuclei except for
the superior rectus and superior oblique, which are innervated by contralateral nuclei.

24-year-old woman complains of visual deterioration over the past 10 days. Her acuities are
6/6 right eye and 6/60 left eye. Ishihara plates are 10/11 right eye, 1/11 left eye. Visual fields
show a centro-coecal scotoma in the left eye, full right eye. Her left optic disc appears mildly
swollen, hyperaemic, with telangiectatic surface vessels. There is no leak on fluorescein
angiography.

All of the following are true of this condition EXCEPT:

diagnostic evaluation must include electrocardiography

she can expect no improvement in the left eye over time

the right eye is likely to be similarly affected within several months

a headache typically accompanies the onset of visual loss

she should be counseled to avoid tobacco and heavy consumption of alcohol

Your answer was CORRECT


Explanation
The features described in this case are typical of Leber's hereditary optic neuropathy. Optic
neuritis is clearly in the differential but the following features work against optic neuritis:

 the presence of disc swelling and hyperaemia (while optic neuritis is typically
retrobulbar)
 no disc leakage on FFA (which would be expected in optic neuritis)

LHON is characterised by:


 mitochondrial inheritance
 typically painless vision loss
 males more commonly affected (80% of cases)
 age of onset usually 10-30 years
 10% of female carriers are affected
 50% of male carriers are affected
 generally sequential bilateral optic nerve involvement
 smoking and alcohol appear to be risk factors for vision loss
 acute: optic disc swelling, hyperaemia, dilated telangiecactic vessels but no leak on FFA
 chronic: optic atrophy
 a small percentage have partial or complete recovery of vision
 type of mutation most important prognostic factor for recovery
 11778 most common mutation (poor prognosis)
 association with ECG cardiac conduction abnormalities, especially preexcitation
syndrome
 association with dystonia and MS-like illness

Which is the correct order for tissue signal intensity in MRI T2-weighted?

CSF > grey matter > white matter > fat

fat > white matter > grey matter > CSF

CSF = grey matter > white matter > fat

fat > grey matter > white matter > CSF

Your answer was CORRECT


Explanation
The order of signal intensity in MRI is as follows:

 T1 weighted: fat > white matter > grey matter > CSF
 T2 weighted: CSF > grey matter > white matter > fat
 FLAIR: fat > grey matter > white matter > CSF
 STIR: CSF = grey matter > white matter > fat

Which condition is most likely to be the cause of a central Horner's syndrome?

nasopharyngeal carcinoma

carotid aneurysm

carotid-cavernous fistula

syringomyelia

Your answer was CORRECT


Explanation
Aetiology of Horner's:
 First Order: (lesions of hypothalamus and brain stem)
o stroke
o tumour (e.g. glioma)
o demyelination
o trauma
o syrinx (syringomyelia and syringobulbia)
 Second Order: (lesions of thoracic outlet, mediastinum, pulmonary apex, neck)
o subclavian artery aneurysm
o cervical rib
o Pancoast tumour
o mediastinal tumours
o thyroid malignancy
o surgery to neck
 Third order (lesions of superior cervical ganglion, ICA, cavernous sinus, sellar regions)
o superior cervical ganglion trauma or invasion
o ICA dissection, aneurysm, thrombosis, invasion by tumour, stenting
o cavernous sinus thrombosis, aneurysm, inflammation, tumour
o invasive pituitary tumours
o cluster headache

Which of the following is most likely to cause facial paralysis with ipsilateral reduced corneal
sensation?
Millard-Gubler syndrome
cavernous sinus meningioma
Gradenigo’s syndrome
cerebello-pontine angle tumour

Your answer was CORRECT


Explanation
Cerebello-pontine angle tumours are characterized by ipsilateral 5th, 6th, 7th and 8th nerve
palsies.

Cavernous sinus lesions cause palsies of the 3rd, 4th, 5th (divisions 1, 2 and 3 depending on
extent) and 6th nerves.

Gradenigo’s syndrome is caused by otitis media, and is characterized by ipsilateral facial pain
(from the trigeminal nerve) and diplopia (from 6th nerve palsy).

Millard-Gubler syndrome involves cranial nerves 6 and 7, with contralateral hemiparesis, due
to a lesion of the 6th nerve fasciculus.

This question came in the 2014 FRCOphth.

Which of the following is most appropriate for the treatment of myasthenia gravis?
pyridostigmine
ecothiopate
hydroxyamphetamine
physostigmine

Your answer was CORRECT


Explanation
Pyridostigmine is an anticholinesterase i.e. it increases availability of ACh at the
neuromuscular junction by preventing its enzymatic breakdown by acetylcholinesterase.
Pyridostigmine has a half-life of 6 hours, making it very useful as an oral treatment in
myasthenia gravis.

Physostigmine is also an anticholinesterase, but its half-life is very short, making it less useful
in myasthenia.

Ecothiopate is a long-acting anticholinesterase, which binds irriversably to the enzyme. As a


result, it can cause life-threatening prolonged paralysis if used systemically; although it has
found use as a topical treatment for chronic glaucoma.

Pheochromocytoma may be seen as part of:


ataxia-telangiectasia
tuberous sclerosis
Wyburn-Mason syndrome
neurofibromatosis

Your answer was CORRECT


Explanation
Pheochromocytomas produce, secrete, and store catecholamines. They are most often derived
from the adrenal medulla but may arise in any of the sympathetic ganglia.

Two phakomatoses, neurofibromatosis and von Hippel-Lindau disease, are associated with
pheochromocytomas.

A 13-year-old has progressive difficulty walking over a period of 4 months. On examination,


there is lower limb weakness and wasting. The reflexes are absent. Plantar responses are
upgoing. Vibration and joint-position sense are absent. There is nystagmus and pigmentary
retinopathy.

What is the most likely diagnosis?


Louis-Bar syndrome
Deuchenne muscular dystrophy
multiple sclerosis
myotonic dystrophy
Friedrich's ataxia

Your answer was CORRECT


Explanation
The findings are most in keeping with Friedrich's ataxia. This is an autosomal recessive
progressive degeneration of dorsal root ganglia, spinocerebellar tracts, corticospinal tracts, and
cerebellar Purkinje cells. Onset is in teenage years, and survival seldom exceeds 20 years from
diagnosis. Patients may have pigmentary retinopathy and optic atrophy. Eye signs may also
occur secondary to cerebellar dysfunction such as nystagmus, opsoclonus, ocular flutter, and
skew deviation.

A 48-year-old lady presents with a 3-month history of irritation and conjunctival injection of
the right eye. On examination, there is right orbicularis weakness, decreased ability to wrinkle
the right forehead, right corneal anaesthesia, and a small angle esodeviation develops on right
gaze.

What is the most appropriate investigation?


CT to exclude aneurysm
thyroid function tests
tensilon test
MRI to evaluate right cerebellopontine angle

Your answer was CORRECT


Explanation
This patient has signs of unilateral fifth (corneal anaesthesia), sixth (abducens weakness) and
seventh nerve (facial movement) palsy. These three cranial nerves are in close proximity in
the cerebellopontine angle, which must be imaged.

Which of the following ocular motor disorders is MOST associated with malignancy?
ocular bobbing
square-wave jerks
dysmetria
opsoclonus

Your answer was CORRECT


Explanation
Opsoclonus consists of nonstop, random, bilateral, conjugate, directionally unpredictable
saccades.
Ocular flutter consists of spontaneous groups of back-and-forth horizontal saccades that may
occur during fixation or at the end of a normal horizontal saccade. Both ocular flutter and
opsoclonus may be associated with malignancy, such as metastatic neuroblastoma in children
or small cell cancer of the lung in adults. These eye movements may be the first presenting
sign of cancer. Multiple sclerosis also can cause ocular flutter.
Square-wave jerks are a microsaccadic fixation disturbance associated with cerebellar
diseases of various kinds.
Ocular motor dysmetria is a back-and-forth saccadic motion about the point of fixation that
occurs following an otherwise normal saccade. It is felt to represent overshooting of the
intended fixation point.
Ocular bobbing refers to abrupt, spontaneous downward jerks of the eyes with a slow return
to the midposition in association with paralysis of spontaneous and reflex horizontal eye
movements. It is most commonly seen in comatose or quadriplegic patients with large infarcts
or brainstem hemorrhages, especially of the pons.

A patient with a history of bilateral occipital lobe infarcts adamantly states that he can see
quite well and confabulates visual images. He most likely has:
blindsight
Charles Bonnet's syndrome
palinopsia
Anton's syndrome

Your answer was CORRECT


Explanation
Anton's syndrome is the denial of blindness in the face of irrefutable evidence to the contrary
and is a recognised phenomenon that can occur with bilateral cortical blindness.

Blindsight is the phenomenon whereby some people who are cortically blind can appear to
appreciate their environment at a subconscious level, for example, they can point to objects
with a higher frequency than by chance.

Charles Bonnet syndrome is a phenomenon whereby fully formed, vivid, life-like


hallucinations occur in individuals who have lost sight from a biological cause (often age-
related macular degeneration). It is considered a release phenomenon due to the absence of
stimuli reaching the visual cortex.

Palinopsia is the persistence of an image after it has passed and may be caused by drugs (e.g.
alcohol, cocaine) or by a lesion in the non-dominant parieto-occipital lobe.

In a 59-year-old man with bilateral optic atrophy and coecocentral scotomas but an otherwise
normal examination, investigation should include all EXCEPT:

Mantoux

serum B12 level

fluorescent treponemal antibody-absorptive test

serum folate level

serum prolactin

Your answer was CORRECT


Explanation
The most common aetiologies of bilateral central or cecocentral scotomas include:

 hereditary optic neuropathy


 nutritional optic neuropathy (vitamin B12 and folate deficiency)
 drug toxicity
 tobacco-alcohol amblyopia
 infiltrative disorders including syphilis and tuberculosis

Prolactin levels are assessed in proven or suspected chiasmal disorders, which are
characterised by bitemporal field defects or junctional field defects (not bilateral centrocoecal
defects).

Which statement is FALSE about optic nerve glioma?


the age range with the highest incidence is 2 to 6 years

the classic CT appearance is the rail-road sign

a syndrome strikingly similar to spasmus nutans may be seen in gliomas involving the
hypothalamus or optic-chiasm

observation may be appropriate

Your answer was CORRECT


Explanation

The classic CT appearance of ON glioma is a fusiform enlargement of the optic nerve,


although the mass may be exophytic in appearance and the nerve may be elongated with
kinking or buckling (as shown in the CT above).

Note: the railroad track sign refers to the classic CT appearance of meningioma.

Chiasmal glioma needs to be excluded in a young child presenting with spasmus nutans.
Characteristics of optic nerve gliomas:
 peak incidence in childhood (age 2-6 years)
 association with NF-1 (30%)
 usually benign (adults higher risk of malignancy)
 presentation insidious
 acute haemorrhage is rare
 proptosis can be non-axial (inferior or temporal dystopia)
 other signs: reduced vision, optociliary shunts, CRVO, choroidal folds
 classic CT appearance: fusiform enlargement
 Treatment: confined to orbit = observation; posterior = radiation
 Excision not possible without losing sight (excision of optic nerve!)

The abducens nerve:


contains sympathetic nerve fibres
is the most slender cranial nerve
lies medial to the internal carotid artery in the cavernous sinus
enters the orbit within the tendinous ring

Your answer was CORRECT


Explanation
The abducens nucleus is located in the floor of the fourth ventricle. It is a pure motor nerve
and lies lateral to the internal carotid artery in the cavernous sinus. The trochlear nerve is the
most slender of all cranial nerves.

Which histopathologic variety of meningioma is most commonly seen within the orbit?
meningothelial
transitional
angioblastic
fibroblastic
pilocytic

Your answer was CORRECT


Explanation
Meningothelial meningioma is the most common histopathologic type of meningioma seen
within the orbit.

The best procedure to dampen nystagmus in a patient with nystagmus and a head turn is:
Faden suture
Knapp
Kestenbaum
Harado Ito

Your answer was CORRECT


Explanation
The Kestenbaum procedure is a bilateral resection/recession that surgically moves the eyes
toward the direction of the head turn to dampen the nystagmus.

A middle-aged patient complains of intermittent vertical diplopia and asymmetrical ptosis.


Her symptoms improve after a short nap.

What is the most likely diagnosis?


myasthenia gravis
myotonic dystrophy
Kearns-Sayre syndrome
thyroid ophthalmopathy
internuclear ophthalmoplegia

Your answer was CORRECT


Explanation
Variability of symptoms with improvement after sleep is suggestive of myasthenia gravis.

Which one of the following statements concerning Botulinum toxin (Botox) injection into the
extra-ocular muscles is FALSE?
systemic side-effects are common but are typically transient
Botox interferes with cholinergic receptors preventing release of acetylcholine
the effect of Botox lasts clinically for 3 months
ocular side-effects include ptosis, diplopia, and, rarely, globe perforation

Your answer was CORRECT


Explanation
Botox is a purified form of botulinum toxin type A, derived from Clostridium botulinum. It
blocks neuromuscular conduction by binding to receptor sites on motor nerve terminals
interfering with the release of ACh into the synaptic cleft. When injected intramuscularly,
Botox produces a localized chemical denervation muscle paralysis. The nerve ending
atrophies but will resprout over time. Paralysis onset occurs in 2 days, increases in intensity
over the next week, and lasts 3 months in extraocular muscles.

Botox is indicated for blepharospasm associated with dystonia. The efficacy of Botox in
strabismus is low and surgery remains the primary treatment for most types of strabismus.
Multiple injections may be necessary but should not exceed 200 units in 1 month to decrease
the incidence of antibody production.

Reported side effects include ptosis, diplopia, and spatial disorientation. These are,
fortunately, temporary. Perforation of the globe has been reported. Systemic effects of Botox
are not seen because a dose over 100 times greater than the normal amount is required for
toxicity.

Which one of the following is NOT a criterion for the diagnosis of pseudotumour cerebri
(idiopathic intracranial hypertension)?
normal cerebrospinal fluid composition
normal neuroimaging studies
bilateral papilloedema
elevated opening pressure on lumbar puncture

Your answer was CORRECT


Explanation
Pseudotumor cerebri is a diagnosis of exclusion. Obstructive, compressive, and infiltrative
CNS lesions must be excluded. Papilloedema, although commonly present, is a result of the
increased intracranial pressure and is not necessary for the diagnosis. The optic nerve swelling
may be unilateral or asymmetric.

See-saw nystagmus is MOST likely to be associated with:


pinealoma
Arnold-Chiari malformation
nasopharyngeal carcinoma
craniopharyngioma

Your answer was CORRECT


Explanation
See-saw nystagmus is characteristically a feature of chiasmal lesions

A 69-year-old Type 2 diabetic was referred for cataract surgery. He has small asymmetrical
pupils, with poor pupillary light reactions, although both pupils constrict briskly to near
stimuli. Dilatation with tropicamide was poor.

What is the most likely diagnosis?


Hutchison pupil
Argyll-Roberston pupil
Marcus Gunn pupil
central Horner's syndrome
Adie tonic pupil

Your answer was CORRECT


Explanation
Argyll-Robertson pupils are usually bilateral but may be asymmetrical. They are caused by a
lesion in the dorsal midbrain affecting the light-reflex fibres but sparing the near-response
fibres. While traditionally associated with syphilis, they can also be caused by diabetes, MS,
trauma, surgery, alcohol and aberrant 3rd nerve regeneration. Typically Argyll-Roberston
pupils do not dilate well to mydriatics such as tropicamide or cocaine 4-10%.

The other main differential in this case is an Adie's tonic pupil, which can also cause light-
near dissociation. While usually enlarged, an Adie's pupil (especially if longstanding) can be
small.  However, the hallmark feature of Adie's is a slow, gradual constriction on near
response, with a similarly slow re-dilatation on relaxing accommodation, which was not
described in this case. Adie’s pupils are also usually unilateral (not bilateral) and there is
super-sensitivity to dilute pilocarpine 0.1%.

Which fundus finding would be MOST suggestive of Leber's hereditary optic neuropathy?
optic nerve drusen
optic disc hypoplasia
sectoral optic atrophy
hyperaemic optic nerve with telangiectatic capillaries
haemorrhagic papilloedema

Your answer was CORRECT


Explanation
Leber's hereditary optic neuropathy (LHON) is caused by an abnormality in mitochondrial
DNA. Since mitochondrial DNA is only transmitted along the maternal line, men cannot pass
this disease to their offspring.

In the acute phase, the optic nerve in LHON is hyperaemic and swollen with telangiectatic
capillaries. The nerve does not leak on fluorescein angiography. Later stages may only
manifest optic atrophy.

What is the mechanism of action for cocaine?


inhibits Catechol-O-Methyl Transferase (COMT)
releases noradrenaline from the pre-synaptic terminal
prevents reuptake of noradrenaline
agonist of the noradrenaline receptor

Your answer was CORRECT


Explanation
Cocaine blocks the reuptake of noradrenaline from the synaptic cleft, prolonging the effect of
released neurotransmitter. It is useful to confirm Horner's syndrome. In Horner's there is little
noradrenaline present in the synapse in the first place because of a defect in the sympathetic
chain so there is no effect from cocaine instillation (no dilatation).

Leber's hereditary optic neuropathy is an example of:


mitochondrial inheritance
X-Linked recessive inheritance
autosomal dominance inheritance
autosomal recessive inheritance

Your answer was CORRECT


Explanation
Leber's hereditary optic neuropathy is a mitochondrially inherited condition.

In which of the following conditions is an endocrine evaluation together with neuroimaging


studies MOST strongly indicated?

morning glory

optic disc coloboma

optic nerve hypoplasia

persistent myelination of the nerve fibre at the disc

optic nerve pit

Your answer was CORRECT


Explanation
Optic nerve hypoplasia may be associated with De Morsier's syndrome (septo-optic
dysplasia), in which there is agenesis of the corpus callosum and absence of the septum
pellucidum with optic nerve hypoplasia, and often pituitary deficiency with growth hormone
being the most common.

Optic nerve hypoplasia can be associated with:

 septo-optic dysplasia
 aniridia
 fetal alcohol syndrome
 maternal: LSD, quinine, and phenytoin
Morning glory disc anomaly and optic disc coloboma are both typically isolated anomalies.
However, they can both be associated on rare occasions with other conditions: morning glory
with transphenoidal encephalocele and hypopituitarism while optic disc coloboma can be
associated with basal encephalocoele and Dandy-Walker anomaly.

However, the chances of systemic association with morning glory anomaly and optic disc
coloboma are much lower than the chances with optic nerve hypoplasia, where coexistent
CNS abnormalities occur in up to 75% of patients. The correct answer is therefore optic nerve
hypoplasia. Reference

Which of the following does NOT help to distinguish optic neuropathy from amblyopia?
kinetic perimetry

color vision testing

visual acuity on Snellen

neutral density filters

swinging flashlight test

Your answer was CORRECT


Explanation
Optic neuropathy can be distinguished from amblyopia by:

 brightness sense: diminished in ON, not amblyopia


 colour vision: diminished in ON, not amblyopia
 neutral density filters: reduce vision in ON, increase vision in amblyopia
 visual fields: affected in ON, full in amblyopia
Visual acuity tested by Snellen is reduced in both amblyopia and optic neuropathy.
A patient on ventilation in the ICU has a dilated, unresponsive right pupil to light. There is no
RAPD. Bedside, portable slit-lamp examination is normal.

Which of the following syndromes involves a lesion of CN 6:

phenylephrine 2.5%

pilocarpine 0.1%

cocaine 10%

pilocarpine 1%

hydroxyamphetamine 1%

Your answer was CORRECT


Explanation
The differential of a dilated pupil includes:

 traumatic mydriasis
 Adie's pupil
 third nerve palsy
 pharmacologic dilation
Iris sphincter tears or a history of blunt trauma would be present with traumatic mydriasis and
can be ruled out in this case straight away.

The sequence to differentiate the remaining three causes should be as follows:


 0.1% pilocarpine is instilled. If the pupil constricts this represents denervation
hypersensitivity due to Adie's pupil. If there is no constriction, proceed to 1% pilocarpine.
 1% pilocarpine is instilled. If there is NO constriction, this suggests pharmacological
mydriasis, if it does constrict, this suggests third nerve.
The saccadic movement that is MOST often affected in progressive supranuclear palsy is:
upward
leftward
rightward
downward

Your answer was CORRECT


Explanation
In progressive supranuclear palsy, downward gaze is generally affected, becoming smaller and
slower. Saccades are affected more than pursuits. This is in distinction to the dorsal midbrain
syndrome, in which upward saccades are generally affected.

All are characteristics of congenital third nerve palsy EXCEPT:


ptosis
esodeviation
hypotropia
abnormal pupillary function

Your answer was CORRECT


Explanation
In a third nerve palsy the eye is usually deviated down and out. Aberrant reinnervation may
manifest as abnormal pupillary constriction with adduction.

All of the following features suggest a surgical third nerve palsy rather than a medical third
nerve palsy EXCEPT:

abberant regeneration

no recovery

pupil involvement

complete third nerve palsy

multiple cranial nerve palsies

Your answer was CORRECT


Explanation
Surgical third nerve palsies (space occupying lesions, especially aneurysm of the PCA) are
often incomplete initially, with progression of the palsy over time. In fact, medical third nerve
palsies are more likely to be complete at presentation.
Factors suggesting a surgical third nerve:

 young patient
 no cardiovascular risks
 pupil involvement (90%, versus only 20% in medical cases)
 progression of pupil involvement
 incomplete palsy with progression
 multiple cranial nerve palsies
 aberrant regeneration
 no recovery

Note: the presence of pain is not particularly useful to discriminate medical from surgical
causes of a third nerve because medical causes such as diabetes, migraine and Tolosa Hunt
may be associated with pain.

Which is FALSE regarding eye movements:


the frontal lobe controls ipsilateral saccades
a para-pontine reticular formation lesion causes ipsilateral horizontal gaze palsy
an MLF lesion causes ipsilateral failure of adduction with contralateral abducting nystagmus
left pursuit is controlled by the left parietal lobe

Your answer was CORRECT


Explanation
The frontal lobes control contralateral saccades. Other statements are true.

The spinothalamic tracts transmit:


pain and temperature
vibration, touch and proprioception
volitional motor pathways
autonomic motor pathways

Your answer was CORRECT


Explanation
The spinothalamic tracts of the spinal column transmit pain and temperature. First order
neurones synapse at the spinal level, second order neurones cross the midline at the spinal
level before ascending to synapse in the thalamus.

A 15-year-old patient has bilateral severe optic atrophy, diabetes insipidus and progressive
neuro-degeneration.

What is the most likely diagnosis?


Wolfram syndrome
Kjer syndrome
Behr syndrome
Leber's congenital amaurosis

Your answer was CORRECT


Explanation
Wolfram syndrome is also known as DIDMOAD = diabetes insipidus, diabetes mellitus, optic
atrophy and deafness. Inheritance is autosomal recessive, presentation is between 5-21 years
and visual prognosis is poor with severe optic atrophy. Systemic abnormalities include
anosmia, ataxia, seizures, mental handicap, short stature, and endocrine abnormalities.

The pterygopalatine ganglion:


is a sympathetic ganglion
contains preganglionic fibers that arise in the lacrimal nucleus of the facial nerve
receives preganglionic parasympathetic fibers from the lesser petrosal nerve
contains secretomotor nerve to the parotid gland

Your answer was CORRECT


Explanation
The pterygopalatine ganglion is a parasympathetic ganglion. Its preganglionic fibers arise
from the lacrimal nucleus of the facial nerve and they run in the greater petrosal nerve before
synapsing in the ganglion. The ganglion contains secretomotor nerve fibres to the lacrimal
gland.

Monocular nystagmus in a toddler raises the possibility of:

craniopharyngioma

optic nerve meningioma

metastatic neuroblastoma

chiasmal glioma

rhabdomyosarcoma

Your answer was CORRECT


Explanation
Monocular nystagmus has been associated with chiasmal and hypothalamic gliomas. It is also
seen in blind eyes, multiple sclerosis, and spasmus nutans, which is a transient, fast-beating,
but low-amplitude nystagmus found during the first 5 years of life.

Spasmus nutans is characterised by:


 onset 6 months to 3 years of age
 idiopathic and self-limiting (usually within 2 years)
 rapid, small-amplitude disconjugate jerk nystagmus
 head nodding
 torticollis
 nystagmus may appear entirely monocular and raise the specter of chiasmal or
hypothalamic glioma, which needs always to be excluded

Which cranial nerve is most likely to be damaged by raised intra-cranial pressure:


three
six
four
five

Your answer was CORRECT


Explanation
The sixth cranial nerve is the most likely to be injured by raised intracranial pressure, which is
called the false localising sign. The fourth cranial nerve is most likely to be injured by closed
head trauma.

A 26-year-old woman presents to ophthalmology casualty with anisocoria, first noticed 3 days
ago. On examination, you find a dilated, unreactive left pupil. The right pupil responds
normally to direct and consensual responses. Pharmacological testing of the dilated pupil is as
follows:

 Cocaine 4%: no change


 Hydroxyamphetamine 1%: no change
 Adrenaline 1 in 1000: no change
 Pilocarpine 1%: no change

What is the most likely diagnosis?


pharmacological mydriasis

third nerve palsy

post-ganglionic Horner’s syndrome

Holmes-Adie pupil

Your answer was CORRECT


Explanation
Pharmacological dilatation is the most likely cause. A Horner’s pupil is miosed not dilated,
and should respond by dilatation to either 1:1000 adrenaline (if post-ganglionic, due to
denervation hypersensitivity) or 1% hydroxyamphetamine (if pre-ganglionic, since the intact
post-ganglionic nerve will be stimulated to release noradrenaline).
A dilated pupil due to third nerve palsy should respond to 1% pilocarpine which is a direct
agonist. A Holmes-Adie pupil should also respond to 1% pilocarpine - in fact, it even
responds to 0.1% pilocarpine due to denervation hypersensitivity.

This question came in the 2014 FRCOphth.

Rapid, chaotic, eye movements in all directions are found in which disorder:
ocular flutter
ocular motor apraxia
ocular myoclonus
opsoclonus
oscillopsia

Your answer was CORRECT


Explanation
Opsoclonus refers to rapid, chaotic, involuntary, multivectoral, conjugate, fast eye movements
without inter-saccadic intervals. It can be associated with neuroblastoma, encephalitis,
paraneoplastic syndromes, MS, toxins, or medications (e.g. serotonin syndrome).

Oscillopsia is a visual disturbance in which objects in the visual field appear to oscillate.

Ocular myoclonus refers to bursts of pendular eye movements normally associated with
lesions in the midbrain.

Ocular flutter refers to an involuntary, rapid, horizontal saccadic oscillation of both eyes while
attempting to fixate an object. It is a sign of cerebellar disease.

Ocular motor apraxia is the absence or defect of controlled, voluntary, and purposeful eye
movement. People with this condition have difficulty moving their eyes horizontally and
moving them quickly. The main difficulty is in saccade initiation. It may be idiopathic or
associated with abnormalities of corpus callosum, cerebellum or fourth ventricle.

All of the following are typical of a dominant parietal lobe lesion EXCEPT:

finger agnosia

dysgraphia

dyscalcula

hemispatial neglect

left-right disorientation

Your answer was CORRECT


Explanation
Hemispatial neglect is a feature of non-dominant parietal lobe pathology, as is dressing
apraxia.

Dominant parietal lobe lesions cause:

 dyscalcula
 dysgraphia
 left-right disorientation
 finger agnosia
 alexia
 speech disturbance

Note the first four features above form Gerstmann syndrome of the dominant parietal lobe

A patient has normal vertical eye movements. However, in laevoversion, the left eye develops
jerk nystagmus while the right eye fails to adduct. Dextroversion is normal.

Where is the most likely location of the causative lesion?


Right third nerve nucleus
Left paramedian pontine reticular formation
Left sixth nerve
Right medial longitudinal fasciculus
Left medial longitudinal fasciculus

Your answer was CORRECT


Explanation
This patient has a right internuclear ophthalmoplegia (INO), affecting the right medial
longitudinal fasciculus.

Note that poor adduction could also occur from third nerve dysfunction, but this patient has no
ptosis, mydriasis, or involvement of the superior rectus, inferior rectus, or inferior oblique
muscles. Damage to the left paramedian pontine reticular formation results in a left gaze palsy.

A 52-year-old lady complains of binocular diplopia for 3 months. On examination, there is


limited elevation, depression, abduction and adduction of the left eye. The left pupil is larger
than the right with ptosis. There is no ocular pain, injection or proptosis.

What is the most likely diagnosis?


Tolosa-Hunt syndrome
nasopharyngeal carcinoma
aneurysm of the intracavernous carotid artery
high-flow carotid-cavernous fistula
acoustic neuroma

Your answer was CORRECT


Explanation

Figure: MRI of right cavernous ICA aneurysm

In the question above, there is a unilateral third, possibly fourth, and sixth nerve palsies. This
constellation of signs suggests involvement of the cavernous sinus or orbital apex.
Involvement of the orbital apex would likely produce proptosis, making cavernous sinus the
most likely. A carotid-cavernous fistula should produce conjunctiva hyperaemia so the most
likely diagnosis is an aneurysm of the intracavernous carotid artery.

A young patient presents with vertical nystagmus, induced by rotating an OKN drum
downwards. There is globe retraction associated with the nystagmus.

What is the MOST likely diagnosis?

spasmus mutans

dorsal midbrain syndrome

Lithium overdose

periodic alternating nystagmus

Maddox's nystagmus

Your answer was CORRECT


Explanation
The description fits with convergence-retraction nystagmus which is a feature of Parinaud's
dorsal midbrain syndrome.

Features of Parinaud's include:


 light-near dissociation
 mydriasis
 Collier’s sign (lid retraction in primary position)
 paralysis of convergence
 paralysis of upgaze (supranuclear palsy)
 convergence-retraction nystagmus

Characteristics considered classic for spasmus nutans include all EXCEPT:


a rapid, small-amplitude, conjugate nystagmus
usually self-limiting
torticollis
head nodding

Your answer was CORRECT


Explanation
Spasmus nutans is characterised by:
 onset 6 months to 3 years of age
 idiopathic and self-limiting (usually within 2 years)
 rapid, small-amplitude disconjugate jerk nystagmus
 head nodding
 torticollis
 nystagmus may appear entirely monocular and raise the specter of chiasmal or hypothalamic
glioma, which needs always to be excluded

A 24-year-old man has recently suffered a closed head injury in a road traffic accident and
complains of vertical diplopia since then. Your orthoptist reports left hypotropia with right inferior
oblique over-action.

What is the most likely diagnosis?

left fourth nerve palsy

right fourth nerve palsy

left sixth nerve palsy

bilateral fourth nerve palsy

right sixth nerve palsy

Your answer was CORRECT


Explanation
Left hypotropia suggests right hypertropia, with right inferior oblique overaction, which is the first
muscle sequelae of an ipsilateral fourth nerve palsy. The answer is therefore right fourth nerve
palsy.

Bilateral fourth nerve palsies are common after closed head injury but the chief complaint is
usually torsional (not vertical) diplopia, with excyclotorsion greater than 10 degrees on Maddox
rod testing and alternating hypertropia on lateral gaze.

Astrocytic hamartomas are MOST likely to feature in which phakomatosis:


von Hippel-Lindau syndrome
Sturge-Weber syndrome
Louis-Bar syndrome
von Recklinghausen disease

Your answer was CORRECT


Explanation
Retinal astrocytic hamartomas may be encountered in both von Ricklinghausen disease and Bourneville's
syndrome (tuberous sclerosis).

NF-1 is characterised by:


 AD inheritence
 cafe-au-lait patches
 axial freckles (pathognomonic)
 short stature
 macrocephaly
 facial hemi-atrophy
 Lisch nodules
 ON gliomas
 meningiomas
 plexiform neurofibromas
 absence of greater wing of sphenoid
 prominent corneal nerves
 choroidal hamartomas
 retinal tumours
o astrocytic hamartomas
o combined hamartomas of RPE and retina

Which statement is FALSE regarding pituitary lesions:


growth hormone tumours can be successfully treated with somatostatin
acidophil adenomas cause gigantism in children
see-saw nystagmus, hemi-field slip and post-fixation blindness are recognised features
secreting tumours are more likely to present with vision loss than non-secreting tumours
prolactinomas can be successfully treated with bromocriptine

Your answer was CORRECT


Explanation
Non-secreting tumors often present with visual field loss, whereas secreting tumors present with endocrine
dysfunction before they reach the size necessary to compress the chiasm to cause visual field defects.

Secreting microadenomas (75% of pituitary tumours) can be classified by their staining:


 chromophobes (50%): secrete prolactin causing infertility, impotence, amenorrhea,
galactorrhea
 acidophil (20%): secrete GH causing acromegaly in adults or gigantism in children
 basophils (5%): secrete ACTH causing Cushing's disease or FSH and TH (rare)

Which is TRUE regarding temporal arteritis?


CRP has a higher sensitivity than ESR
temporal artery biopsy is unhelpful after 1 week of starting steroids
the platelet count remains abnormal on steroid treatment
it is not possible to make a diagnosis on clinical signs alone

Your answer was CORRECT


Explanation
The diagnosis of temporal arteritis in practice relies on a combination of clinical signs, laboratory tests
(inflammatory markers: FBC, ESR, CRP) and histopathological findings (temporal artery biopsy). However,
strictly speaking it is possible to diagnose GCA on clinical signs alone in keeping with the American College
of Rheumatology GCA 1990 classification. A patient must have any 3 of the following 5 criteria for a
diagnosis:
 age over 50
 new onset headache
 temporal artery abnormality (reduced pulses, tenderness)
 ESR over 50 mm
 positive temporal artery biopsy

Admittedly these criteria are becoming dated but as yet they have not been superceded by more recent
criteria. It is true to say it is not recommended to make a diagnosis of GCA on clinical signs alone but it is
also true to say a diagnosis can be made on clinical signs alone.

Note: The CRP has a higher sensitivity than ESR for temporal arteritis; when used together the sensitivity
increases to 97-99%.

Histopathological findings persist for at least 2-6 weeks after commencing steroids. Steroids will normalize
blood markers if the disease is successfully treated, including ESR, CRP and platelets

Which one of the following statements about oculomotor apraxia is FALSE?

pursuits are generally affected more than saccades


in the congenital form, compensatory exaggerated head turns to refixate are common

in the acquired form, blinks are frequently used to break fixation

horizontal movements are typically affected more than vertical movements

Your answer was CORRECT


Explanation
Oculomotor apraxia is an inability to initiate voluntary horizontal saccades. Congenital
oculomotor apraxia is characterized by striking compensatory head movements. Acquired lesions
(e.g. resulting from bilateral frontal lobe strokes) usually produce defects in initiation of
bidirectional saccades; patients may blink to break fixation and then turn their head to fixate on
something else. Pursuits remain relatively unaffected.

All of the following are true regarding disorders of higher visual function EXCEPT:

lightning streaks of Moore's is a physiological entoptic phenomenon

temporal lobe seizures cause formed visual hallucinations

Charles-Bonnet syndrome usually comprises colourful, well-defined, formed images

palinopsia refers to persistence of images after their removal

unilateral retrogeniculate lesions can lead to cortical blindness

Your answer was CORRECT


Explanation
Cortical blindness is decreased vision secondary to bilateral retrogeniculate lesions. Unilateral
retrogeniculate lesions do not lead to cortical blindness. Other options above are true.

A 5-year-old boy is evaluated for squint. Ophthalmic examination reveals esotropia in the primary
position with markedly limited abduction of the left eye. There is minimal restriction of adduction
of the left eye and lid fissure narrowing on attempted adduction. Right ocular motility and fissure
height are normal.

This case MOST likely represents which one of the following syndromes?
Brown's syndrome
Duane's retraction syndrome type I
Duane's retraction syndrome type III
Duane's retraction syndrome type II
Mobius syndrome

Your answer was CORRECT


Explanation
Duane's retraction syndrome (DRS):
 unilateral or bilateral abnormality of horizontal gaze
 caused by co-contraction of medial and lateral recti
 retraction of the globe on attempted adduction
 upshooting or downshooting of the globe on adduction
 the left eye is affected more frequently than the right
 females are affected more frequently than males
 Type 1: abduction limitation greater than adduction, eso (most common)
 Type 2: adduction limitation greater than abduction, exo (least common)
 Type 3: abduction and adduction limited equally
 amblyopia in 10%
Associations with Duane's retraction syndrome:
 cataracts
 iris anomalies
 Marcus Gunn jaw winking
 microphthalmos
 crocodile tears
 Goldenhar's syndrome
 maternal thalidomide
 Klippel-Feil syndrome

An infant with developmental delay is noted to have pale, hypoplastic optic discs. A CT brain scan
shows absence of the septum pellucidum.

What is the MOST likely diagnosis?


Apert syndrome
Crouzon syndrome
Pfeiffer syndrome
de Morsier syndrome
Hallermann Streiff syndrome

Your answer was CORRECT


Explanation
The features suggest de Morsier's syndrome, which is characterised by:
 optic nerve hyoplasia
 pituitary gland dysfunction
 absence of the septum pellucidum
A patient with the following visual fields is MOST likely to have a lesion in the:

temporal lobe

parietal lobe

frontal lobe

occipital lobe

Your answer was CORRECT


Explanation
The optic radiations split into an upper portion passing through the parietal lobe with fibres
representing the lower visual field (a lesion here will therefore cause a contralateral lower
quadrantanopia) and a lower portion passing through the temporal lobe carrying information from
the upper visual field (a lesion here causes a contralateral upper quadrantanopia).

This question came in the 2014 FRCOphth.

The facial nerve is responsible for all of the following EXCEPT:

motor innervation to the muscles of facial expression

parasympathetic innervation to the parotid gland

taste sensation to the anterior two-thirds of the tongue

motor innervation to the stapedius


Your answer was CORRECT
Explanation
The facial nerve is responsible for the parasympathetic supply to the pterygopalatine ganglion
which supplies the lacrimal gland (not the parotid), as well as the choroid and glands in and
around the nose and mouth. The facial nerve also supplies motor innervation to the muscles of
facial expression, the stapedius, the stylohyoid and the digastric. Finally, the facial nerve is
responsible for taste sensation to the anterior two-thirds of the tongue.

A 58-year-old man develops headache and diplopia. An MRI shows abnormal signal in the area of
the dorsal midbrain.

Which is LEAST likely on examination:


pupils that react better to a near stimulus than to light
skew deviation
lid retraction
sixth nerve palsy

Your answer was CORRECT


Explanation
The sixth nerve arises in the pons and should not be affected by this lesion.

Features of Parinaud's dorsal midbrain syndrome:


 light near dissociation
 defective accommodation and convergence
 upgaze palsy (supranuclear)
 convergence retraction nystagmus
 Collier's sign (lid retraction on upgaze)
 Skew deviation

Causes of Parinaud's:
 stroke
 demyelination
 pineal tumour
 VP shunt malfunction

A patient has a homonymous visual field defect respecting the midline and complains of
experiencing unformed visual hallucinations.

These features are MOST in keeping with a lesion of the:

temporal lobe

parietal lobe
occipital lobe

frontal lobe

Your answer was CORRECT


Explanation
An occipital lobe lesion causes a congruous, contralateral homonymous hemianopia. It may also
give rise to unformed visual hallucinations, visual inattention, visual agnosia, cortical blindness (if
bilateral), Anton's syndrome (cortical blindness with denial of blinding), Riddoch's phenomenon
(can see moving target but not stationary target), Balint's syndrome (ocular apraxia with visual
inattention).

Lesions of the temporal lobe can cause a contralateral upper quadrantanopia with formed visual
hallucinations.

A 28-year-old man presents complaining of sudden loss of vision in his right eye. His acuities are
6/60 in the right eye and 6/6 in the left eye. Examination reveals a swollen right optic nerve,
dilated right retinal veins, and scattered dense retinal haemorrhages in the right fundus.

The factor that most convincingly argues against the diagnosis of papillophlebitis is the patient's:

age

visual acuity

absence of cotton-wool spots

sex

retinal hemorrhages

Your answer was INCORRECT


Explanation
Patients with papillophlebitis have normal or near normal visual acuity. It may be a form of
incomplete central retinal vein occlusion (CRVO) occurring in young patients and usually resolves
spontaneously within 12 months.

A lady in her 70's develops sudden-onset vomiting, dizziness, and double vision. On examination,
she has a concomitant left hypertropia and ataxia.

What test should be ordered?

Tensilon test

carotid ultrasonography

cerebral arteriogram
brain MRI and MRA

Your answer was CORRECT


Explanation
This lady has symptoms (vomiting, imbalance) and signs (skew deviation, ataxia) of vertebral-
basilar insufficiency. Cerebral MRI and MRA would be the best tests because the brainstem,
cerebellum, and arteries (vertebral, basilar) could be evaluated.

Which of the following structures is not associated with vertical gaze:

riMLF

fourth cranial nerve

PPRF

third cranial nerve

Your answer was CORRECT


Explanation
The paramedian pontine reticular formation is the horizontal gaze center and has no role in
vertical gaze.

The following arteries form the circle of Willis EXCEPT:


the posterior cerebral arteries
the anterior cerebral arteries
the posterior communicating arteries
the middle cerebral arteries

Your answer was CORRECT


Explanation
The following arteries make up the circle of Willis:
 Anterior cerebral arteries
 Anterior communicating artery
 Internal carotid arteries
 Posterior cerebral arteries
 Posterior communicating arteries
The basilar artery and middle cerebral arteries, though they supply the brain, are not considered part of the
circle.
Branches of the pterygopalatine ganglion enter the orbit through:

the foramen ovale

the superior orbital fissure

the optic canal

the inferior orbital fissure

Your answer was CORRECT


Explanation
Branches of the pterygopalatine ganglion enter the orbit through the inferior orbital fissure.

Raised intracranial pressure is associated with:


a widened pulse pressure
a regular, reduced respiratory rate
an increase in pulse rate
a decrease in blood pressure

Your answer was CORRECT


Explanation
Raised intracranial pressure is associated with:
 raised blood pressure
 bradycardia
 widened pulse pressure
 periodic Cheyne-Stokes respiration
In patients with raised ICP, therapeutic hyperventilation can be used to decrease the PCO2 which in turn
reduces the intracranial pressure by decreasing the cerebral blood flow to the arachnoid granulations.
However, the ensuing reduction in cerebral blood flow may exacerbate cerebral ischemia

Which is TRUE of skew deviation?


usually involves inferior oblique underaction
usually comitant
usually involves superior oblique underaction
horizontal component predominates

Your answer was CORRECT


Explanation
Skew deviation is characterised by:
 motility disturbance with a vertical component
 does not have a pattern consistent with a discrete muscle underaction/ overaction
 typically (but not always) comitant
 generally due to supranuclear or vestibuloocular dysfunction
 generally reflects brainstem disease

A unilateral brainstem lesion at the level of the CNS shown in red in the image above is MOST
likely to produce?

third nerve palsy and contralateral hemiplegia

one-and-a-half syndrome

combined abducens and facial palsies

alexia without agraphia

Your answer was CORRECT


Explanation
The image above demonstrates anatomy at the level of the midbrain. A unilateral brainstem lesion
in the midbrain could result in various syndromes, including Weber's syndrome. This refers to a
fascicular third nerve lesion within the substance of the cerebral peduncle causing an ipsilateral
third nerve palsy and contralateral hemiplegia.

Alexia with agraphia suggests a dominant parietal lobe lesion.


Alexia without agraphia suggests a dominant occipital lobe lesion.
One-and-a-half syndrome involves the MLF and ipsilateral PPRF in the pons.
Combined abducens and facial palsy again suggests the pons

Which visual field best corresponds to a patient with acute papilloedema?

enlarged blind spot


inferior quadrantanopia

generalized depression of field

superior altitudinal defect

Your answer was CORRECT


Explanation
In acute papilloedema, the visual field generally shows only an enlargement of the blind spot. The
earliest loss of visual field in chronic papilloedema is typically in the inferior nasal quadrant.

The dorsal midbrain syndrome is associated with all of the following EXCEPT:
light-near dissociation
lid retraction
paradoxic optokinetic nystagmus
upward gaze paresis
accommodative abnormalities

Your answer was CORRECT


Explanation
Paradoxic OKN is a feature of congenital nystagmus, not dorsal midbrain syndrome.

Features of Parinaud's dorsal midbrain syndrome include:


 light-near dissociation
 mydriasis
 Collier's sign (lid retraction in primary position)
 paralysis of convergence and accommodation
 paralysis of upgaze (supranuclear palsy)
 convergence-retraction nystagmus (worsened by downward rotation of OKN drum)
 skew deviation

Bromocriptine treatment may be indicated in the management of a pituitary tumour secreting:

growth hormone

TSH

FSH

prolactin

Your answer was CORRECT


Explanation
Bromocriptine is effective primarily in the management of prolactin-secreting pituitary tumors and
is less effective or ineffective with other types of pituitary tumors. Somatotropin may be helpful in
GH secreting tumours.

The indications for treatment of idiopathic intracranial hypertension include all EXCEPT:

visual field loss

severe headache

reduction in colour vision

obesity

Your answer was CORRECT


Explanation
Treatment of IIH is required for symptomatic relief of severe headache, and for preventing optic
neuropathy, which is monitored by tests of optic nerve function including visual acuity, colour
vision and fields.

Obesity is not an indication for treatment, although weight loss (even as little as 6% of total body
weight) often improves the condition.

Which is FALSE regarding relative afferent pupillary defects:

optic tract injury can cause an ipsilateral RAPD because of asymmetric decussation in the
chiasm

anisocoria is never associated directly with an RAPD

the presence of an RAPD without any visual loss suggests damage to the contralateral brainstem

media opacities do not generally cause an RAPD

Your answer was CORRECT


Explanation
A contralateral RAPD would be present if the optic tract was damaged because relatively more
nasal fibers from the contralateral eye cross in the chiasm. Approximately 52% of the optic nerve
axons cross in the chiasm, whereas 48% remain ipsilateral.

Damage to the pupillomotor fibers after they separate from the visual fibers in the midbrain can
result in a small contralateral RAPD without visual loss because of the asymmetric decussation in
the chiasm.
Anisocoria is caused by asymmetric efferent pupillomotor input. Asymmetric afferent
pupillomotor damage does not cause asymmetric input to the efferent pupillomotor system
(Edinger-Westphal nuclei) because of its double decussation in the chiasm and posterior
commissure

Which of the retrochiasmal locations below can induce a monocular visual field defect?

1. lateral geniculate body


2. parietal lobe
3. temporal lobe
4. occipital lobe

1 and 4

2 and 4

1, 2 and 3

1 alone

4 alone

Your answer was CORRECT


Explanation
The only retro-chiasmal lesion of the visual pathway to produce a monocular field defect is a
lesion of the anterior visual cortex in the occipital lobe.

The temporal 30 degrees of a binocular visual field (from 60 degrees to 90 degrees from fixation
temporally) is perceived by the nasal-most retina of the ipsilateral eye only. These temporal
crescents are represented in the anterior-most occipital lobe. Therefore, a lesion in this area will
produce a monocular visual field defect in the far temporal periphery of the contralateral eye, the
so-called temporal crescent syndrome. For example, a right anterior occipital lobe lesion would
produce a far temporal field defect in the left eye.

All of the following are consistent with a diagnosis of internuclear ophthalmoplegia EXCEPT:
impaired vestibulo-occular reflex
impaired OKN response when the drum is rotated away from the eye with the adduction
deficit
ipsilateral failure of adduction on horizontal gaze
normal convergence

Your answer was CORRECT


Explanation
In INO there is asymmetry of OKN response, with slowing of saccades in the eye with adduction deficit.
This is elicited by rotating the drum in the direction of the MLF lesion (i.e. in the direction of the eye with
the adduction deficit, this causes the eye with the adduction deficit to made an abnormally slow, corrective
saccade).

INO is characterised by the triad of:


 failure of ipslateral adduction
 ataxic nystagmus of abducting contralateral eye
 normal convergence
However, other features can include:
 slowing of saccades in adducting eye
 horizontal nystagmus of adducting eye
 asymmetric OKN response when drum rotated towards MLF lesion
 vestibulo-ocular reflex impairment
 impaired convergence (for Cogan's anterior INO)
 manifest exotropia (for bilateral INO)
 vertical nystagmus
 impaired vertical pursuit
 impaired vertical vestibulo-ocular reflex
 difficulty maintaining upgaze

A 76-year-old hypertensive man with Type 2 diabetes has a cerebrovascular accident involving
infarction of the anterior-most portion of the visual cortex.

All of the following are true EXCEPT:


a monocular field defect is typical
the defect is incongruous
the macular is likely to be spared
wide-field static perimetry up to 60 degrees from fixation is necessary to elicit its full extent

Your answer was INCORRECT


Explanation
The anterior-most visual cortex is the only part of the retro-chiasmal visual pathway that can cause
a uni-ocular field defect, in the form of a temporal crescent. The field defect extends from 60
degrees out to 90 degrees from fixation, and requires kinetic full-field perimetry to detect. Static
perimetry methods will miss it!

All of the following are included in the differential diagnosis of eyelid retraction except:
progressive supranuclear palsy
myasthenia gravis
thyroid eye disease
a history of superior rectus resection
dorsal midbrain compression

Your answer was INCORRECT


Explanation
Recession of the superior rectus muscle may cause lid retraction; superior rectus resection will
lead to ptosis.

Note: myasthenia gravis may be associated with fatigueable ptosis but it can also be associated
with eyelid retraction called the Cogan's lid twitch. This is elicited by asking the patient to first
look down for a short period and then make a saccade back to primary position, which causes a
transient lid retraction.

A 50-year-old presents with gradual onset diplopia. On examination, there is limitation of right
eye adduction, upgaze and downgaze. The right pupil is mid-dilated. There is right mild proptosis,
inferior dystopia and reduced sensation of the right forehead. The other eye is normal.

What is the most likely site of the lesion?


cavernous sinus
orbital apex
midbrain
pons
superior orbit

Your answer was INCORRECT


Explanation
The inferior dystopia, proptosis and reduced forehead sensation provide vital clues that this is a
superior orbital mass, which is affecting the superior branches of V1 (supra-orbital and
supratrochlear nerves) and compressing the third nerve downwards as it exits the orbital apex
causing an incomplete third nerve palsy.

A 35-year-old female presents with profound loss of vision in her left eye, a left relative afferent
papillary defect, but a relatively normal looking left optic disc. Her right optic disc, however,
looks slightly pale.

What is the most likely diagnosis?


demyelinating optic neuropathy
pseudotumour cerebri
sagittal sinus thrombosis
non-arteritic anterior ischaemic optic neuropathy

Your answer was INCORRECT


Explanation
The patient demographics and clinical presentation are most in keeping with retrobulbar neuritis
secondary to demyelination. She has likely had a similar episode in the other eye previously.

What is the most likely cause?


absence of greater wing of sphenoid
optic nerve meningioma
optic nerve glioma
carotid-cavernous fistula

Your answer was INCORRECT


Explanation
Clinical features suggest NF-1, with a plexiform neurofibroma of the lid and Lisch nodules. NF-1
can be associated with absence of the greater wing of sphenoid, although this is a rare
manifestation. Absence of the sphenoidal wing can cause pulsatile exophthalmos and dystopia.

pupil
 (P) [pu´pil]

the opening in the center of the iris through which light enters the eye; see also Plate 17.

Adie's pupil tonic pupil.

Argyll Robertson pupil one that is miotic and responds to accommodation effort, but not to
light.

fixed pupil a pupil that does not react either to light or on convergence, or in accommodation.

Hutchinson's pupil one that is dilated while the other is not.

tonic pupil a usually unilateral condition of the eye in which the affected pupil is larger than
the other, responds to accommodation and convergence in a slow, delayed fashion, and
reacts to light only after prolonged exposure to dark or light; see also ADIE'S SYNDROME.
Called also Adie's pupil.

Which of the following statements about giant cell arteritis is FALSE?


in a patient with ischaemic optic neuropathy and no other localising signs or symptoms, a
normal erythrocyte sedimentation rate rules out the diagnosis
it is vanishingly rare in patients younger than 50 years
about 65% of untreated patients will have contralateral involvement after permanent visual loss in
one eye
it is more common in women

Your answer was INCORRECT


Explanation
Occult GCA is a recognized entity in which atypical clinical findings are present (e.g., malaise,
anorexia). Despite a normal ESR, temporal artery biopsy results are positive and steroids are
effective.

The risk to the contralateral eye in untreated cases of GCA has been quoted in the literature
between 54% to 95%.
Which is TRUE regarding non-arteritic anterior ischaemic optic neuropathy?
visual acuity typically improves with time
FFA shows leakage from the optic disc
there is an association with giant cell arteritis
visual fields typically show an enlarged blind spot

Your answer was INCORRECT


Explanation
In non-arteritic anterior ischaemic optic neuropathy the FFA shows leakage from the optic disc.
The visual acuity does not typically improve with time and may worsen over the first couple
weeks before stabilising. Some studies have shown recovery of 3 lines of acuity or better in a
variable minority of cases, ranging from 13% to 43% (reference).  The typical visual field loss is
inferior altitudinal.

All of the following features may be found in association with bilateral internuclear
ophthalmoplegia EXCEPT:

vertical nystagmus

rotary nystagmus

horizontal nystagmus of adducting eye

asymmetric OKN response

skew deviation

Your answer was INCORRECT


Explanation
Bilateral internuclear opthalmoplegia is often associated with upbeat nystagmus and a manifest
exotropia (so-called wall-eyed bilateral internuclear ophthalmoplegia WEBINO). WEBINO is
highly suggestive of demyelinating disease, because ischaemic events tend to respect the midline
(lateralized circulation) whereas demyelinating plaques do not have to respect the midline.

INO is characterised by the triad of:

 failure of ipslateral adduction (often best seen with saccades)


 ataxic nystagmus of abducting contralateral eye
 normal convergence (if posterior INO)
However, other features can include:
 slowing of saccades in adducting eye
 asymmetric OKN response when drum rotated towards MLF lesion
 horizontal nystagmus of adducting eye
 impaired horizontal vestibulo-ocular reflex
 impaired convergence (for Cogan's anterior INO)
 manifest exotropia (for bilateral INO)
 vertical nystagmus
 impaired vertical pursuit
 impaired vertical vestibulo-ocular reflex
 difficulty maintaining upgaze

A pure, complete lesion of the optic tract is characteristically associated with:

ipsilateral homonymous hemianopia

bitemporal hemianopia

post-fixation blindness

hemifield slip

contralateral RAPD

Your answer was INCORRECT


Explanation
An optic tract lesion produces a contralateral incongruous homonymous hemianopia. It can also
produce a contralateral afferent pupillary defect (because more than half of afferent fibers cross at
the chiasm).

Note: bitemporal hemianopia, hemifield slip and post-fixation blindness are all features of a
chiasmal lesion.

A 38-year-old man presents with recurrent, unilateral, episodic temporal headache and periocular
pain over 6 weeks. The pain lasts for up to an hour. His nose is congested during an attack. He has
anisocoria and ptosis on the same side as the pain during an attack. He had a recent MRI head
which was normal.

What is the most likely diagnosis?

central Horner's syndrome

Hutchison's pupil

Herpes zoster ophthalmicus

post-ganglionic Horner's

pre-ganglionic Horner's

Your answer was INCORRECT


Explanation
The history suggests cluster headache, which can cause a post-ganglionic Horner's syndrome.

All are true of the Hess chart EXCEPT:


it can be used to differentiate a primary esotropia from a paretic squint

a markedly sloping field suggests torsion

a compressed field in opposing directions suggests mechanical restriction

a larger field is always from the normal eye

in longstanding paretic squints the Hess charts of the two eyes become symmetrical

Your answer was INCORRECT


Explanation
The Hess chart cannot measure torsion, it can only measure deviations in the vertical and
horizontal planes. To measure torsion, a double Maddox rod or a synoptophore can be used.

Regarding the inferior oblique:


the primary action is extortion, which is isolated at 51 degrees abduction
the primary action is elevation, which is isolated at 51 degrees abduction
the primary action is extortion, which is isolated at 39 degrees abduction
the primary action is elevation, which is isolated at 39 degrees abduction

Your answer was INCORRECT


Explanation
The inferior oblique's primary action is extorsion, which is isolated at 39 degrees abduction; while
at 51 degrees adduction the secondary action of the inferior oblique of elevation is isolated. The
inferior oblique is also an abductor.

Some helpful rules to try and remember the muscle actions:


 the primary action of the obliques is intorsion/extorsion
 the primary action of the recti is in the direction of their insertion (e.g. medial adduct etc)
 vertical recti (sup and inf) always ADDuct as a secondary action
 obliques always ABDuct as a secondary action
 superior muscles INtort
 inferior muscles EXtort
 39 degrees abduction isolates primary action of the obliques (intort/extort)
 51 degrees adduction isolates secondary action of the obliques (elevate/depress)
 23 degrees abduction isolates primary action of sup/inf recti (elevate/depress)
 67 degrees adduction isolates secondary action of sup/inf recti (intort/extort)

A patient with a recent stroke has a formal Goldmann visual field assessment which shows a
congruous homonymous hemianopia with macular sparing.

Ischaemia in which vessel's territory is most likely to explain this visual field finding?

anterior cerebral artery


posterior inferior cerebellar artery

posterior cerebral artery

middle cerebral artery

Your answer was INCORRECT


Explanation
A congruous homonymous hemianopia with macula sparing suggests a lesion in the primary
visual cortex (striate cortex), which is supplied primarily by the posterior cerebral artery.

Interestingly, the occipital pole may have a dual blood supply between the posterior and middle
cerebral arteries, which may explain the macular sparing phenomenon.

Which of the following congenital optic disc anomalies is associated with a risk for developing
serous macular detachments?

optic disc coloboma

persistent myelination of the nerve fibre at the disc

morning glory

optic nerve hypoplasia

optic nerve pit

Your answer was INCORRECT


Explanation
Optic disc pits are associated with serous macular detachments.

Which feature is LEAST consistent with an acquired sixth nerve palsy:

amblyopia

esotropia

head turn toward the side of the paretic muscle

incomitance

deviation greater at distance than near

Your answer was INCORRECT


Explanation
Unless the palsy is acquired early in childhood, fails to resolve and goes untreated, amblyopia is
highly unlikely.

A 74-year-old man with a sudden-onset left inferior homonymous quadrantanopia is most likely to
exhibit which other feature on examination?

unformed visual hallucinations

poor optokinetic nystagmus with the drum rotating left

formed visual hallucinations

poor optokinetic nystagmus with the drum rotating right

Your answer was INCORRECT


Explanation
A left inferior quadrantanopia suggests damage to the right parietal lobe. Ipsilateral pursuit may be
affected by parietal lesions. Thus, the patient could have difficulty pursuing the optokinetic drum
as it rotates towards the side of the brain lesion (that is poor OKN response when the drum is
rotated to the right).

Unformed visual hallucinations typically occur with occipital lobe damage, whereas formed visual
hallucinations are associated with temporal lobe lesions.

Nerve fibers in the optic tract are rotated:

45 degrees

90 degrees

0 degrees

180 degrees

Your answer was INCORRECT


Explanation
Nerve fibres in the optic tract are rotated 90 degrees with inferior fibres located laterally.

Which syndrome includes cranial nerve III palsy, contralateral intention tremor in the extremities
and contralateral partial hemi-paresis?
Benedikt's syndrome
Nothnagel's syndrome
Weber's syndrome
Claude's syndrome
Your answer was INCORRECT
Explanation
The features described above suggest Benedikt's syndrome, which is caused by a midbrain lesion
affecting the third nerve fasciculus and the red nucleus.

Features of Benedikt's syndrome: 1

 ipsilateral CN III palsy


 contralateral cerebellar ataxia with intention tremor (main feature)
 contralateral hemiparesis with hyperactive tendon reflexes (lesser feature)
Claude syndrome is characterised by: 2

 ipsilateral 3rd nerve palsy


 contralateral cerebellar ataxia
Weber syndrome is characterised by:
 ipsilateral 3rd nerve
 contralateral hemi-paresis
Nothnagel syndrome is characterised by:
 ipsilateral 3rd nerve
 ipsilateral cerebellar ataxia

1. Reference for Benedikt syndrome


2. Reference for Claude syndrome
3. General reference for 3rd nerve syndromes

A 38-year-old man complains of binocular diplopia following a motor vehicle accident. Findings
suggestive of a bilateral fourth nerve palsy include all EXCEPT:
right hypertropia in left gaze and left hypertropia in right gaze
a right hypertropia with right head tilt and left hypertropia with left head tilt
V pattern esotropia
excyclodeviation of 5 degrees on double Maddox rod testing

Your answer was INCORRECT


Explanation
Features suggesting a bilateral fourth nerve palsy are:
 excyclodeviation over 10 degrees on double Maddox rod testing
 a right hypertropia with right head tilt and left hypertropia with left head tilt
 right hypertropia in left gaze and left hypertropia in right gaze
 V pattern esotropia
 chin down head position (no head tilt)
If a single object stimulates two non-corresponding retinal points, all of the following may occur
EXCEPT:

stereopsis

confusion

diplopia

peripheral suppression

Your answer was INCORRECT


Explanation
A single object stimulating two non-corresponding retinal points is responsible for stereopsis. If
the images are too disparate, then diplopia results. In longstanding cases of diplopia, suppression
of the peripheral retina occurs to relieve the diplopia.

Note that confusion occurs when corresponding retinal points centrally (such as the fovea) fixate
on different objects. Confusion is relieved by central suppression.

Ocular disorders associated with optic nerve hypoplasia include all EXCEPT:
aniridia
Brown's syndrome
albinism
coloboma

Your answer was INCORRECT


Explanation
Causes of optic nerve hypoplasia:
 de Morsier's syndrome (rare but exam favourite!)
 aniridia
 high myopia
 albinism
 coloboma
 Goldenhar
 Duane's
 microphthalmos
 Aicardi
 Potter's
 midline facial defects: cleft palate, hare lip

Ocular pulsations may be seen in all of the following EXCEPT:


carotid-cavernous sinus fistulas
capillary haemangioma
orbitoencephaloceles
neurofibromatosis

Your answer was INCORRECT


Explanation
Ocular pulsations are either from:

 Abnormal vascular flow:


o AV malformations
o carotid-cavernous fistulas
 Transmission of normal intracranial pulsations:
o mucocele/encephalocoele
o surgical removal of bone
o sphenoid abnormalities as seen in neurofibromatosis

Note: the latter tend to cause pulsation without audible bruits.

Capillary haemangiomas consist of endothelial cells and small vascular spaces. Although typically
located on the periocular skin of the lids, a significant orbital component may be present causing
proptosis. The flow through these tumours is not high enough to cause ocular pulsation

After a stroke, a patient is only aware of moving objects in the left hemi-field while stationary
objects are not seen. This phenomenon is called:

Purkinje phenomenon

Riddoch phenomenon

Haidinger's brush

Palinopsia

Pulfrich phenomenon

Your answer was INCORRECT


Explanation
Riddoch phenomenon is caused by lesions of the occipital lobe. Moving objects are not perceived
to have colour or shape. Patients may be aware of movement without perception of it
(agnosanopsia) and may deny blindness (Anton syndrome).

For a patient to be reassured that systemic disease is unlikely, ocular myasthenia should remain
localized for what length of time?

3 months

1 year
6 months

2 years

5 years

Your answer was INCORRECT


Explanation
Bever and co-workers reported that 82% of patients who later developed generalized weakness did
so in the first 2 years after diagnosis of ocular myasthenia.[2] Hence, patients who keep having
strictly ocular symptoms for 2 or more years are unlikely to revert to the generalized aspect of the
disease.

A lesion causing ipsilateral ophthalmoplegia and contralateral hemiplegia is most likely to be


located in the:

dorsal midbrain

ventral midbrain

dorsal pons

ventral pons

Your answer was INCORRECT


Explanation
A third nerve palsy (causing ophthalmoplegia, ptosis and mydriasis) with contralateral hemiplegia
is called Weber’s syndrome. Weber's syndrome results from a defect (usually ischaemia) of the
ventromedial midbrain.

This question appeared in the 2014 FRCOphth Part 2.

What is the mechanism of action of hydroxyamphetamine?

releases noradrenaline from the pre-synaptic terminal

agonist of the noradrenaline receptor

inhibits catechol-o-methyl transferase (COMT)

prevents reuptake of noradrenaline

Your answer was INCORRECT


Explanation
Hydroxyamphetamine causes the release of noradrenaline from the presynaptic terminal. In first
and second order Horner's, noradrenaline released from the third order neuron causes dilation of
the pupil. In third order Horner's, this neuron is damaged and no noradrenaline is released (no
dilatation).

The synergist of the right superior rectus is the:

right inferior rectus

left inferior rectus

right inferior oblique

left inferior oblique

right superior oblique

Your answer was INCORRECT


Explanation
Synergists are pairs of muscles in the same eye which move the eye in the same direction (e.g.
right superior rectus and right inferior oblique)

Agonist-antagonists are pairs of muscles in the same eye that move the eye in opposite directions
(e.g. right lateral rectus and right medial rectus)

Yoke muscles are pairs of muscles in different eyes which produce conjugate, directional gaze.
Thus the yoke muscle of the left superior oblique (acting to cause the left eye to depress and
adduct) is the right inferior rectus (causing the right eye to depress and abduct) for gaze down and
right.

All are characteristics of Holmes-Adie pupil EXCEPT:

light-near dissociation

vermiform pupil movements

slow constriction and dilatation

reduced deep tendon reflexes

denervation hypersensitivity to 1:1000 adrenaline

Your answer was INCORRECT


Explanation
In Adie's pupil there is denervation hypersensitivity to dilute pilocarpine 0.125%. The Adie's pupil
will constrict, while a normal pupil will not. Adie's pupil is caused by lesions at the level of the
ciliary ganglion or the short ciliary nerves. It can be idiopathic or a similar, tonic pupil can occur
secondary to syphilis, diabetes, trauma or surgery.
A 36-year-old Afro-Caribbean woman presents with floaters and blurred vision in her right eye.
Slit lamp examination shows nodules on the iris. A chest X-ray reveals hilar adenopathy.

Biopsy of her lacrimal gland would likely reveal:

multinucleated giant cells of the Langhans type

epithelioid cells within necrotic tissue

multinucleated giant cells of the Touton type

diffuse lymphocytic infiltration

Your answer was INCORRECT


Explanation
The clinical description is highly suggestive of sarcoidosis. A lacrimal gland biopsy is useful in
the diagnosis of sarcoidosis, especially if it is enlarged. The gland is involved in approximately
25% of patients.

The histologic lesion found in sarcoid biopsies is the non-caseating epithelioid cell tubercle. The
tubercle is composed of multinucleated giant cells of the Langhans type surrounded by a rim of
lymphocytes.

Touton giant cells classically are found in juvenile xanthogranuloma.

A patient has corneal clouding and optic atrophy with dermatan sulphate accumulation.

What is the diagnosis?


Maroteaux-Lamy syndrome

Sanfilippo syndrome

Morquio syndrome

Hurler-Scheie syndrome

Hunter syndrome

Your answer was INCORRECT


Explanation
Maroteaux-Lamy syndrome is part of the mucopolysaccharidoses group of enzyme deficiency
disorders. It results in excessive accumulation of dermatan sulphate. This causes corneal clouding
and optic atrophy.

Signs of high-grade carotid artery stenosis include all EXCEPT:


anterior segment flare
ipsilateral corneal arcus
hypotony
cataract

Your answer was INCORRECT


Explanation
Unilateral arcus is highly suggestive of contralateral carotid disease. Carotid stenosis protects the
ipsilateral cornea from serum lipid deposition.

Causes of unilateral corneal arcus:


 An atrophic eye
 Ipsilateral relative ocular hypotony
 Contralateral carotid artery disease 1-3

 Contralateral Sturge-Weber syndrome 4

All of the following drugs may be associated with idiopathic intracranial hypertension EXCEPT:
tetracycline
gentamicin
vitamin A
corticosteroids
amiodarone

Your answer was INCORRECT


Explanation
Drug causes of raised ICP:
 tetracyclines
 fluoroquinolones
 vitamin A/retinoids
 progesterones
 corticosteroids
 anabolic steroids
 amiodarone
 lithium
 phenytoin
 tamoxifen

A third cranial nerve palsy which involves the pupil is MOST likely in which setting:

hypertension

aneurysm

diabetes
vasculitis

Your answer was INCORRECT


Explanation
Aneurysms damage cranial nerves by extrinsic compression. The parasympathetic pupillomotor
fibers controlling the pupil are found on the outside of the oculomotor nerve and are the first to be
injured with external pressure. Diabetic microvascular insults affect the central fibers of the nerve
to a greater degree than the outer fibers, and as a result, pupil involvement is less common
(<20%).

What type of cells are seen in optic nerve sheath meningioma?

asteroid bodies

psammoma bodies

Schaumann bodies

verocay bodies

Your answer was INCORRECT


Explanation
Optic nerve sheath meningioma has two main cell types: psammoma bodies and meningothelial.

Note: asteroid bodies and Schaumann bodies are seen in sarcoidosis; while verocay bodies are
seen in neurilemmoma.

Optic atrophy is LEAST likely to occur in this mucopolysaccharidosis:


Type III (Sanfilippo's syndrome)

Type IS (Scheie's syndrome)

Type VII (Sly's syndrome)

Type II (Hunter's syndrome)

Your answer was INCORRECT


Explanation
Optic atrophy has been described in but is not a major feature of Sly syndrome (Type VII). Optic
atrophy is more commonly encountered in the other mucopolysaccharidoses presented above. 
Reference: Textbook of Ophthalmology by Nema and Nema. JP Medical; 2011.
A lesion of the tip of the occipital lobe (the occipital pole) is MOST likely to produce which visual
field defect:

congruous homonymous paracentral scotoma

bilateral homonymous hemianopia with macular sparing

congruous homonymous hemianopia with macular sparing

incongruous homonymous hemianopia

Your answer was INCORRECT


Explanation
A lesion of the tip of the occipital lobe will cause a highly congruous paracentral scotoma. This is
because the posterior 50% of the visual cortex subserves the central 10 degrees of the visual field,
and 80% of the visual cortex subserves the central 30 degrees of the visual field. As such defects
isolated to the posterior occipital pole will tend to be confined to congruous paracentral scotomas.
More anterior lesions of the visual cortex will affect more peripheral vision and will produce
congruous homonymous hemianopias, often with macular sparing. Lesions at the anterior extreme
of the visual cortex can produce monocular temporal crescents between 60 degrees to 90 degrees
from fixation.

A 55-year-old right-handed man presents with a left homonymous hemianopia without sparing of
the macula.

Which of the following is most likely to accompany his presentation:


alexia
object agnosia
speech impairment
prosopognosia

Your answer was INCORRECT


Explanation
This man has a lesion of his non-dominant parieto-occipital lobe.

Non-dominant parietal lobe lesions cause:


 hemispatial neglect
 difficulty with spatial orientation
 dressing apraxia
 object agnosia (or astereoagnosia: unable to name object in hand with eyes closed)

Dominant parietal lobe lesions cause:


 dyscalcula
 dysgraphia
 left-right disorientation
 finger agnosia
 alexia
 speech disturbance

Note the first four features above constitute Gerstmann syndrome

Other, retrochiasmal visual disorders:


 alexia with agraphia: dominant parietal lobe (angular gyrus)
 alexia without agraphia: dominant occipital lobe and corpus callosum
 hemi-alexia: corpus callosum
 prosopagnosia: bilateral inferior occipito-temporal junction
 formed hallucinations: temporal lobe
 unformed hallucinations: occipital lobe
 Balint syndrome (bilateral superior parieto-occipital lesions):
o inability to perceive the visual field as a whole (simultanagnosia)
o difficulty in fixating the eyes (oculomotor apraxia)
o inability to move the hand to a specific object by using vision (optic ataxia)

This question came in the FRCS (Glasgow) October 2014 exam.

A patient with a bulbar syrinx is found on examination to have nystagmus in which one eye
elevates and intorts while the other depresses and extorts.

What is the MOST likely diagnosis?


periodic alternating nystagmus
congenital nystagmus
Maddox's nystagmus
dorsal midbrain syndrome
spasmus mutans

Your answer was INCORRECT


Explanation
The description fits with see-saw nystagmus of Maddox.

The causes of see-saw nystagmus are:


 syringobulba
 brain-stem CVA
 parasellar masses

A lesion involving both medial longitudinal fasciculi near their junctions with the third nerve
nuclei may cause:

one-and-a-half syndrome

INO with skew


Foville's syndrome

WEBINO

Fisher's syndrome

Your answer was INCORRECT


Explanation
Bilateral INO may result in exotropia in primary position resulting in a syndrome called wall-eyed
bilateral INO (WEBINO).

Which statement is TRUE regarding the Hess chart?

with time, the charts in both eyes become more incomitant

each square on the chart represents 10 prism dioptres

the smaller chart indicates the overacting eye

the bigger charter indicates the eye with the paretic muscle

Your answer was INCORRECT


Explanation
The smaller chart indicates the eye with the paretic muscle, while the larger chart indicates the
overacting eye. Each square on the chart represents 10 PD and gives a good indication of the size
of the deviation. Due to muscle sequelae over time, the charts become more concomitant.

An 81-year-old lady presents to casualty with sudden onset right ptosis and left hemi-tremor worse
on intention movements. On examination, there is partial left hemi-paresis and ocular motility
shows limited adduction and elevation of the right eye.

What is the diagnosis?


Nothnagel's syndrome
Benedikt's syndrome
Weber's syndrome
Foville's syndrome
Claude's syndrome

Your answer was INCORRECT


Explanation
The features described above suggest Benedikt's syndrome, which is caused by a midbrain lesion
affecting the third nerve fasciculus, the red nucleus and cerebral peduncle.

Features of Benedikt's syndrome: 1


 ipsilateral CN III palsy
 contralateral cerebellar ataxia with intention tremor
 contralateral hemiparesis (usually partial) with hyperactive tendon reflexes
Claude syndrome is characterised by: 2

 ipsilateral 3rd nerve palsy


 contralateral cerebellar ataxia
Weber syndrome is characterised by:
 ipsilateral 3rd nerve
 contralateral hemi-paresis
Nothnagel syndrome is characterised by:
 ipsilateral 3rd nerve
 ipsilateral cerebellar ataxia

1. Reference for Benedikt syndrome


2. Reference for Claude syndrome
3. General reference for 3rd nerve syndromes

Which clinical feature might be expected in a young child with bilaterally poor vision and the CT
scan shown?
pheochromocytoma

panhypopituitarism

ash-leaf macules

mental retardation and lacunar peripheral retinopathy


precocious puberty

Your answer was INCORRECT


Explanation
The image shows absence of the septum pellucidum, a thin layer of serous connective tissue
separating the two lateral ventricles. This is associated with bilateral optic nerve hypoplasia and
hypothalamic-pituitary disturbances as part of de Morsier's syndrome.

Aicardi's syndrome features mental retardation, lacunar retinopathy, and/or congenital absence of
the corpus callosum.

Precocious puberty may be a part of polyostotic fibrous dysplasia (Albright's syndrome).

Ash-leaf macules are seen in tuberous sclerosis while pheochromocytoma may be seen in
neurofibromatosis and von Hippel-Lindau disease.

All statements are true of the abducens nerve EXCEPT:

it is bound down to the brainstem close to its origin by the anterior superior cerebellar
artery

it courses upwards in the pontine cistern

it changes from a vertical to horizontal course on reaching the apex of the petrous temporal bone

it enters the orbit through the superior orbital fissure and within the tendinous ring

Your answer was INCORRECT


Explanation
The abducens nerve is bound down to the brainstem close to its origin by the anterior inferior
cerebellar artery

The pterygopalatine ganglion:

contains preganglionic fibers that arise in the lacrimal nucleus of the facial nerve

receives preganglionic parasympathetic fibers from the lesser petrosal nerve

contains secretomotor nerve to the parotid gland

is a sympathetic ganglion

Your answer was INCORRECT


Explanation
The pterygopalatine ganglion is a parasympathetic ganglion. Its preganglionic fibers arise from the
lacrimal nucleus of the facial nerve and they run in the greater petrosal nerve before synapsing in
the ganglion. The ganglion contains secretomotor nerve fibres to the lacrimal gland.

Hemifacial spasm is most likely to be caused by:

compression of the facial nerve in the cerebellopontine angle by anomalous vessels

a pontine lesion involving the facial nucleus or fascicle

cerebellar dysfunction

basal ganglia dysfunction

Your answer was INCORRECT


Explanation
Hemifacial spasm is due to compression of the facial nerve in the cerebellopontine angle by
anomalous vessels, which has been demonstrated in 90% of cases. Tumors in the cerebellopontine
angle also can cause hemifacial spasm.

Essential blepharospasm is related to basal ganglia dysfunction.

Facial myokymia (involuntary, spontaneous, localised quivering of facial muscles) is caused by


disease in the pons involving the facial nucleus or fascicle. The most common causes include
multiple sclerosis (MS) in adults and pontine glioma in children.

Regarding adrenaline and noradrenaline:

both cause vasodilatation in the skeletal muscles

both are produced by the adrenal medulla

their release is under the control of the pituitary gland

both cause increased peripheral resistance of the blood vessels

Your answer was INCORRECT


Explanation
Both adrenaline and noradrenaline are produced by the adrenal medulla. They are secreted in
response to stress. Adrenaline causes vasodilatation in the skeletal muscles but noradrenaline has
no such effect. While adrenaline decreases the peripheral resistance of the blood vessels,
noradrenaline increases it.

In a paretic squint which muscle sequelae happens last:


underaction of an agonist
inhibitional palsy of a Yoke muscle antagonist
overaction of Yoke muscle
contracture of unopposed antagonist

Your answer was INCORRECT


Explanation
The sequelae of a paretic squint are:
 Primary underaction (e.g. left superior oblique palsy)
 Overaction of yoke muscle by Hering's law (e.g. right inferior rectus)
 Secondary contracture of unopposed antagonist (e.g. left inferior oblique)
 Secondary inhibitional palsy of the Yoke muscle's antagonist by Sherrington's law (e.g.
right superior rectus)

A healthy 46-year-old patient presents with painful double vision. Examination reveals normal
visual acuity, diminished corneal sensation, and global impairment of ocular motility in the left
eye, with no proptosis. There is pain with eye movements, and the globe and orbit are slightly
tender. CT and MRI are both normal, and the patient rapidly and completely responds to 60 mg of
oral prednisone a day.

The MOST likely diagnosis is:

orbital pseudotumour

sphenoid wing meningioma

cavernous sinus thrombosis

intracavernous carotid artery aneurysm

Tolosa-Hunt syndrome

Your answer was INCORRECT


Explanation
Tolosa-Hunt syndrome is a non-specific inflammatory process of the cavernous sinus. Findings
include painful ophthalmoplegia, sensory deficits of the ophthalmic division of the trigeminal
nerve and a dramatic response to systemic corticosteroid therapy. Remissions may be
spontaneous, with partial or complete reversal of deficits. Episodic recurrence also can occur.
Aneurysms of the circle of Willis can produce a similar picture, including steroid responsiveness.
A normal MRI makes this unlikely. Tolosa-Hunt syndrome is a diagnosis of exclusion.

Which visual phenomenon is most likely to accompany see-saw nystagmus?

formed visual hallucinations

unformed visual hallucinations

post-fixation blindness
cortical blindness

Your answer was INCORRECT


Explanation
See-saw nystagmus is a feature of chiasmal syndrome; which can also cause post-fixation
blindness. Formed visual hallucinations are caused by temporal lobe lesions, while unformed
hallucinations are caused by lesions of the occipital lobe.

Which segment of the optic nerve is shortest:


intraorbital
intracanalicular
intracranial
intraocular

Your answer was CORRECT


Explanation
The optic nerve is 45mm long and is divided into the following segments:
 intraocular: 1mm
 intraorbital: 30mm
 intracanalicular: 6mm
 intracranial: 10mm

Compared to blood the CSF has:

more lymphocytes
higher protein

lower glucose

higher immunoglobulins

Your answer was CORRECT


Explanation
Most constituents exist in lower concentration in the CSF compared to blood, including glucose,
which is 60% of its blood concentration. Only chloride, magnesium and hydrogen ions are in
higher concentration in the CSF.

The synergist of the right superior rectus is the:

right inferior oblique


right inferior rectus

right superior oblique

left inferior rectus

left inferior oblique

Your answer was INCORRECT


Explanation
Synergists are pairs of muscles in the same eye which move the eye in the same direction (e.g.
right superior rectus and right inferior oblique)

Agonist-antagonists are pairs of muscles in the same eye that move the eye in opposite directions
(e.g. right lateral rectus and right medial rectus)

Yoke muscles are pairs of muscles in different eyes which produce conjugate, directional gaze.
Thus the yoke muscle of the left superior oblique (acting to cause the left eye to depress and
adduct) is the right inferior rectus (causing the right eye to depress and abduct) for gaze down and
right.

Kearns-Sayre syndrome is primarily a disorder of:


motor nucleus
muscle fibres
neuromuscular junction
myelinated nerve fibres

Your answer was INCORRECT


Explanation
Kearns-Sayre syndrome is a myopathic condition, which can cause ptosis and chronic progressive
external ophthalmoplegia.

Kearns-Sayre is characterised by:


 mitochondrial inheritence
 ragged red fibres on muscle biopsy
 presentation in 1st or 2nd decades
 ptosis
 external ophthalmoplegia
 cardiac conduction defects
 deafness
 diabetes
 short stature
 pigmentary retinopathy
A 47-year-old man awakes with unilateral visual loss and a supero-nasal visual field defect.

What is the most likely cause:


psuedotumour cerebri
demyelinating optic neuropathy
non-arteritic ischaemic optic neuropathy
arteritic ischaemic optic neuropathy

Your answer was INCORRECT


Explanation
The presentation is most in keeping with non-arteritic ischaemic optic neuropathy. The
predispositions for this condition include:
 age 45-65
 small crowded discs
 hypertension
 diabetes
 obstructive sleep apnoea

A lesion of the right frontal lobe will result in all of the following EXCEPT:

eyes will look away from the paralyzed left side

gaze paralysis to the right

pyramidal signs on the left side

left hemiparesis

Your answer was INCORRECT


Explanation
Right frontal lobe lesion will cause left hemiparesis and deviation of eyes to the right side, with
gaze paresis to the left.

A patient presents with complete left homonymous hemianopia. On optokinetic drum testing, the
patient is unable to follow the drum when it is rotated to the patient's right, but manages to follow
on rotation to the left. Where is the lesion?

right frontal lobe

right parietal lobe

left occipital lobe

right occipital lobe

left parietal lobe


Your answer was INCORRECT
Explanation
Failure of ipsilateral pursuit occurs in parietal lobe lesions, which manifests as OKN drum
asymmetry. An occipital lobe lesion has no effect on OKN responses.

This question came in the 2014 FRCOphth.

A 66-year-old man experienced a sudden episode of monocular loss of vision in his right eye
yesterday lasting for 10-minutes before returning completely to normal. His eye examination is
unremarkable apart from moderate nuclear sclerosis. On questioning, he denies headache but does
recall a 5-minute episode of double vision recently, and he has noticed trouble chewing food
because of jaw ache.

What is the most appropriate investigation?

antiphospholipid antibody levels

temporal artery biopsy

acetylcholine receptor antibody level

erythrocyte sedimentation rate

carotid dopplers

Your answer was INCORRECT


Explanation
The most important differential to exclude acutely in this case is temporal arteritis, which is
consistent with this man's presentation (including transient diplopia which could be a result of
ocular motor nerve ischaemia and jaw claudication). An urgent ESR, CRP and blood count should
be requested and depending on clinical suspicions, a temporal artery biopsy should be arranged in
the coming days.

A 64-year-old lady experiences burning pain over the right side of her face, before developing a
left hemiplegia. Her eye movements are full but you notice a mild right ptosis.

What is the most likely diagnosis?

embolus at the top of the basilar artery

carotid artery dissection

lateral medullary syndrome

posterior cerebral artery occlusion

Your answer was INCORRECT


Explanation
Carotid artery dissection classically presents with facial pain, ipsilateral post-ganglionic Horner's
syndrome and may lead to a cerebrovascular accident affecting the contralateral side of the body if
pre-existent arterial disease has rendered the circle of Willis susceptible to unilateral occlusion.

A patient has a miosed right pupil with mild right ptosis. The following pharmacological tests
results are obtained from the right pupil:
Drop Right Pupil
Cocaine 4% No dilatation
Hydroxyamphetamine 1% No dilatation

Where is the lesion MOST likely:


lung apex

carotid artery

spinal chord

thyroid mass

Your answer was INCORRECT


Explanation
Horner’s syndrome is a very popular topic in MCQ’s and the OSCE. You must know it well.

The pharmacological test results shown are in keeping with a post-ganglionic (third-order)
Horner’s. A lesion of the carotid artery (such as carotid dissection) can cause a third-order
Horner’s. The other options cause first-order (spinal chord) or second order defects (thyroid mass,
Pancoast tumour).

This question appeared in the 2014 FRCOphth Part 2.

Regarding giant cell arteritis, which is FALSE:

the erythrocyte sedimentation rate (ESR) is typically the best haemotalogical marker of
disease activity

it is more common in females

tongue claudication is a recognized feature

the defining characteristic on histology is interruption, reduplication and fragmentation of the


internal elastic lamina with macrophage infiltration

Your answer was INCORRECT


Explanation
C-reactive protein (CRP) is of hepatic origin and usually rises before ESR in most disease states,
including giant cell arteritis. In fact, CRP has a higher sensitivity and specificity than ESR for
GCA and is relatively insensitivity to age, gender, and other hematologic parameters. In these
respects, CRP is often a better marker for disease activity in GCA than ESR.

This question came in the 2014 FRCOphth.

Which contralateral extraocular muscle is LEAST affected as a result of a compressive third nerve
palsy?

lateral rectus

inferior rectus

superior rectus

inferior oblique

Your answer was INCORRECT


Explanation
The word "compressive" has been added to this question to highlight that we are talking here
about a peripheral third nerve palsy and not a nuclear third nerve palsy. In other words, we are not
referring to ipsilateral and contralateral innervation supplied by the third cranial nerve nucleus.
We are instead asking about muscle sequelae on the contralateral eye after an oculomotor palsy.

Third nerve palsy primarily causes overaction of the contralateral abductors and elevators: lateral
rectus, superior rectus and inferior oblique. It has little consequence on the contralateral inferior
rectus (which is instead affected in superior oblique palsy).

Downbeat nystagmus may be a result of all of the following EXCEPT:


pinealoma
a paraneoplastic syndrome
normal lithium use
craniocervical junction abnormalities

Your answer was INCORRECT


Explanation
Downbeat nystagmus has been associated with:
 Drugs (lithium, phenytoin, barbituates)
 Wernicke's encephalopathy
 Paraneoplastic cerebellar degeneration (e.g. small cell lung)
 Craniocervical junction lesions (tumour, syrinx, Arnold-Chiari)
Pinealomas or other lesions in the dorsal midbrain result in Parinaud's dorsal midbrain syndrome.

See-saw nystagmus is MOST likely to be associated with:

pinealoma

craniopharyngioma

nasopharyngeal carcinoma

Arnold-Chiari malformation

Your answer was INCORRECT


Explanation
See-saw nystagmus is characteristically a feature of chiasmal lesions.

Which is FALSE regarding Leber's hereditary optic atrophy:

it affects boys more than girls

it has an insidious onset

it is mitochondrially inherited

peripapillary telangiectasia are characteristic

Your answer was INCORRECT


Explanation
LHON is characterized by rapid onset visual failure. It affects boys more than girls for reasons
that are not understood. On presentation, there is papilloedema and peripapillary telangiectasia
progressing to optic atrophy and vision loss.

Which is NOT an example of aberrant regeneration?


superior oblique myokymia

crocodile tears

pseudo-Graefe's sign

Marcus-Gunn jaw wink

Your answer was INCORRECT


Explanation
Superior oblique myokymia is an episodic twitching of the superior oblique muscle. The exact
cause is unknown, but carbamazepine and propranolol have been effective treatments

A 65-year-old man presents to A&E with unsteadiness, left facial pain, hoarseness and double
vision. On examination, he has a wide-based gait and left intention tremor. There is mild left
ptosis and a fast-phase nystagmus to the left.

What is the most likely diagnosis?


Balint syndrome
Nothnagel syndrome
Wallenberg syndrome
Steele-Richardson-Olszewski syndrome
Syringobulbia

Your answer was INCORRECT


Explanation
This patient has a Horner's syndrome (partial ptosis) with ipsilateral cerebellar signs, ipsilateral
fifth nerve involvement (facial pain), probable sixth nerve involvement (double vision) and bulbar
signs (hoarseness). These features suggest lateral medullary syndrome (also called Wallenberg
syndrome). There can also be sensory loss and hemiparesis on the contralateral side.

Lateral medullary syndrome (Wallenberg syndrome) is characterised by deficits in:


 sympathetic pathway: ipsilateral central Horner's
 vestibular nuclei: vertigo and nystagmus
 5th nerve nucleus: ipsilateral loss of facial sensation
 cerebellum: ipsilateral cerebellar signs
 lateral spinothalamic tract: contralateral pain/temperature loss in trunk and limbs
 nucleus ambiguous: bulbar signs such as hoarseness and speech problems

A patient with a chiasmal lesion undergoes Goldman visual fields, which shows a uniocular "pie
in the sky" defect on the left.

Where is the lesion located?


postero-inferior chiasm on the left

anterior chiasm on the right

postero-inferior chiasm on the right

anterior chiasm on the left

Your answer was INCORRECT


Explanation
This patient has a junctional scotoma, caused by a chiasmal lesion at the junction of the optic
nerve and chiasm. This usually causes an ipsilateral central scotoma and a contralateral, uniocular
"pie-in-the-sky" supero-temporal scotoma. The reason for these defects is that the lesion involves
the ipsilateral optic nerve and von Willebrand's knee, which is a group of fibres from the
contralateral infero-nasal retina representing the contralateral supero-temporal field.

This question appeared in the 2014 FRCOphth Part 2.

Note: the term "pie-in-the-sky" visual field defect is also used to refer to a homonymous upper
quadrantanopia due to a temporal lobe lesion, while a "pie-in-the-floor" lesion refers to a
homonymous lower quadrantanopia due to a parietal lobe lesion.

What is the most common cause of communicating hydrocephalus?

sub-arachnoid haemorrhage

Colloid cyst

Arnold-Chiari malformation

meningitis

Your answer was INCORRECT


Explanation
Communicating hydrocephalus (non-obstructive hydrocephalus) is caused by impaired CSF re-
absorption in the absence of any CSF flow obstruction between the ventricles and subarachnoid
space. This is due to functional impairment of the arachnoid granulations, which are located along
the superior sagittal sinus and are the site of CSF re-absorption back into the venous system.
Various conditions may result in communicating hydrocephalus, most commonly subarachnoid
haemorrhage, meningitis and congenital absence of arachnoid villi.
Non communicating (obstructive) hydrocephalus is caused by CSF flow obstruction due to
external compression or intra-ventricular mass lesions (e.g. Colloid cyst).

This question came in the 2014 FRCOphth.

A 45-year-old woman comes to the emergency room with complaints of double vision and
headache for the last 2 days. On examination, the left eye is turned downward and outward. The
pupil is larger compared to the right.

What is the site and type of lesion:

laceration or contusion in the parotid region

stretching of the nerve as it courses around the brainstem

fracture of the cribriform plate

pressure from herniating uncus on the nerve or fracture in the cavernous sinus

acoustic neuroma

Your answer was INCORRECT


Explanation
Pressure from the uncus, a fracture in the cavernous sinus, or aneurysms (especially of the
posterior communicating artery) can damage the occulomotor nerve (III), resulting in a dilated
pupil, ptosis, the eye being turned downward and outward, and a loss of the pupillary light reflex
on the side of the lesion.

Which is FALSE regarding non-arteritic anterior ischemic optic neuropathy (NAION) ?


vision loss is typically less severe than in the arteritic variety
aspirin has no clear effect on prophylaxis of the other eye from NAION
there is a 15% chance of subsequent occurrence in the contralateral eye within 5 years
fluorescein angiography is usually not helpful in differentiating it from optic neuritis
hypertension is the most frequent systemic association

Your answer was INCORRECT


Explanation
NAION is far more common than AAION (approximately 95% vs 5%). Patients with NAION
have a lower mean age at diagnosis than patients with AAION (60 years vs 70 years) and optic
neuritis is an important differential in NAION.

The ways to differentiate NAION from optic neuritis include:


 the absence of pain with eye movement
 the age group affected (younger in optic neuritis)
 delayed optic disc filling present in 75% of NAION cases (whereas filling should be
normal in optic neuritis)
Aspirin has no proven effect on reducing the risk of NAION in the contralateral eye.

According to Alexander's law, in which position should upbeat nystagmus be most prominent?

left gaze

down gaze

right gaze

convergence

up gaze

Your answer was INCORRECT


Explanation
Alexander's law refers to the increased frequency and amplitude of nystagmus movements
accompanying gaze in the direction of the fast-beating component. Downbeat nystagmus most
frequently violates the law.
A 74-year-old hypertensive, diabetic patient has diplopia on upgaze. On examination, abduction
and adduction are full bilaterally, but on upgaze the left eye only elevates halfway up. His pupils
appear normal.

Which additional finding is MOST likely in this case?


right upper lid retraction

miotic left pupil

left nystagmus on upgaze

chin-down posturing

Your answer was INCORRECT


Explanation
This patient has an isolated superior division of cranial nerve III paresis affecting the left superior
rectus and levator palpebrae. The most likely cause for this condition is a diabetic ischemic
neuropathy. The weakened levator would cause a left ptosis. He can compensate for this by
overstimulating the levator to raise the left lid, consequently raising the right lid higher.

Which of the following are seen histologically in neurilemmoma?

verocay bodies

Schaumann bodies

psammoma bodies

asteroid bodies

Your answer was INCORRECT


Explanation
Verocay bodies are seen in neurilemmoma

Asteroid and Schaumann bodies are seen in sarcoidosis.

Optic nerve sheath meningioma has two main cell types: psammoma bodies and meningothelial.

Where is the cell body of the second order neurone in the sympathetic pathway for the pupil?
ciliospinal center of Budge
hypothalamus
superior cervical ganglion
ciliary ganglion

Your answer was INCORRECT


Explanation
Sympathetic pathway:
 First order: hypothalamus along the brainstem to synapse in the ciliospinal center of
Budge
 Second order: centre of Budge to superior cervical ganglion
 Third order: superior cervical ganglion, travels along the carotid plexus, branches join the
ophthalmic division of the trigeminal nerve and pass through the ciliary ganglion to the
nasociliary and short ciliary nerves

A patient presents with acute onset vertigo. On examination, you notice hoarseness of the voice
and right-sided dysmetria and dysdiadokinesia. There is reduced sensation to pin-prick and
temperature on the left side of the body and reduced sensation to pin-prick and temperature on the
right side of the face.

Where is the lesion?


right side of spinal chord at C1-2
right side of medulla
right side of cerebella vermis
right side of pons

Your answer was INCORRECT


Explanation
The signs and symptoms described are consistent with lateral medullary syndrome. These include:
 ipsilateral vestibular nuclei symptoms (vertigo, nausea, nystagmus)
 ipsilateral cerebellar signs such as dysdiadokinesia and dysmetria
 lateral spinothalamic tract damage causing contralateral pain and temperature sensation
loss
 spinal trigeminal nucleus damage causing ipsilateral facial paraesthesia
 ipsilateral Horner’s
 ipsilateral bulbar signs such as hoarseness and dysphagia

This question came in the 2014 FRCOphth.

The commonest defect of colour vision is due to:

protanomaly

protanopia

deuteranopia

deuteranomaly

Your answer was INCORRECT


Explanation
The commonest defect of colour vision is deuteranomaly which occurs in 5% of men and 0.3% of
women. It is due to a shift in the spectral sensitivity of green cones. Deuteranopia indicates
complete absence of green cones and is rare. The genes for red and green pigment are on the X
chromosome while chromosome 7 carries the blue pigment gene. Thus deuteranomaly and
protanomaly are X-linked.

All of the following are characteristics of morning glory syndrome EXCEPT:


unilateral

more common in men

glial tissue and radiating blood vessels

peripapillary atrophy

visual acuity is typically worse than 6/12

Your answer was CORRECT


Explanation
Figure: Morning glory syndrome.

Morning glory syndrome is characterised by:


 congenital
 unilateral
 glial tissue and radiating vessels
 peripapillary atrophy
 poor vision - only 30% have vision better than 6/12 1

 more common in women than in men (2:1)

Reference 
1. Harasymowycz P., Chevrette L., Decarie J. Morning glory syndrome: clinical,
computerized tomographic, and ultrasonographic findings. J Pediatr Ophthalmol
Strabismus. 2005;42:290–295. 
2. Dutton GN. Congenital disorders of the optic nerve: excavations and hypoplasia. Eye
(Lond). 2004 Nov;18(11):1038-48.
f cold water is irrigated in the left ear of an awake patient, in what direction is the slow phase of
the nystagmus?

down

right

up

left

Your answer was INCORRECT


Explanation
The mnemonic COWS (cold opposite; warm same; in awake patient) is useful and gives the
direction of the fast phase of the nystagmus. In this case, the left eye is irrigated with cold water,
so the fast phase will be toward the right and slow phase to the left.

A right-handed 72-year-old man has a stroke after which he suffers from alexia, agraphia and a
homonymous visual field defect.

What is the most likely finding on visual field testing?


incongruous right lower homonymous quadrantanopia
incongruous left lower homonymous quadrantanopia
congruous right homonymous hemianopia
congruous left homonymous hemianopia

Your answer was INCORRECT


Explanation
Alexia and agraphia are features of a dominant parietal lobe lesion (left lobe in a right-handed
individual), which will be consistent with an incongruous right lower homonymous
quadrantanopia.

Note: alexia can also occur in dominant occipital lobe lesions but it occurs without agraphia
(patient can write but cannot read what is written!)

Dominant parietal lobe lesions cause:


 dyscalcula
 dysgraphia
 left-right disorientation
 finger agnosia
 alexia
 speech disturbance

Note the first four features above form Gerstmann syndrome of the dominant parietal lobe.
Eaton-Lambert syndrome is primarily a disorder of the:

neuromuscular junction

muscle fibres

motor nucleus

nerve axon

Your answer was INCORRECT


Explanation
Eaton-Lambert syndrome is an autoimmune disease similar to myasthenia gravis, caused by
antibodies against voltage-gated calcium channels. It is often a paraneoplastic syndrome, typically
caused by small cell lung cancer. Abnormalities of conduction at the neuromuscular junction lead
to muscle weakness and fatigue. Ocular manifestations are less common with Eaton-Lambert than
with myasthenia gravis.

A 42-year-old lady presents with left visual loss to perception of light developing over 2 weeks.
She has associated left-sided periocular pain. The left optic disc is swollen. She has weakness and
numbness in her lower limbs. Lumbar puncture showed elevated total protein levels and
pleocytosis. There was no oligoclonal band. The MRI brain was normal. Spinal MRI showed
contiguous signal abnormality over 3 vertebral segments.

Which test is MOST likely to confirm the diagnosis?


anti-voltage gated calcium channel antibodies
serology for Lyme disease
serum ACE and chest X-ray
serum anti-aquaporin 4 antibodies
nerve conduction studies

Your answer was INCORRECT


Explanation
The history is suggestive of neuromyelitis optica, which has been associated with anti-aquaporin 4
antibodies.

Devic's is characterised by:


 optic neuritis (often severe, and may be bilateral)
 transverse myelitis (T-2 lesions on MRI extending over 4 contiguous vertebral sections)
 positive for anti-aquaporin 4 antibodies

Which is FALSE regarding myasthenia gravis?

anti-ACh receptor antibodies are more prevalent in systemic than ocular myasthenia
70% of patients present with ocular signs

ocular signs are present in 90% of patients at some point in the disease

thymectomy is more effective at inducing remission of MG in patients with thymoma than


those without thymoma

Your answer was INCORRECT


Explanation
80% of MG patients without thymoma had remission of disease after thymectomy. This compares
to only 10% of patients with thymoma having remission of MG after thymectomy. Other
statements above are true.

What is the mechanism of action of edrophonium?

prevents reuptake of ACh

releases ACh from the presynaptic terminal

direct-acting cholinergic agonist

inhibits acetylcholinesterase

Your answer was INCORRECT


Explanation
Edrophonium (Tensilon) is a short-acting anti-cholinesterase inhibitor. It prolongs the duration of
acetylcholine in the synaptic cleft.

Regarding congenital nystagmus which is FALSE:


nystagmus is minimal at the null position
manifests reverse optokinetic nystagmus
it is accentuated by fixating a near object
it is associated with nystagmus blockage syndrome

Your answer was INCORRECT


Explanation
Fixation on a distant object usually worsens congenital nystagmus. Conversely, purposely
inducing esotropia or near fixation is used to suppress the nystagmus (nystagmus blockage
syndrome).

Optokinetic drum responses are reversed in congenital nystagmus. The pursuit movement in a
normal person is towards the direction of movement of the drum, with the quick phase in the
opposite direction; this is reversed in congenital nystagmus.
Characteristics of congenital nystagmus:
 jerk or pendular nystagmus
 normal or near-normal visual acuity
 no change in nystagmus with unilateral occlusion or blurring (as opposed to manifest
latent nystagmus)
 fast phase switches: to right in right-gaze, to left in left-gaze
 null point can occur in any position of gaze
 compensatory head postures vary by patient: face-turns either way or chin up, chin down
depending on the position of the null point
 titubation
 tends to dampen with convergence, darkness, sleep
 increases with distance fixation, often dampens with near fixation
 paradoxical OKN response

The most likely diagnosis in the figure above is:


medulloepithelioma
melanocytoma
congenital hypertrophy of the RPE
choroidal melanoma
Your answer was INCORRECT
Explanation
Optic nerve head melanocytomas:
 darkly pigmented lesions
 typically benign
 rarely can undergo malignant transformation
 RAPD and VF defects possible
 followup recommended to document growth

A 40-year-old woman was referred for evaluation of ptosis and abnormal eye movements. Ocular
motility shows generalised ophthalmoplegia.

All of the following are recognised clinical associations with this history EXCEPT:
pigmentary retinopathy
French-Canadian ancestry
sluggishly reactive pupils
polychromatic lenticular deposits
cardiac conduction defects

Your answer was INCORRECT


Explanation
Ptosis and generalised ophthalmoplegia suggest either myasthenia or chronic progressive external
ophthalmoloplegia (CPEO). Neither of these conditions affect pupillary reactivity.

CPEO can be caused by:


 myotonic dystrophy (associated with polychromatic cataract)
 oculopharyngeal dystrophy (associated with French-Canadian ancestry)
 Kearns-Sayre (cardiac conduction defects and pigmentary retinopathy)
 idiopathic

Which of the following is a test for eccentric fixation?

visioscopy

Goldmann perimetry

Titmus fly

Worth 4-dot

Your answer was INCORRECT


Explanation
Visioscopy is a test for eccentric fixation using a direct ophthalmoscope fitted with a reticule
target. The patient is asked to fixate on the centre of the target. The location of the anatomical
centre of the fovea is measured on the reticule scale and provides a measure of the degree of
eccentric fixation.

A 73-year-old woman presents to eye casualty complaining of painless decreased vision in her left
eye, worsening over the past 6 hours. She denies any other systemic problems and has no other
medical history. Blood pressure is 142/90. Ophthalmic examination reveals a visual acuity of 6/6
right and counting fingers left. Ophthalmic examination reveals a normal right eye, a relative
afferent pupillary defect OS, and the left fundus reveals a haemorrhagic, swollen optic disc.

Which of the following would be the most appropriate next step?

if there is no history of jaw claudication, initiate aspirin 300mg and refer to stroke team

arrange a carotid doppler

obtain an ESR and C-reactive protein

admit for a computed tomography (CT) scan

Your answer was INCORRECT


Explanation
The history and the fundus findings are consistent with a diagnosis of anterior ischemic optic
neuropathy (AION), a condition presenting with acute painless visual loss and typically with optic
disc edema and pallor. Anyone with AION should be ruled out for arteritic AION (AAION)
secondary to giant cell arteritis (GCA), as this is a potentially fatal condition. The best initial way
to rule out GCA is with an erythrocyte sedimentation rate (ESR) and C-reactive protein (CRP), as
up to 20% of patients with AAION can present with no systemic symptoms (jaw claudication,
scalp pain, fever, malaise, etc.). ESR and CRP (when both are abnormal) have a 97% specificity
for AAION. The gold standard for diagnosing GCA is a temporal artery biopsy. Steroids can be
initiated before the biopsy (which should be performed within 1 week).

In Horner's syndrome, which sign on the affected side is consistent with the diagnosis?
retraction of upper lid
pupil dilation
vasoconstriction of arteries of facial skin
anhidrosis
exophthalmos

Your answer was INCORRECT


Explanation
The following signs are consistent with Horner's:
 ptosis
 miosis
 anhidrosis
 enophthalmos
 vasodilation of arteries of the facial skin

A blockage of the posterior inferior cerebellar artery may produce all of the following EXCEPT:
hoarsness of the voice and speech disturbance
ipsilateral sensory deficit to the face
contralateral cerebellar signs
contralateral sensory loss to the trunk and limbs

Your answer was INCORRECT


Explanation
This is the lateral medullary syndrome or Wallenberg syndrome. Cerebellar signs are ipsilateral.
Other options above are true.

Lateral medullary syndrome is characterised by deficits in:


 sympathetic pathway: ipsilateral central Horner's
 vestibular nuclei: vertigo and nystagmus
 5th nerve nucleus: ipsilateral loss of facial sensation
 cerebellum: ipsilateral cerebellar signs
 lateral spinothalamic tract: contralateral pain/temperature loss in trunk and limbs
 nucleus ambiguous: bulbar signs such as hoarseness and speech problems

A patient with multiple sclerosis could have all of the following EXCEPT:

retinal venous sheathing

amaurosis
skew deviation

bitemporal visual field deficit

Your answer was INCORRECT


Explanation
Bitemporal hemianopsia and skew deviation can occur if demyelination occurs in the optic chiasm
or supranuclear vertical gaze pathway, respectively. Uveitis, including iritis, pars planitis, and
retinal venous sheathing, has been reported in multiple sclerosis.

A meningioma arising in which location is MOST likely to cause optic disc oedema?

planum sphenoidale

sphenoid wing
optic nerve sheath

cavernous sinus

olfactory groove

Your answer was INCORRECT


Explanation
Optic nerve compression can cause unilateral disc oedema; this typically arises if the compression
of the nerve is within 1cm of the globe. Optic nerve compression greater than 1cm posterior to the
globe is less likely to cause disc oedema and will typically manifest as late optic disc pallor.

Which test object has four times the area and the same light intensity as the Goldmann II4e target?

V2a

II4c

II2e

III4e

Your answer was INCORRECT


Explanation
On the Goldmann perimeter, the test objects can be varied in both size and intensity by using
different filters.
Size: of the test object is represented by the Roman numeral (I-V). Each increment of the Roman
numeral doubles the radius (and quadruples the area) of the test object.
Light intensity: can be altered using different neutral density filters. Filters 1-4 are in increments
of 5 dB each. Filters a-e are in increments of 1 dB each.
Thus, the III4e test object will have twice the radius and four times the area of the II4e test object,
with the same light intensity.

Which is FALSE regarding the treatment of essential blepharospasm?

botulinum injections offer effective but temporary relief

stripping of orbicularis muscle fibers is a surgical alternative to botulinum injections

treatment of essential blepharospasm may exacerbate coexistent dry eye

facial nerve ablation is a surgical modality preferable to orbicularis myectomy

Your answer was INCORRECT


Explanation
Any treatment for blepharospasm is designed to decrease eyelid closure. Thus, dry eye will be
aggravated. Facial nerve ablation suffers from recurrence rates as high as 30% and is associated
with complications such as hemifacial paralysis. Consequently, its use has significantly decreased.
Botulinum injection is the initial treatment of choice followed by orbicularis myectomy.

All are true of the pituitary gland and optic chiasm EXCEPT:
macroadenomas tend to be non-secreting tumours
the anterior lobe of the pituitary secretes ACTH, GH, LH, FSH, TH and prolactin
an acidophil pituitary adenoma is likely to cause galactorrhea and infertility
a pre-fixed chiasm produces optic tract features

Your answer was INCORRECT


Explanation
Secreting microadenomas (75% of pituitary tumours) can be classified by their staining:
 chromophobes (50%): secrete prolactin causing infertility, impotence, amenorrhea,
galactorrhea
 acidophil (20%): secrete GH causing acromegaly in adults or gigantism in children
 basophils (5%): secrete ACTH causing Cushing's disease or FSH and TH (rare)

A right-handed 74-year-old man has difficulty performing simple arithmetic and cannot
distinguish the fingers on his hand. He cannot decipher left and right. He has asymmetrical
optokinetic nystagmus.

What field defect do you anticipate?

left superior homonymous quadrantanopia

left inferior homonymous quadrantanopia

right superior homonymous quadrantanopia

right inferior homonymous quadrantanopia

Your answer was INCORRECT


Explanation
The signs and symptoms constitute Gerstmann syndrome, which occurs with a dominant parietal
lobe lesion. As this patient is right-handed, his dominant lobe is the left, so the lesion is in the left
parietal lobe, causing a right inferior homonymous quadrantanopia.

Which is FALSE of the abducens nerve:


it has the longest intracranial course of any cranial nerve

its nucleus lies in the midpons

it emerges from the brainstem at the border of the pons and medulla near the midline
it passes through the cavernous sinus lying lateral to the ascending internal carotid artery

Your answer was INCORRECT


Explanation
The fourth cranial nerve (not the sixth) has the longest intracranial course of any cranial nerve. 
The sixth nerve has the longest intra-dural course.

Which is FALSE regarding Argyll-Robertson pupil?


caused by a lesion in the ventral midbrain
the speed of constriction and dilatation is normal
cocaine 4-10% has no effect on the pupil size
the pupil is miosed
occurs typically in elderly patients

Your answer was INCORRECT


Explanation
Argyll-Robertson pupil is due to a lesion in the dorsal midbrain. Specifically, the pretectal
interneurons to the Edinger Westphal nucleus are involved while sparing ventrally located
accommodative reflex neurons; hence the light-near dissociation seen.

Features of Argyll-Robertson pupils:


 small irregular pupils
 typically bilateral
 asymptomatic
 caused by lesion in dorsal midbrain
 light-near dissociation (responds better to near stimulus than light)
 respond poorly to pharmacological dilation

Causes of Argyll-Robertson-like include:


 syphilis (traditional cause)
 MS
 alcoholism
 trauma
 surgery
 aberrant 3rd nerve regeneration
 SLE
 HZO

A Goldmann visual field shows a bitemporal hemianopic central scotoma. What is the most likely
site of the lesion:

optic nerve

mid-chiasm
posterior chiasm

anterior chiasm

thalamus

Your answer was INCORRECT


Explanation
A bitemporal hemianopic central scotoma is produced by a lesion compressing the macular fibers
of the posterior chiasm.

Characteristic symptoms of vertebrobasilar insufficiency include all of the following EXCEPT:

ataxia

monocular loss of vision

vertigo

hemiparesis

dysarthria

Your answer was INCORRECT


Explanation
Monocular disturbance of vision is suggestive of carotid (anterior circulation) disease.
Vertebrobasilar insufficiency typically causes binocular visual blurring, homonymous field defects
or oculomotor symptoms.

A patient with cortical blindness is LEAST likely to have which of the following findings:

visual hallucinations

nystagmus

intact pupillary response

denial of blindness

Your answer was INCORRECT


Explanation
Nystagmus is not a feature of bilateral cortical damage.

Denial of blindness (Anton syndrome) can occur in cortical blindness. Riddoch syndrome may
also occur, where objects in motion can be seen or appreciated but not static objects.
A 68-year-old man has an episode of sudden monocular loss of vision like a shutter coming down
that lasts for 10 minutes before his vision returns to normal.

What is the most likely cause?


embolus

temporal lobe seizure

vertebro-basilar insufficiency

migraine

Your answer was INCORRECT


Explanation
The history is most suggestive of amaurosis fugax, which is caused by temporary obstruction to
the retinal arterial circulation (e.g by an embolus). A monocular visual deficit suggests the anterior
circulation. The posterior circulation (vertibro-basilar insufficiency) tends to produce bulbar and
cerebellar signs such as dysarthria, ataxia, and drop attacks. A temporal lobe seizure is associated
with formed visual hallucinations. Migraine is certainly in the differential of this case and can
produce both positive (scintillating phenomenon) and negative (visual deficits) symptoms, but it is
less likely given the age of the patient, monocularity and suddenness of onset.

This question came in the FRCS (Glasgow) Part 2 exam in October 2014.

Which is FALSE regarding papilloedema?

symptoms can include transient visual loss and diplopia

disc hyperaemia is an early sign

loss of venous pulsation is a specific sign

static perimetry often shows defects when kinetic perimetry is normal

enlargement of the blind spot is the most common field defect on kinetic perimetry

Your answer was INCORRECT


Explanation
Although loss of spontaneous venous pulsation is an early sign of papilloedema, approximately
20% of normal patients lack venous pulsations, so it is not a specific sign.

The most common kinetic visual field finding in papilleodema is an enlarged blind spot. Static
automated perimetry can often pick up early defects such as nasal steps or general reduction in
sensitivity when kinetic perimetry is normal.
A patient with thyroid eye disease has progressive loss of visual field and colour vision. Which of
the following is NOT an appropriate treatment:

optic nerve sheath decompression

surgical decompression of the orbit

steroids

radiotherapy

Your answer was INCORRECT


Explanation
Visual field loss from thyroid eye disease is usually the result of compression of the optic nerve
from soft tissue swelling and enlargement of the extraocular muscles. Treatment options include
radiotherapy, corticosteroids, and surgical decompression.

Optic nerve sheath decompression does not relieve the orbital congestion and would not be
effective in this condition.

Which of the following is the most appropriate surgical treatment for acquired sixth nerve paresis
with poor residual lateral rectus function:

Harada-Ito

Botulinum toxin to the medial rectus

Jensen's procedure

Knapp's procedure

Medial rectus recession

Your answer was INCORRECT


Explanation
The most widely accepted approach in sixth nerve palsy is a muscle-splitting operation utilising
half of the superior and inferior rectus muscles in a transposition laterally (Jensen's procedure).

A patient has complete right ptosis. On extraocular movement testing, there is significant
limitation of right adduction, elevation and depression. Left eye movements are full. A complete
neurological assessment is performed and shows left hemi-ataxia with incoordination.

What is the diagnosis?


Nothnagel syndrome
Wallenberg syndrome
Millard-Gubler syndrome
Claude syndrome
Weber syndrome
Your answer was INCORRECT
Explanation
The features described above suggest Claude syndrome.

Claude syndrome is characterised by: 1

 ipsilateral 3rd nerve palsy


 contralateral cerebellar ataxia
Weber syndrome is characterised by:
 ipsilateral 3rd nerve
 contralateral hemi-paresis
Nothnagel syndrome is characterised by:
 ipsilateral 3rd nerve
 ipsilateral cerebellar ataxia
Benedikt syndrome is characterised by: 2

 ipsilateral 3rd nerve


 contralateral cerebellar ataxia with intention tremor
 contralateral hemiparesis with hyperactive tendon reflexes

1. Reference for Claude syndrome


2. Reference for Benedikt syndrome
3. General reference for 3rd nerve syndromes

Which one of the following statements about optic nerve drusen is FALSE?

usually of no clinical consequence

may cause a relative afferent pupillary defect

associated visual field defects may resemble those of glaucoma

more common in African Americans and Asians

usually bilateral

Your answer was INCORRECT


Explanation
Optic disc drusen occur almost exclusively in white individuals. It is shown above in an
autofluorescence image from the Heidelberg OCT machine.

While optic disc drusen are usually clinically silent, they can occasionally cause visual field
defects resembling glaucoma, and can even cause an RAPD, though visual acuity is typically
unaffected

A 37-year-old woman develops a left-sided facial weakness. She has impairment of taste in the
anterior two thirds of the tongue, and complains that sounds are much louder than usual on that
side. There is no disturbance of balance or nystagmus evident. Tear production is normal in both
eyes.

Where is the lesion affecting her seventh cranial nerve most likely to be:

in the left parotid gland

proximal to the left geniculate ganglion

at the left cerebellopontine angle

at the left stylomastoid foramen


in the left facial canal

Your answer was INCORRECT


Explanation
Generally speaking, the more proximal a lesion, the greater the number of functions affected. A
distal lesion (outside the stylomastoid foramen) may only affect motor function. Lesions in the
facial canal can affect taste to the anterior two thirds of the tongue, more proximally in the canal
may produce hyperacusis due to involvement of stapedius muscle. Abnormalities in tear
production can occur with lesions of the geniculate ganglion. Cerebellopontine angle lesions
usually produce eighth nerve disturbance, in addition.

A 66-year-old retired labourer presents with a three-month history of proximal weakness and
diplopia, which is worse at night. He has noticed some difficulty with swallowing liquids. On
examination, he has a bilateral ptosis and some weakness of the shoulders and hips. He becomes
tired toward the end of the examination, and his limb weakness becomes more pronounced.

Where is the most likely site of his pathology:

muscle spindle cells

motor neuron ganglia

the presynaptic membrane

anterior horn cells

the postsynaptic membrane

Your answer was INCORRECT


Explanation
Fatigability of muscle groups in conjunction with extraocular weakness and dysphagia, strongly
suggests a neuromuscular junction disorder. This is more likely to be the postsynaptic membrane
(myasthenia gravis), due to antibodies against acetylcholine receptors. Lambert Eaton (LEMS)
(presynaptic) patients tend to get stronger with repetitive stimulation. Myasthenia gravis is also
more common than LEMS. Motor neuron disease is not variable in its course and it typically
spares the extraocular muscles.

A patient presents to A&E with acute right hemiplegia and right homonymous hemianopia
developing 2 hours previously. What is the most appropriate imaging modality to arrange for this
patient?
CT

CT angiogram

MR angiogram

MRI
Your answer was INCORRECT
Explanation
CT scans are quicker to obtain than MRI and more readily available in the emergency setting.
They help to exclude intracranial haemorrhage as a cause for stroke, and can allow a rapid referral
to the stroke team for thrombolysis as necessary.

A patient is asked to protrude his tongue, and it is seen to be deviating to the right. This involves
damage to the:
right vagus nerve

left hypoglossal nerve

left genioglossus muscle

right hypoglossal nerve

right glossopharyngeal nerve

Your answer was INCORRECT


Explanation
Damage to the right hypoglossal nerve will cause unilateral wasting of the tongue and ipsilateral
deviation of the tongue to the right side.
A 29-year-old obese woman is seen for a routine optician examination. The examination is
entirely normal with the exception of bilaterally elevated discs with indistinct margins. A pre-
injection photo from fluorescein angiography is shown above.

Which one of the following statements about this patient is TRUE?


the condition is typically associated with mild to moderate visual loss
the clinical and histologic findings reflect axoplasmic stasis and congestion
there may be an associated arcuate field defect
inflammatory markers are typically mildly deranged

Your answer was INCORRECT


Explanation
The presence of bilateral indistinct disc margins raises the possibility of papilloedema, especially
in young, obese females in whom idiopathic intracranial hypertension is recognised. However the
fundus photo in this case demonstrates distinct autofluorescent spots on the disc which is
consistent with optic disc drusen. Typically, visual acuity is normal, although there may be
associated field defects such as arcuate scotomas and in rare cases an RAPD.

Axoplasmic stasis is a feature of true papilloedema, rather than pseudopapilledema, which is


present here.

Causes of autofluorescence of the optic disc:


 optic disc drusen
 astrocytic hamartomas

What finding in a child with isolated abduction deficit most strongly argues for the diagnosis of
Duane's syndrome rather than a congenital sixth nerve palsy?
involvement of the left eye only
normal adduction
inability to fully abduct the eye volitionally
orthophoria in primary gaze
normal abduction on oculocephalic rotational testing

Your answer was INCORRECT


Explanation
When the diagnosis of Duane's Type 1 is entertained, the differential of congenital sixth nerve
palsy must be considered. It can sometimes be difficult to differentiate the two but important signs
are:
 narrowing of fissure in adduction, and widening in abduction: only seen in Duane's
 in Duane's, patients are often orthophoric or only minimally esotropic by comparison to
the degree of restriction of abduction, while in sixth nerve palsy the esotropia is larger
Note that oculocephalic responses are restricted in both Duane's and sixth nerve palsy. Also, note
that while the left eye is more commonly involved in Duane's syndrome, this does not help in
distinguishing this syndrome from a congenital sixth nerve palsy.

Which of the following statements about morning glory disc anomaly is FALSE?
the condition is unilateral
retinal vessels originating from the periphery of the disc is a classic finding
patients with this condition are at risk for retinal detachment
central visual acuity is often normal
an RAPD may be present

Your answer was INCORRECT


Explanation
Morning glory disc is characterised by:
 unilateral
 dense amblyopia
 RAPD may be present
 excavated optic nerve head
 overlying glial proliferation
 retinal vessels originating from the periphery of disc radiating out
 surrounding retina often thrown into folds
 associated with retinoschisis and retinal detachment

An 82-year-old woman experienced three 10-minute episodes of transient visual disturbance in her
right eye over the past 1 week. Her eye examination is normal apart from moderate nuclear
sclerosis.

Which sign on examination would elicit the most prompt attention to prevent permanent visual
loss?
blood pressure of 180/95

a cardiac murmur

right carotid bruit

scalp tenderness

Your answer was INCORRECT


Explanation
Temporal arteritis must be considered in anyone with transient visual loss who is over 55 years of
age. The presence of scalp tenderness should raise suspicions further. Permanent visual loss
caused by ischemic optic neuropathy could occur at any moment, and prompt treatment with
corticosteroids could be preventative.

While carotid bruits, cardiac murmurs and systemic hypertension are cardiovascular risks for non-
arteritic amaurosis fugax and should be looked into further in due course, they are much less
urgent in the acute setting

Which of the following conditions is LEAST likely to be confused with essential blepharospasm?

retained subtarsal foreign body

blepharitis with marginal keratitis

severe dry eye

hemifacial spasm

Your answer was INCORRECT


Explanation
Reflex blepharospasm can be caused by a number of disorders, including severe dry eye,
intraocular inflammation, and foreign bodies.

Hemifacial spasm is clinically distinct from essential blepharospasm in that it is unilateral, and
multiple facial muscles are involved.

Which statement is TRUE regarding treatment of myasthenia gravis?

plasma electrophoresis is useful in long-term disease control

azathioprine is useful in long-term control

thymectomy is most likely to induce remission in immunoglobulin negative disease

steroid treatment is contra-indicated in acute myesthenic crisis

Your answer was INCORRECT


Explanation
Plasmaphoresis (like intravenous immunoglobulin) is used in acute (not chronic) disease, as a tool
for crisis management by reducing the circulating load of antibodies. Steroid treatment is also very
useful in acute crises, with replacement of steroid by immunosuppresants such as azathioprine for
long-term control. Thymectomy may help to induce disease remission. This occurs more
frequently in young patients with a short duration of disease, hyperplastic thymus, more severe
symptoms, and a high antibody titer.

This question appeared in the 2014 FRCOphth Part 2.

What proportion of patients with myasthenia gravis will develop Graves' disease?

25%

10%

20%

5%

1%

Your answer was INCORRECT


Explanation
5% of MG patients develop Graves disease, so the condition should be screened for during work-
up. A tiny fraction of patients with Graves' disease will develop myasthenia gravis as well.

A 62-year-old man presents to A&E with acute headache. On examination, anisocoria is noted.
There is no RAPD and ocular movements are full but there is a mild right ptosis. Instillation of
cocaine 4% fails to dilate the smaller right pupil.

The most important urgent investigation is:

MRI brain

serum VDRL and FTA-ABS

CT head and chest

carotid angiography studies

Your answer was INCORRECT


Explanation
Painful Horner's is an acute dissection of the internal carotid artery until proven otherwise. The
diagnosis of exclusion then becomes migraine if the angiogram is normal.
A patient has variable ophthalmoplegia and proximal weakness of the arms and legs. He has a
diagnosis of small cell lung cancer.

Which test is most likely to confirm the diagnosis?


anti-dsDNA
anti-ACh receptor antibodies
anti-voltage-gated calcium channel antibodies
anti-GQ1b antibodies

Your answer was INCORRECT


Explanation
The clinical features suggest Lambert-Eaton syndrome, which can be confirmed by positive anti-
voltage-gated calcium channel antibodies.

Lambert-Eaton syndrome is characterised by:


 autoimmune neuromuscular junction disorder
 paraneoplastic syndrome in 60% (especially small cell lung cancer)
 proximal weakness of arms and legs
 ophthalmoplegia (though less common than myasthenia gravis)
 dry eye
 anti-voltage gated calcium channel antibodies

Note: ophthalmoplegia in Lambert-Eaton syndrome may improve with repeat testing, unlike
myasthenia where there is fatigueability.

Hypertropia associated with an ipsilateral adduction weakness is MOST suggestive of:


decompensated congenital fourth nerve palsy
progressive supranuclear palsy
one-and-a-half syndrome
dorsal midbrain syndrome
skew deviation with internuclear ophthalmoplegia

Your answer was INCORRECT


Explanation
Skew deviation is, by definition, a vertical misalignment of gaze that cannot be assigned to a
single nerve or muscle weakness. In the presence of an INO, the hypertropic eye is often on the
same side as the adduction deficit (medial rectus dysfunction). Fourth nerve palsy does not
produce difficulty with adduction. The one-and-a-half syndrome results in complete horizontal
paralysis of one eye and loss of adduction in the other eye. Progressive supranuclear palsy is a
disorder of movement that does not produce a resting vertical gaze deviation, but produces smaller
and slower saccades especially of downgaze. Dorsal midbrain syndrome (Parinaud's syndrome)
does not feature an adduction deficit but can cause a supranuclear upgaze palsy.
INO is characterised by the triad of:
 failure of ipslateral adduction
 ataxic nystagmus of abducting contralateral eye
 normal convergence
However, other features can include:
 slowing of saccades in adducting eye
 horizontal nystagmus of adducting eye
 asymmetric OKN response when drum rotated towards MLF lesion
 vestibulo-ocular reflex impairment
 impaired convergence (for Cogan's anterior INO)
 manifest exotropia (for bilateral INO)
 vertical nystagmus
 impaired vertical pursuit
 impaired vertical vestibulo-ocular reflex
 difficulty maintaining upgaze

Which of the following nerves travel through the lateral wall of the cavernous sinus at the level of
the pituitary fossa:

6th cranial nerve

3rd, 4th, 5th and 6th cranial nerves

3rd and 4th cranial nerves

3rd, 4th and 5th cranial nerves

4th, 5th and 6th cranial nerves

Your answer was INCORRECT


Explanation
The 3rd, 4th and 5th (V1 and V2) cranial nerves travel through the lateral wall of the cavernous
sinus. The 6th nerve travels through the cavernous sinus but goes through it rather than involving
the lateral wall.

A 24-year-old rugby player complains of intermittent diplopia since a rugby tackle 2 weeks ago.
On examination, he has a right hypertropia worse on left gaze and improved by left tilt. Cover-
uncover testing shows no shift but alternate cover test shows a deviation neutralized with 13 PD
base down prism in front of his right eye.

What can this patient be told about his condition?

the injury from the accident is temporary and will resolve with observation

the accident has caused damage to nerves bilaterally

this condition has been present for many years but has just recently been unmasked

his symptoms and findings do not correspond to any organic neurologic condition

Your answer was INCORRECT


Explanation
Vertical fusion amplitudes are useful for distinguishing congenital from acquired fourth nerve
palsies. Normal vertical fusion amplitudes are 3 to 5 PD. In congenital (longstanding) IV palsy,
patient can develop amplitudes of 10 to 25 PD. This patient has a right hyperphoria of 12 PD; this
means that the vertical fusion amplitude is at least 12 PD, indicating a long-standing deviation.
Although trauma is the most common cause of acquired IV palsy, it is also (often coincidentally)
the trigger that allows congenital palsies to be manifest. In addition to increased vertical fusion
amplitudes, these patients may have a head tilt to the contralateral side to reduce the hypertropia.
Old photographs may be helpful in demonstrating this feature.

What proportion of patients with myasthenia gravis present with ocular findings only?

75%

10%

20%

50%

5%

Your answer was INCORRECT


Explanation
75% of all myasthenics will have eye findings at presentation, but only 33% to 50% will have
ocular myasthenia only at presentation. Up to 90% of MG patients will develop ocular symptoms
at some point during the course of the disease. Ptosis is the most common feature.

The most common cause of an acquired, non-traumatic sixth nerve palsy in childhood is:

CNS tumour

idiopathic intracranial hypertension

demyelination

post-viral

Your answer was INCORRECT


Explanation
Sixth nerve palsy in a child is a common post-viral syndrome and typically resolves entirely of its
own accord. Nonetheless, neuroimaging should be arranged to exclude intracranial pathology. The
second most common aetiology of childhood abducens palsy is increased intracranial pressure
(either a CNS tumour or idiopathic intracranial hypertension)
Which is FALSE regarding the visual evoked potential (VEP)?

VEP cannot reliably distinguish optic neuropathy from retinal disorders

the stimulus may consist of either a flash of white light or a pattern, presented either transiently or
continuously by pattern reversal

the two crucial parameters used for functional evaluation include the height of the first positive or
upward wave (amplitude) and the time between stimulus presentation and the appearance of this
wave (latency)

uses of the flash VEP include visual acuity assessment in preverbal children, assessment of
optic nerve function in suspected multiple sclerosis, and reliable establishment of factitious
visual loss

the VEP is an electrical signal that must be extracted from the simultaneously generated
electroencephalogram (EEG)

Your answer was INCORRECT


Explanation
A pattern VEP (as opposed to a flash VEP) is required for visual acuity assessment in preverbal
children. Although the VEP is useful in establishing factitious visual loss, its reliability is limited
by the fact that patients can produce false readings by using accommodation to fog their vision.

Abnormalities in VEP latency and amplitude have been reported in various maculopathies and
retinopathies, and, therefore, these features cannot distinguish optic neuropathy from retinal
disorders with complete reliability.

A 44-year-old Chinese man presents with diplopia. On examination, there is a right abduction
deficit, right facial hypoesthesia and right neuro-sensory hearing loss.

This picture is most suggestive of:

nasopharyngeal carcinoma

diabetic vascular neuropathies

chordoma

aneurysm

meningioma

Your answer was INCORRECT


Explanation
Nasopharyngeal carcinoma can involve numerous cranial nerves because of its proximity to the
pre-pontine basal cistern. Most frequently, the trigeminal nerve is involved, causing facial
hypoesthesia or facial pain. The abducens nerve is the second most common. Cerebellopontine
angle involvement can also occur with 7th and 8th nerve involvement. The hallmark of
nasopharyngeal carcinoma is its propensity to involve multiple cranial nerves non-contiguously.
Nasopharyngeal carcinoma is common in Chinese men.

The muscle responsible for aggravation of the right hypertropia seen upon right head tilt with a
right superior oblique paresis is the:
left inferior rectus

right inferior rectus

right superior oblique

right inferior oblique

right superior rectus

Your answer was INCORRECT


Explanation
Right head tilt stimulates intorsion of the right eye to keep the visual field at the proper angle. The
intorters are the superior rectus and the superior oblique. In the setting of a superior oblique palsy,
the vertical, elevating force generated by the superior rectus is not effectively balanced by the
depressing force of the superior oblique, so the eye goes up.

Which of the following statements about the lateral geniculate body is FALSE:

M-cells are contained in layers 3-6

crossed fibers project to layers 1, 4, and 6

fibers are rotated 90 degrees

P-cells sub-serve color vision

Your answer was INCORRECT


Explanation
M-cells are contained in layers 1 and 2. Other statements are true.

A 76-year-old lady presents with a left third nerve palsy, a right hemi-tremor and right ataxia and
partial weakness.

What is the most likely affected anatomical site?

superior cerebellar peduncle

upper pons

ventral midbrain near the cerebral peduncle


superior colliculus

red nucleus

Your answer was INCORRECT


Explanation
The features described are consistent with Benedikt's syndrome, which affects the third nerve
fasciculus near the red nucleus, causing an ipsilateral 3rd nerve palsy with a contralateral tremor
and ataxia with partial hemiparesis. 

Summary of third nerve fasciculus syndromes:

Benedikt's syndrome is characterised by: 1

 ipsilateral CN III palsy


 contralateral cerebellar ataxia with intention tremor
 contralateral hemiparesis with hyperactive tendon reflexes
Claude syndrome is characterised by: 2

 ipsilateral 3rd nerve palsy


 contralateral cerebellar ataxia
Weber syndrome is characterised by:
 ipsilateral 3rd nerve
 contralateral hemi-paresis
Nothnagel syndrome is characterised by:
 ipsilateral 3rd nerve
 ipsilateral cerebellar ataxia

1. Reference for Benedikt syndrome


2. Reference for Claude syndrome
3. General reference for 3rd nerve syndromes

The most common location for a cerebral aneurysm associated with acute third nerve palsy is:

the junction of posterior communicating artery and the internal carotid artery

the junction of posterior communicating and posterior cerebral arteries

the junction of the posterior cerebral artery and the internal carotid artery

the junction of the vertebral and superior cerebellar arteries

the junction of the internal carotid artery and the anterior communicating artery

Your answer was INCORRECT


Explanation

Figure: Anatomy of the Circle of Willis in relation to the chiasm and the third cranial nerve.

The most common location for a cerebral aneurysm with third nerve palsy is the junction of the
posterior communicating artery and the internal carotid artery.

Which statement is FALSE regarding optic nerve hypoplasia?


it can be unilateral or bilateral
it is linked with maternal diabetes
the retinal blood vessels are of normal caliber
the visual acuity is almost always impaired

Your answer was INCORRECT


Explanation
Figure: Optic nerve hypoplasia

The visual acuity in ONH may be normal. Optic nerve hypoplasia is more common in children of
diabetic mothers.

Characteristics of optic nerve hypoplasia:


 a relatively common aetiology of childhood blindness
 presentation variable: VA may be normal
 incidence 6:100,000
 decreased number of nerve axons
 disc is small and gray or pale
 double ring sign
 retinal vessels may be tortuous or anomalous but are of normal calibre

Which of the following clinical tests is BEST for differentiating a congenital from an acquired
fourth nerve palsy:

distance and near prismatic measurements

Park's three-step test

vertical fusional amplitude measurements

the Lancaster red-green test

double Maddox rod testing

Your answer was INCORRECT


Explanation
The three-step test is useful for diagnosis of a fourth nerve palsy but it does not differentiate
between congenital and acquired causes. Large vertical fusional amplitudes (over 5 prism diopters,
while 2-3 prism dioptres is normal) and facial asymmetry from childhood head-tilting suggest a
decompensated congenital lesion.

The double Maddox rod test is useful for measuring the degree of excyclotorsion, which can help
to discriminate unilateral from a bilateral fourth nerve palsy.

Which of the following statements about juvenile idiopathic arthritis is FALSE?


the uveitis is frequently discovered incidentally

surgery for band keratopathy may be necessary for photophobia

cataract surgery with intraocular lens implantation is the procedure of choice for visually
significant cataract

visual loss from amblyopia is a frequent concern

Your answer was INCORRECT


Explanation
Generally, IOL insertion is best avoided in JIA patients because of the risk of aggressive
postoperative inflammation, which can be further aggravated by an IOL. Cycitic membranes can
ensue, with hypotony and phthisis.

More recently, encouraging results have been published regarding IOL insertion in these patients,
but it remains uncommon practice.

Which is FALSE regarding multiple sclerosis?


there is a female preponderance

the most common cranial mononeuropathy seen in MS is an isolated abducens nerve palsy

5% to 10% of patients with MS have posterior uveitis, including pars planitis or retinal
periphlebitis

from the Optic Nerve Treatment Trial 50% of all patients with optic neuritis develop MS by 15
years of follow-up, if the MRI findings are not taken into account

MS is more common in relatives of patients because there is clearly a genetic component to the
disease

Your answer was INCORRECT


Explanation
The most common cranial mononeuropathy in multiple sclerosis (MS) is optic neuritis, and it is
the one of the presenting features of MS in approximately 25% of patients. The most frequently
affected motor nerve is the sixth nerve. Other statements above are true.
Acquired optic nerve disease tends to cause:
deuteranopia
red-green defects
blue-yellow defects
symmetric depreciation of colours

Your answer was INCORRECT


Explanation
 red-green defects: acquired optic nerve disease (except glaucoma and autosomal
dominant optic atrophy)
 blue-yellow defects: acquired retinal disease (except cone dystrophy and Stargardt's)

A Goldmann visual field test shows a binasal hemianopia. What is the most likely site of
pathology:
thalamus
optic nerve
anterior chiasm
mid-chiasm
posterior chiasm

Your answer was INCORRECT


Explanation
A binasal hemianopsia is almost never due to chiasmal compression and is usually the result of
glaucoma.

A binasal field defect may be due to:


 Retina
o atypical RP
o schisis
o vascular occlusion (2 vessels)
o juxtapapillary retinochoroiditis
o myopia with peripapillary atrophy
 ON Head
o glaucoma
o disc drusen
o chronic papilloedema
 Chiasm (rare)
o tumor
o aneurysm

Regarding aneurysmal third nerve palsy, which is FALSE:


partial third nerve palsy is typical on presentation
the most common location for an aneurysmal 3rd nerve palsy is the junction of the posterior
communicating artery and the internal carotid artery
lid elevation on adduction is a recognised feature
pain is a helpful sign to discriminate from a microvascular third nerve
pupil involvement occurs in over 90% of cases

Your answer was INCORRECT


Explanation
Factors suggesting a surgical third nerve (as opposed to microvascular):
 young patient
 no cardiovascular risks
 pupil involvement (90% versus only 20% in medical cases)
 progression of pupil involvement
 incomplete palsy with progression
 multiple cranial nerve palsies
 aberrant regeneration
 no recovery over time

Note: the presence of pain is not a particularly helpful feature to discriminate medical from
surgical causes of a third nerve because medical causes such as diabetes, migraine and Tolosa
Hunt may be associated with pain.

Note: lid elevation on adduction may occur with aneurysmal third nerve palsy due to aberrant
regeneration. Aberrant regeneration can occur with any compressive cause of a third nerve,
including trauma, tumours and aneurysms.

A patient with Arnold-Chiari malformation has significant, symptomatic down-beat nystagmus.


Which of the following is MOST likely to benefit?

psychotherapy

orthoptic excercises

basilar craniectomy

carbamazepine

clonazepam

Your answer was INCORRECT


Explanation
Clonazepam has been used in the treatment of downbeat nystagmus. By contrast, carbamazepine
toxicity has been associated with downbeat nystagmus!

All of the following features are consistent with a temporal lobe lesion EXCEPT:
optokinetic nystagmus asymmetry

pie-in-the-sky field defect

incongruity of visual field deficits

formed visual hallucinations

seizures

Your answer was INCORRECT


Explanation
Optokinetic nystagmus abnormalities indicate lesions of the parieto-occipital (slow-phase pursuit
abnormalities) or the frontal lobe (fast-phase recovery saccade abnormalities). High congruity of
visual field deficits indicates a lesion in the occipital lobe. Both partial complex seizures and
formed visual hallucinations may be seen with temporal lobe lesions. Inferior nerve fibers from
the superior retina course anteriorly in Meyer's loop through the temporal lobe; therefore, lesions
affecting Meyer's loop will result in pie-in-the-sky (superior homonymous quadranopic defects)
contralateral to the lesion.

A 36-year-old lady attends clinic complaining of headache and transient visual obscuration. On
examination, there is bilateral disc oedema.

What should be your next step?

automated perimetry

measurement of sedimentation rate

measurement of blood pressure

brain imaging
lumbar puncture

Your answer was INCORRECT


Explanation
Bilateral disc oedema and headache may be caused by several conditions, and in this demographic
idiopathic intracranial hypertension should be considered high on the list. However, systemic
hypertension is another differential and should be the first to be excluded because it is a
straightforward test. After checking blood pressure, neuroimaging should be obtained
immediately, followed by diagnostic LP as appropriate.

In an infant with nystagmus, which of the following is LEAST likely to be a cause?


achromatopsia
Leber's congenital amaurosis
congenital stationary night blindness
ocular albinism

Your answer was INCORRECT


Explanation
The differential of congenital nystagmus includes:
 albinism
 Leber's congenital amaurosis
 achromatopsia or rod monochromatism
 optic nerve hypoplasia
 aniridia

Achromatopsia or rod monochromatism is:


 autosomal recessive (or rarely a milder X-linked form)
 total lack of cones
 6/60 vision
 photophobia
 nystagmus
 fundus normal in infancy

Which statement is FALSE regarding visual-evoked potentials?

the P100 is the largest and most reproducible component

demyelinating optic neuropathy produces a delay in P100

the P100 is predominantly a macula response

the latency of the P100 correlates with the visual acuity

Your answer was INCORRECT


Explanation
The amplitude (not the latency) of the P100 can be used as an objective measure of the visual
acuity. When the checked pattern is reduced to a degree where it cannot be resolved by the eye,
the cortical response subsides.

In acute demyelination, the VEP is usually undetectable. When the visual acuity has improved, the
VEP tends to show almost normal amplitudes but remains permanently delayed by at least 30ms.

Aberrant regeneration does NOT occur after injury to the oculomotor nerve with which one of the
following conditions?

trauma

tumour compression

aneurysm
ischaemia secondary to diabetes

Your answer was INCORRECT


Explanation
Trauma or compressive lesions can cause aberrant regeneration. Ischaemic neuropathy does not
result in aberrant regeneration.

Which of the following is NOT a universal finding in patients with idiopathic intracranial
hypertension?
normal CSF analysis
neuroimaging studies that fail to provide a cause of raised intra-cranial pressure
papilloedema
increased intracranial pressure

Your answer was INCORRECT


Explanation
Idiopathic intracranial hypertension is characterised by:
 increased intracranial pressure on lumbar puncture
 neuroimaging studies that fail to provide a cause of raised intra-cranial pressure (findings
that may be consistent with IIH include small ventricles and an empty sella)
 normal cerebrospinal fluid (CSF)
Papilloedema need not be present for the diagnosis.

In which of the following conditions is neuroimaging MOST likely to be normal?


Louis-Bar syndrome
Sturge-Weber syndrome
von Recklinghausen's disease
Bourneville's syndrome

Your answer was INCORRECT


Explanation
Louis-Bar syndrome (ataxia-telangiectasia) is not associated with any CNS abnormalities
detectable with neuroimaging. This is in contrast to the other phakomatosis syndromes supplied
above.

 von Recklinghausen's (NF-1): optic nerve glioma, meningiomas, schwannomas


 Bourneville's (tuberous sclerosis): calcified astrocytomas and cortical tubers
 Sturge-Weber's: intracranial calcification with pial angiomatosis
Features of ataxia telangiectasisa (Louis-Bar syndrome):
 autosomal recessive inheritance
 cerebellar dysfunction: ataxia, oculomotor apraxia, nystagmus
 telangiectasia of conjunctiva and the skin of nose, ears, elbows and knees
 delayed motor development
 immunodeficiency: recurrent respiratory infections
 premature graying of hair
 cafe-au-lait spots
 susceptability to cancer: lymphoma and leukaemia

A miotic pupil in which there is no dilatation to cocaine 4% or to hydroxyamphetamine 1%


suggests:
a Horner's of any order
first-order Horner's
third-order Horner's
second-order Horner's

Your answer was INCORRECT


Explanation
Cocaine 4% prevents reuptake of noadrenaline into the presynaptic terminal from the synaptic
cleft. When the sympathetic chain is intact, cocaine increases the duration that noadrenaline
remains in the synaptic cleft, causing pupillary dilation. In Horner's syndrome (first, second, and
third order), there is little activity at the cleft because of sympathetic disruption and hence no
dilation.

Hydroxyamphetamine 1% causes the release of noradrenaline from the presynaptic terminal. In


first- or second-order Horner's, this results in pupillary dilation. If the third-order neuron is
injured, it is unable to release noradrenaline and the pupil remains miotic.

The use of dilute adrenaline 1:1000 (or simply phenylephrine 1%) can also be useful in
distinguishing pre-ganglionic from post-ganglionic Horner's syndrome. Both adrenaline and
phenylephrine are direct agonists at the post-synaptic cleft. In a third-order Horner's there is
denervation hypersensitivity, which will cause dilation, while there is no response (or
comparatively less response) with a normal pupil, or a pupil that has a pre-ganglionic Horner's.

Which of the following is LEAST likely to cause cerebellopontine angle syndrome?

pinealoma

nasopharyngeal carcinoma

pontine glioma

clivus meningioma

acoustic neuroma

Your answer was INCORRECT


Explanation
Based on the location of the pineal gland, a pinealoma may cause Parinaud's dorsal midbrain
syndrome, but would not affect the cerebellopontine angle.

Causes of cerebellopontine angle syndrome:

 acoustic neuroma
 nasopharyngeal carcinoma
 clivus meningioma
 pontine glioma
 trigeminal nerve neuroma
 trauma (including basal skull fracture)

Which visual field defect is most likely to be associated with seesaw nystagmus?
congruous hemianopia
central scotoma
bitemporal hemianopia
incongruous hemianopia

Your answer was INCORRECT


Explanation
See-saw nystagmus results from third ventricle tumors or diencephalic lesions, which have a
propensity to compress the optic chiasm causing features of chiasmal syndrome, including
bitemporal hemianopia

Which of the following has the worst prognosis for vision and for survival?

Kjer syndrome

Wolfram syndrome

Leber's optic atrophy

Behr syndrome

Your answer was INCORRECT


Explanation
These conditions are all hereditary optic neuropathies. The most severe is Wolfram syndrome
(DIDMOAD syndrome) which is characterised by:

 AR inheritance
 severe optic atrophy
 diabetes insipidus
 diabetes mellitus
 ataxia
 seizures
 mental handicap
 short stature
 endocrine abnormalities

A 72-year-old man presents with anisocoria, with the left pupil larger than the right. After bilateral
installation of 4% cocaine the anisocoria increases in size. The patient is brought back the next
day, when the anisocoria increases in size following bilateral instillation of 1%
hydroxyamphetamine.

All of the following statements are true of this case EXCEPT:


occult lung cancer is a possible cause
associated findings may include unilateral hypotony or mild anterior chamber reaction
anisocoria is worse in low ambient illumination
no workup is necessary if the findings can be shown to be longstanding
the right pupil is the abnormal pupil

Your answer was INCORRECT


Explanation
Anisocoria aggravated by cocaine implies Horner's syndrome. Cocaine blocks reuptake of
catecholamines at the synapse (causing increased sympathetic activity and therefore mydriasis in a
normal pupil) but in Horner's syndrome, there is no native catecholamine present at the synapse.

Topical apraclonidine is an alternative to topical cocaine (which can be difficult to source) to


confirm Horner's syndrome. Apraclonidine is an alpha adrenergic agonist. It causes pupillary
dilation in the Horner's pupil due to denervation supersensitivity while producing a mild pupillary
constriction in the normal pupil presumably by down-regulating the norepinephrine release at the
synaptic cleft. A reversal of anisocoria after instilling two drops of 0.5% apraclonidine is
suggestive of Horner's syndrome.

Hydroxyamphetamine stimulates release of catecholamines from the presynaptic neuron at the


effector junction. If hydroxyamphetamine fails to dilate a Horner's pupil (as in the case described
above), the lesion must be post-ganglionic (third-order). Thus the findings in this case fit with
third order (not a second-order, pre-ganglionic lesion as would be caused by a Pancoast tumour).

Note: Although rare, in the acute phase of Horner's, the affected eye may be injected (lack of
sympathetic vaso-control), hypotonous (reduced aqueous secretion), and mildly inflamed
(breakdown of the blood-ocular barrier).

All are TRUE of a lesion affecting the left optic tract EXCEPT:
more likely to occur in a post-fixed than pre-fixed optic chiasm
right RAPD
incongruous right homonymous hemianopia
contains more crossed than uncrossed fibres

Your answer was INCORRECT


Explanation
An optic tract lesion is more likely in a pre-fixed chiasm (a chiasm anterior to the pituitary gland)
than a post-fixed chiasm (a chiasm posterior to the pituitary gland, where compression involves
the optic nerve and chiasmal junction, causing a juntional scotoma).

Note: a lesion of the optic tract can produce a contralateral RAPD, because there are more crossed
than uncrossed fibres.

Botulinum toxin injections are the treatment of choice in which of the following cases:

1. non resolving cranial nerve VI palsy


2. moderate angle distance exotropia
3. blepharospasm
4. Brown's syndrome
1, 2, 3 and 4
2 and 3
3 only
1, 2, 3

Your answer was INCORRECT


Explanation
Botox is the treatment of choice for most types of blepharospasm associated with dystonia.

While Botox is also used in the context of concomitant and paretic squints, it is generally used as a
temporising measure or as a diagnostic tool (for example, to test for post-operative diplopia) and is
not considered a primary treatment for these conditions. Surgery remains the treatment of choice.

A 40-year-old woman was referred for evaluation of ptosis and abnormal eye movements. Ocular
motility shows generalised ophthalmoplegia.

All of the following are recognised clinical associations with this history EXCEPT:

French-Canadian ancestry

sluggishly reactive pupils

polychromatic lenticular deposits

cardiac conduction defects

pigmentary retinopathy

Your answer was INCORRECT


Explanation
Ptosis and generalised ophthalmoplegia suggest either myasthenia or chronic progressive external
ophthalmoloplegia (CPEO). Neither of these conditions affect pupillary reactivity.
CPEO can be caused by:

 myotonic dystrophy (associated with polychromatic cataract)


 oculopharyngeal dystrophy (associated with French-Canadian ancestry)
 Kearns-Sayre (cardiac conduction defects and pigmentary retinopathy)
 idiopathic

Pupillary dilation may be the only sign of oculomotor nerve palsy in which of the following
disorders?

uncal herniation

diabetic microvascular disease

malignant hypertension

cerebral aneurysm

Your answer was INCORRECT


Explanation
Oculomotor nerve palsy usually includes ptosis, limitation of ocular motility, and pupillary
abnormalities. In uncal herniation and basilar meningitis, pupil dilation may be the only sign of
third nerve palsy. In the case of a cerebral aneurysm, the pupil is usually involved, along with
other functions of the third nerve. If the pupil is not involved, it is less likely that an aneurysm or
other surgical cause is to blame. Total oculomotor palsy with a spared pupil in elderly patients
usually suggests a microvascular aetiology.

Factors suggesting a surgical third nerve (as opposed to microvascular):

 young patient
 no cardiovascular risks
 pupil involvement (90% versus only 20% in medical cases)
 progression of pupil involvement
 incomplete palsy with progression
 multiple cranial nerve palsies
 aberrant regeneration
 no recovery over time

Note: the presence of pain is not a particularly helpful feature to discriminate medical from
surgical causes of a third nerve because medical causes such as diabetes, migraine and Tolosa
Hunt may be associated with pain.

A patient has left orbital exploration plus biopsy for a suspicious infiltrate on CT imaging.
Postoperatively, the patient is noted to have a widely dilated pupil and poor vision at near in the
left eye. He also complains of binocular diplopia. On motility testing, there is an inability to
elevate the eye when it is adducted.

What findings would you expect on the three-step test?


a left hypertropia worse in right gaze and left head tilt
a right hypertropia worse in left gaze and right head tilt
a left hypertropia worse in left gaze and right head tilt
a right hypertropia worse in right gaze and left head tilt
a right hypertropia worse in right gaze and right head tilt

Your answer was INCORRECT


Explanation
The clinical features in this case of mydriasis and accommodative paresis indicate damage to the
parasympathetic supply to the globe. These nerves travel with the nerve to the inferior oblique
before forming the short root of the ciliary ganglion. The motility findings in this case indicate a
probable inferior oblique palsy (inability to elevate the adducted eye). The three-step test should
show a left hypotropia (right hypertropia) worsened in right gaze and right head tilt.

A 52-year old lady has an acquired vertical diplopia with marked head tilt to the left. The right
hypertropia measures 20 prism dioptres in the primary position and is worse on looking to the left.

What is the most appropriate surgical procedure?


right superior rectus and right inferior oblique weakening
right superior oblique tuck
right inferior oblique weakening
right inferior oblique and left inferior rectus weakening

Your answer was INCORRECT


Explanation
The clinical history is in keeping with a large-angle vertical squint secondary to right superior
oblique weakness. A right superior oblique tuck or a right inferior oblique weakening are unlikely
on their own to correct such a large angle. The most appropriate management is combined right
inferior oblique weakening with left inferior rectus weakening. Note that weakening the inferior
oblique and superior rectus unilaterally is likely to cause a reduction in elevation so is not an
appropriate option.

The most common cause of permanent visual loss in patients with carotid-cavernous sinus fistulae
is:
neovascular glaucoma
corneal exposure
open-angle glaucoma
choroidal effusion
compressive optic neuropathy

Your answer was INCORRECT


Explanation
80% of patients with CCF develop ocular hypertension, 25% develop optic disc cupping and 20%
visual field defects. This is due to raised episcleral venous pressure causing a secondary open-
angle glaucoma.

A patient has normal vertical movements. On horizontal eye movements, he has no movement on
attempted left gaze. On right gaze, the right eye abducts with ataxic nystagmus while there is no
movement of the left eye.

Which statement about this condition is correct?


the left parapontine reticular formation is involved
both medial longitudinal fasciculi are involved
the right medial longitudinal fasciculus is involved
this is most likely a congenital defect of horizontal motility

Your answer was INCORRECT


Explanation
This patient has one-and-a-half syndrome. It is caused by a lesion of the PPRF and MLF on the
same side, thus causing an ipsilateral gaze palsy and an INO. The only residual movement is
contralateral abduction with ataxic nystagmus. The most common causes are demyelination (MS)
and ischaemia.

A patient has corneal clouding and optic atrophy with keratin sulphate accumulation.

What is the diagnosis?


Hunter syndrome
Sanfilippo syndrome
Maroteaux-Lamy syndrome
Morquio syndrome
Hurler-Scheie syndrome

Your answer was INCORRECT


Explanation
Morquio syndrome is part of the mucopolysaccharidoses group of enzyme deficiency disorders. It
results in excessive accumulation of keratin sulphate. This causes corneal clouding, optic atrophy,
severe bone deformity and aortic valve disease.

A 30-year-old woman has had migrainous headaches for several years. She recently developed
episodes of "flashing lights off to the right" that affect her peripheral vision. Her automated
perimetry suggests the possibility of an incongruous right homonymous hemianopia.

The next step would be:


cerebral MRI
sumatriptan
tangent screen
discontinue oral contraceptives
Your answer was INCORRECT
Explanation
Although migraine can cause persistent visual field deficit, it is unusual. Cerebral arteriovenous
malformations and tumours can mimic migraine and cause visual field loss. Therefore,
neuroimaging is essential. Oral contraceptives can exacerbate migraine, and sumatriptan is an
effective form of migraine treatment in about 80% of patients. Both of these measures would
apply only if the appropriate evaluation was normal. A tangent screen can be used if malingering
is suspected.

A patient suffers from left exotropia which is causing diplopia. On the cover test, when the left
eye is covered:

the right eye will move inward and subjectively the image on the right will disappear

the right eye will stay stationary and subjectively the image on the left will disappear

the right eye will stay stationary and subjectively the image on the right will disappear

the right eye will move inward and subjectively the image on the left will disappear

Your answer was INCORRECT


Explanation
The left eye is exotropic, while the right eye fixates, so the right eye will NOT move when the left
eye is covered.

 Exotropia causes crossed diplopia (right image comes from left eye and vice versa)
 Esotropia causes uncrossed diplopia (right image comes from right eye, etc)

The sixth cranial nerve:


arises from a nucleus closely related to the facial nerve
passes through the medial wall of the cavernous sinus
innervates the lacrimal gland
is involved in the light reflex

Your answer was INCORRECT


Explanation
The sixth cranial nerve nucleus is found medial to the facial nerve nucleus. It is a pure motor
nerve and supplies only the lateral rectus muscle.

In terms of its course, the nerve leaves the brainstem at the junction of the pons and medulla and
enters the subarchnoid space. It runs upward in the subarachnoid space between the pons and the
clivus, and then pierces the dura mater to run between the dura and the skull through Dorello's
canal. At the tip of the petrous temporal bone it makes a sharp turn forward to enter the cavernous
sinus. In the cavernous sinus it runs alongside just lateral to the internal carotid artery. It then
enters the orbit through the superior orbital fissure and innervates the lateral rectus muscle of the
eye.

A 59-year-old homeless man presents to A&E with sudden-onset diplopia and oscillopsia.
Examination reveals a stong alcohol odor to his breath, visual acuity 6/9 bilaterally, with bilateral
abduction deficits, and coarse binocular nystagmus.

Which intervention is most likely to be appropriate in this case?


intravenous chlordiazepoxide
intravenous glucose
intravenous naloxone
intravenous thiamine

Your answer was INCORRECT


Explanation
Acute thiamine deficiency (Wernicke's encephalopathy) can result in central scotomas as well as
ophthalmoplegia, primarily affecting cranial nerves III and VI. It can be precipitated in
nutritionally depleted alcoholics given intravenous glucose alone because of sudden consumption
of systemic thiamine stores.

Systemic associations with Duane's syndrome include all EXCEPT:


Goldenhar's syndrome

sensorineural hearing loss

spinal column anomalies

congenital absence of the pectoralis muscle

Your answer was INCORRECT


Explanation
Congenital absence of the pectoralis muscle is seen in Poland syndrome, which is a recognised
association of Moebuis syndrome.

Duane's retraction syndrome (DRS):


 unilateral or bilateral abnormality of horizontal gaze
 caused by co-contraction of medial and lateral recti
 retraction of the globe on attempted adduction
 upshooting or downshooting of the globe on adduction
 the left eye is affected more frequently than the right
 females are affected more frequently than males
 Type 1: abduction limitation greater than adduction, eso (most common)
 Type 2: adduction limitation greater than abduction, exo (least common)
 Type 3: abduction and adduction limited equally
 amblyopia in 10%
Associations with Duane's retraction syndrome:
 cataracts
 iris anomalies
 Marcus Gunn jaw winking
 microphthalmos
 crocodile tears
 Goldenhar's syndrome
 maternal thalidomide
 Klippel-Feil syndrome (congenital cervical vertebral fusion, congenital high scapula,
scoliosis, spina bifida, cleft palate)

An 80-year-old male presents with headache and fever, having just had a seizure and is noted to
have bilateral swollen optic discs.

What is the most likely diagnosis?

bilateral arteritic ischaemic optic neuropathy

sagittal sinus thrombosis

carotid cavernous fistula

pseudotumour cerebri

bilateral non-arteritic ischaemic optic neuropathy

Your answer was INCORRECT


Explanation
Venous sinus thrombosis may result from:

 skull or paranasal sinus infection


 dehydration
 severe illness
 oral contraceptive
 antiphospholipid syndrome
It presents with raised ICP, headache, fever, papilloedema and epilepsy.

A previously well 54-year-old man develops blurred vision and headache overnight. On
examination, he has normal movements of the right eye, but complete ophthalmoparesis of the left
eye. The left pupil is dilated. There is some sensory loss to light touch and pin prick over the left
eyebrow. The rest of the neurological examination is normal. Pulse is 80/min, blood pressure
130/87.

Which of these is the LEAST likely anatomical site to explain his problem:
brainstem and pons
left superior orbital fissure
left posterior communicating artery
left cavernous sinus
left internal carotid artery
Your answer was INCORRECT
Explanation
Loss of all eye movements unilaterally is unusual. It is unlikely to be due to neuromuscular causes
such as myasthenia gravis or Guillain-Barre Syndrome, as both eyes would be affected.
Involvement of the 3rd, 4th and 6th cranial nerves could produce the clinical picture above. All
these nerves arise in the brainstem/pons and pass through the cavernous sinus, where they may be
affected by aneurysm (of the internal carotid artery) or thrombosis. Infection at this site would be
another cause. All these nerves pass through the superior orbital fissure. An aneurysm of the
posterior communicating artery causes a 3rd nerve palsy.

Select the FALSE statement below. The optic nerve blood supply:
includes the central retinal vessels and their branches
includes the scleral vessels called the circle of Zinn-Haller
the majority of capillaries run external to the optic nerve
includes the pial vessels

Your answer was INCORRECT


Explanation
The majority of capillaries pierce the nerve and course within the nerve via the glial septae.

Where is the lesion located?


left anterior chiasm
left posterior chiasm
right anterior chiasm
right posterior chiasm

Your answer was INCORRECT


Explanation
This is a junctional scotoma. The lesion is compressing the right optic nerve and the right anterior
chiasm (producing a corresponding left superotemporal defect).

Which one of the following is NOT involved with vertical eye movements?
frontal eye fields
trochlear nucleus
interstitial nucleus of Cajal
paramedian pontine reticular formation

Your answer was INCORRECT


Explanation
The supranuclear control of vertical saccades originates in the frontal eye fields or in the superior
colliculus. They project to neurons in the rostral interstitial nucleus of the medial longitudinal
fasciculus (riMLF) and on to the nuclei of cranial nerve III and IV. The interstitial nucleus of
Cajal is involved with vertical pursuit control. The paramedian pontine reticular formation (PPRF)
controls horizontal eye movements

A patient is orthophoric in primary position with normal vertical eye movements. On attempted
right gaze, both eyes fail to move from primary position. Left gaze is normal.

A lesion in which of the following locations might explain these findings?


left paramedian pontine reticular formation
right sixth nerve fascicle
both medial longitudinal fasciculi
right sixth nerve nucleus

Your answer was INCORRECT


Explanation
The sixth nerve nucleus contains both sixth nerve axons and interneurons destined for the
contralateral medial rectus sub-nucleus via the medial longitudinal fasciculus. Thus, a lesion of the
sixth nerve nucleus can produce an ipsilateral gaze palsy. Left paramedian pontine reticular
formation damage would result in a left gaze palsy. Damage to both medial longitudinal fasciculi
would cause a bilateral internuclear ophthalmoplegia. A lesion of the right sixth nerve fascicle
would produce a right abduction deficit.

All of the following ocular findings are associated with acromegaly EXCEPT:

angioid streaks

optic atrophy

bitemporal hemianopia

pigmentary retinopathy

extra-ocular muscle enlargement

Your answer was INCORRECT


Explanation
Acromegaly is associated with:

 angioid streaks
 chiasmal syndrome
 retinopathy (hypertensive or diabetic)
 optic atrophy, papilloedema
 muscle enlargement

Which of the following statements about Horner's syndrome is FALSE?


the distribution of anhidrosis is helpful in locating the lesion

Horner's syndrome with coincident ipsilateral headache is indicative of spontaneous carotid


dissection, even in the setting of a normal carotid angiogram

a Pancoast tumour will typically cause a Horner's that does not dilate to cocaine but dilates to
hydroxyamphetamine 1%

4% cocaine will not dilate a Horner's pupil

Your answer was INCORRECT


Explanation
Horner's syndrome is characterised by ipsilateral partial ptosis and miosis. Anhidrosis may or may
not be present and can assist localisation:
 First-order lesions (CNS) cause ipsilateral anhidrosis of the entire body
 Second-order lesions (Pancoast's tumor, neck trauma) cause ipsilateral facial anhidrosis
 Third-order lesions cause anhidrosis only around the affected eye or none at all

Note: a painful Horner's syndrome may be caused by many disorders (neck trauma, migraine,
cluster headaches), but spontaneous dissection of the common carotid artery must be ruled out
with angiography or MRI/MRA.
 
Pharmacological Testing in Horners:
Topical Cocaine may be used to confirm Horner's syndrome in subtle cases. Cocaine blocks
reuptake of the neurotransmitter norepinephrine from the synaptic cleft and will cause dilation of
the pupil with intact sympathetic innervation. One hour after instillation of two drops of 10%
cocaine, the normal pupil dilates more than the Horner's pupil, thus increasing the degree of
anisocoria. It is becoming increasingly difficult to obtain cocaine eye drops due to increased
regulations.
Topical Apraclonidine is an alternative to topical cocaine to confirm Horner's syndrome.
Apraclonidine is an alpha adrenergic agonist. It causes pupillary dilation in the Horner's pupil due
to denervation supersensitivity while producing a mild pupillary constriction in the normal pupil
presumably by down-regulating the norepinephrine release at the synaptic cleft. A reversal of
anisocoria after instilling two drops of 0.5% apraclonidine is suggestive of Horner's syndrome.
Topical Hydroxyamphetamine is used to differentiate pre and postganglioninc Horner's.
Hydroxyamphetamine causes a release of norepinephrine from intact adrenergic nerve endings
causing pupillary dilation. One hour after instillation of 1% hydroxyamphetamine eye drops
dilation of both pupils indicate a lesion of the 1st or 2nd order neuron. If the smaller pupil fails to
dilate it indicates a lesion of the 3rd order or postganglionic neuron.

All of the following favour the diagnosis of benign essential blepharospasm over hemifacial
spasm EXCEPT:
no involvement of lower facial muscles along with orbicularis muscle

absence of abnormal movements during sleep

synchronous contractures of involved muscles

lack of response to neurosurgical decompression of the facial nerve


bilateral involvement

Your answer was INCORRECT


Explanation
Hemifacial spasm is rarely bilateral and is usually caused by vascular compression of the seventh
cranial nerve at the brainstem and can result in synchronous contractures of the orbicularis oculi
and other facial mimetic muscles

Each of the following statements is TRUE regarding Leber's hereditary optic neuropathy
EXCEPT:
affected family members are far more likely to be maternally related than paternally related
no specific test exists to confirm the suspected diagnosis
the disease is frequently associated with abnormalities of the X chromosome
a patient's brothers are more likely to be affected than sisters

Your answer was INCORRECT


Explanation
Genetic tests can confirm the diagnosis of LHON. Although Leber's is mitochondrially inherited, the
expression of the condition can be facilitated by certain additional genes on the X-chromosome. (Trick
question!)

LHON is characterised by:


 mitochondrial inheritance
 typically painless vision loss
 males more commonly affected (80% of cases)
 age of onset usually 10-30 years
 10% of female carriers are affected
 50% of male carriers are affected
 generally sequential bilateral optic nerve involvement
 smoking and alcohol appear to be risk factors for vision loss
 acute: optic disc swelling, hyperaemia, dilated telangiecactic vessels but no leak on FFA
 chronic: optic atrophy
 a small percentage have partial or complete recovery of vision
 type of mutation most important prognostic factor for recovery
 11778 most common mutation (poor prognosis)
 association with ECG cardiac conduction abnormalities, especially preexcitation syndrome
 association with dystonia and MS-like illness

Relative to generalised myasthenia, ocular myasthenia is:


more responsive to steroids
more responsive to anticholinesterase drugs
more commonly associated with Graves' disease
less commonly associated with thymoma
Your answer was INCORRECT
Explanation
Generalised myasthenia (compared to ocular):
 more auto-immune disease correlations e.g. Graves
 more commonly associated with thymoma
 more likely to be anti-ACh receptor positive
 more likely to have positive nerve conduction studies
 more likely to have remission induced by thymectomy

Which type of nystagmus is correctly paired with its localising lesion:

dissociated: posterior fossa

downbeat: cerebellopontine angle

convergence-retraction: craniocervical junction

gaze-evoked: suprasellar

Your answer was INCORRECT


Explanation
Convergence-retraction nystagmus is a feature of Parinaud's dorsal midbrain syndrome, and is
caused by lesions of the dorsal midbrain such as pinealomas.

Dissociated divergence nystagmus is a feature of posterior fossa lesions.

Downbeat nystagmus is caused by craniocervical junction tumours, Wernicke's, drugs (e.g.


alcohol, lithium, phenytoin) and paraneoplastic syndromes (e.g. small cell lung cancer)

Upbeat nystagmus is caused by posterior fossa lesions, drugs and Wernicke's.

See-saw nystagmus is caused by parasellar/chiasmal lesions.

Gaze-evoked nystagmus can be: physiological, congenital motor nystagmus, posterior fossa
(brainstem/cerebellar)

The test that correlates most closely with the pathophysiology underlying the Pulfrich
phenomenon is:

ERG

VEP

fluorescein angiography

electronystagmography

EOG
Your answer was INCORRECT
Explanation
The Pulfrich phenomenon reflects delayed conduction in the demyelinated optic nerve (in optic
neuritis) and occurs where oscillating objects viewed binocularly and moving in one plane (e.g. a
pendulum) appear to have three-dimensional movement.

The delayed implicit time in the VEP is the electrophysiologic correlate of the bizarre perception
known as the Pulfrich phenomenon

All are true of a horopter EXCEPT:

objects on the horopter are seen stereoscopically

imaginary plane in external space

all points stimulate corresponding retinal elements

includes the point of fixation in binocular single vision

Your answer was INCORRECT


Explanation
Objects on the horopter stimulate corresponding retinal elements and are seen singly and in one
plane. Objects just in front and behind the horopter (in Panum's fusional space) are seen
stereoscopically by stimulating slightly non-corresponding retinal points, with the discrepancy
used to create the perception of depth.

A patient has bilateral, simultaenous optic neuritis with a severe reduction in visual acuity. An
MRI reveals a T-2 signal abnormality of the spinal chord.

Which of the following tests is MOST likely to be positive?

anti-aquaporin 4 antibodies

anti-dsDNA

anti-ACh receptor antibodies

anti-voltage gated calcium channel antibodies

anti-GQ1b antibodies

Your answer was INCORRECT


Explanation
The features in this case suggest neuromyelitis optica (Devic's disease).

Devic's is characterised by:

 optic neuritis (often severe, and may be bilateral)


 transverse myelitis (T-2 lesions on MRI extending over 4 contiguous vertebral sections)
 positive for anti-aquaporin 4 antibodies

Of the other blood test options provided above:


 anti-GQ1b == Miller Fisher syndrome
 anti-voltage gated calcium channel antibodies == Lambert Eaton myasthenic syndrome
 anti-dsDNA == SLE
 anti-ACh receptor == myasthenia gravis

All of the following drugs should be avoided in myasthenia gravis EXCEPT:

ibuprofen

gentamicin

ciprofloxacin

chlorpromazine

suxamethonium

Your answer was INCORRECT


Explanation
NSAID's are generally considered safe in myasthaenia gravis. In terms of antibiotics, penicillins
and tetracyclines are also generally safe.

Drugs to avoid in myasthenia gravis:

 aminoglycosides (e.g. gentamicin, streptomycin)


 fluoroquinolones (e.g. ciprofloxacin, levofloxacin)
 macrolides (erythromycin, azithromycin)
 neuromuscular junction blockers (e.g. suxamethonium, botox)
 beta blockers
 chlorpromazine
 respiratory suppressants (e.g. morphine)
 procainamide
 penicillamine

What is the most common cause of seesaw nystagmus:

glioblastoma multiforme

eppendymoma

craniopharyngioma
pontine glioma

congential achiasma

Your answer was INCORRECT


Explanation
See-saw nystagmus is caused by chiasmal lesions, with craniopharyngioma being the most
common.

Optociliary shunt vessels are found in all of the following conditions EXCEPT:

optic nerve glioma

sphenoid wing meningioma

chronic open angle glaucoma

chronic papilloedema

central retinal artery occlusion

Your answer was INCORRECT


Explanation
Collateral vesssels (optociliary shunt vessels) are a feature of central retinal vein occlusion (not
CRAO).

Causes of optociliary shunt vessels:

 optic nerve glioma


 optic nerve meningioma
 sphenoid wing meningioma
 central retinal vein occlusion
 longstanding primary open angle glaucoma
 chronic papilloedema

Which is TRUE regarding idiopathic intracranial hypertension?

abducens palsy occurs in 40%

headache is universal

opening pressure and CSF protein levels are typically elevated

many cases are suspected to be due to decreased reabsorption of CSF

Your answer was INCORRECT


Explanation
Headache is not universal in IIH. Neurologic abnormalities are usually absent except for
occasional abducens paralysis (and of course papilloedema). Although the opening pressure is
elevated, CSF analysis is normal.

Idiopathic intracranial hypertension is characterised by:

 raised opening pressure over 250mmHg


 normal brain imaging (or ventricles slightly small or empty sella)
 normal CSF analysis

A 46-year-old lady is diagnosed with otitis media. She presents a day later with right-sided facial
pain and weakness. She has limited abduction of her right eye.

What is the most likely site of pathology?

cavernous sinus

lower pons

orbital apex

upper pons

petrous apex

Your answer was INCORRECT


Explanation
The features described are consistent with Gradenigo's syndrome, which is caused by localised
infection or inflammation of the petrous apex. Features include trigeminal neuralgia (5th nerve),
sixth palsy, seventh palsy and otitis media

A 65-year-old woman has intermittent diplopia for 2 years, worse in the evenings. She is
systemically well with no other symptoms. Initially, the examination seems normal but when the
patient is asked to sustain upgaze, she has difficulty after 30 seconds.

All of the following statements are true EXCEPT:

she is more likely to develop dysthyroidism than an otherwise normal person her age

she is unlikely to develop systemic muscular weakness

a negative acetylcholine receptor antibody is helpful for excluding myasthenia gravis

MRI brain is unnecessary

Your answer was INCORRECT


Explanation
Fatigueable ophthalmoplegia is virtually pathognomonic for myasthenia gravis. The patient has no
systemic weakness and thus has ocular myasthenia gravis. Acetylcholine receptor antibodies with
no systemic involvement are only found in about 60% of patients. Dysthyroidism is more common
in patients with myasthenia gravis. Tensilon testing, not MRI, should be obtained. When a patient
has isolated ocular myasthenia gravis for more than 2 years, there is a less than 20% chance that
he or she will develop systemic disease.

Latent nystagmus is characterised by all of the following statements EXCEPT:

null point in adduction

increases with monocular occlusion

fast phase to the fixing eye

is often associated with intermittent exotropia

Your answer was INCORRECT


Explanation
Latent nystagmus is associated with disruption of early binocular visual development (e.g.
congenital esotropia and congenital monocular cataracts), but it is not associated with acquired
strabismus (e.g. intermittent XT or accommodative ET).

Characteristics of manifest-latent nystagmus:

 bilateral
 becomes manifest when one eye occluded, blurred or intermittently suppressed
 jerk-type nystagmus
 null point in adduction
 fast phase towards fixing eye
 face turn towards the fixing eye dampens nystagmus
 associated with interruptions to binocular development: congenital esotropia, but also
monocular congenital cataracts

A 39-year-old with binocular distant diplopia, unilateral hearing loss, facial nerve palsy and
decreased corneal sensation is most likely to have:

Gradenigo syndrome

a cerebello-pontine angle tumour

Ramsay Hunt syndrome

dorsal midbrain disease

Your answer was INCORRECT


Explanation
This case demonstrates a fifth, sixth, seventh and eighth nerve palsy, which is most suggestive of
CPA tumour.

Gradenigo syndrome is due to extradural abscess at the petrous apex complicating otitis media. It
affects 5, 6, 7 and 8, but causes facial pain in the distribution of the fifth nerve.

Ramsay Hunt syndrome involves the ophthalmic division of the fifth and seventh nerves

A Goldman visual field shows a right central scotoma and a left superotemporal defect.

Where is the lesion located?

right posterior chiasm

left anterior chiasm

left posterior chiasm

right anterior chiasm

Your answer was INCORRECT


Explanation
This is a junctional scotoma. The lesion is compressing the right optic nerve and the right anterior
chiasm (producing a corresponding left superotemporal defect).

A synoptophore can do all of the following EXCEPT:


measure the degree of torsion in all cardinal positions
measure fusional vergence
measure abnormal retinal correspondence
measure the degree of stereopsis

Your answer was INCORRECT


Explanation
The synoptophore determines crudely whether stereopsis is present or not, but it does not measure the precise
angle of arc. This requires another test such as the Lang, Frisby or TNO.

The synoptophore can measure:


 angle kappa
 subjective and objective vertical, horizontal and torsional misalignments in all positions
 simultaneous perception
 sensory fusion
 motor fusion
 the presence of stereopsis
 suppression
 eccentric fixation
 abnormal retinal correspondence
 the potential for binocular single vision in the presence of squint

All are true of the oculomotor nerve parasympathetic fibres EXCEPT:

they follow the inferior division of the third nerve after its bifurcation in the cavernous sinus

they originate from the Edinger-Westphal nuclei in the midbrain

they synapse in the ciliary ganglion

they are transmitted via the long ciliary nerve to the iris sphincter

Your answer was INCORRECT


Explanation
The parasympathetic fibres are transmitted via the short ciliary nerve to the iris sphincter.

A 24-year-old woman complains of visual deterioration over the past 10 days. Her acuities are 6/6
right eye and 6/60 left eye. Ishihara plates are 10/11 right eye, 1/11 left eye. Visual fields show a
centro-coecal scotoma in the left eye, full right eye. Her left optic disc appears mildly swollen,
hyperaemic, with telangiectatic surface vessels. There is no leak on fluorescein angiography.

All of the following are true of this condition EXCEPT:

she can expect no improvement in the left eye over time

a headache typically accompanies the onset of visual loss

the right eye is likely to be similarly affected within several months

she should be counseled to avoid tobacco and heavy consumption of alcohol

diagnostic evaluation must include electrocardiography

Your answer was CORRECT


Explanation
The features described in this case are typical of Leber's hereditary optic neuropathy. Optic
neuritis is clearly in the differential but the following features work against optic neuritis:

 the presence of disc swelling and hyperaemia (while optic neuritis is typically
retrobulbar)
 no disc leakage on FFA (which would be expected in optic neuritis)

LHON is characterised by:


 mitochondrial inheritance
 typically painless vision loss
 males more commonly affected (80% of cases)
 age of onset usually 10-30 years
 10% of female carriers are affected
 50% of male carriers are affected
 generally sequential bilateral optic nerve involvement
 smoking and alcohol appear to be risk factors for vision loss
 acute: optic disc swelling, hyperaemia, dilated telangiecactic vessels but no leak on FFA
 chronic: optic atrophy
 a small percentage have partial or complete recovery of vision
 type of mutation most important prognostic factor for recovery
 11778 most common mutation (poor prognosis)
 association with ECG cardiac conduction abnormalities, especially preexcitation
syndrome
 association with dystonia and MS-like illness

You might also like